Respiratory: Exam 1

Ace your homework & exams now with Quizwiz!

What are the two components to hyper-responsiveness? Define each component.

1) Hyper-SENSITIVITY: *Normal response to abnormally LOW levels* of stimulation 2) Hyper-REACTIVITY: *Exaggerated response to normal-high levels* of stimulation

TEST: What are the five major causes for hypoxemia?

1) Hypoventilation (increased PaCO2) 2) Decreased PiO2 (high altitude) 3) V/Q mismatch (< 1) 4) Shunt 5) Diffusion limitation (decreased DLCO)

What are the four types of hypoxia? BRIEFLY describe what each type is due to

1) Hypoxemia - Blocked airways 2) Anemic hypoxia - Loss of transport capacity 3) Hypoperfusion hypoxia - Circulatory ischemia 4) Histotoxic hypoxia - Poisoning

What are two models for assessing the clinical probability for a PE? (2 scores)

1) Well's score 2) Geneva score - *< 2 = low probaility*

TEST: List at least 4 major attachment points for the diaphragm

1) Xiphisternal junction (anteriorly) 2) Costal CARTILAGES of ribs 6-10 (anterior) 3) Ribs 10-12 on the back 4) L1-L2 vertebral bodies (via CRURA)

TEST: List the two leukotriene receptor antagonists

1) ZafirLUKAST 2) MonteLUKAST

Airway resistance is the resistance to flow of air in the airways and is due to TWO areas of friction. What are they?

1) internal friction between gas molecules 2) friction between gas molecules and the walls of the airways

List four factors that can influence the affinity of hemoglobin for oxygen

1) pH 2) PCO2 3) Temperature 4) 2,3-BPG

TEST: What are the 1st gen and 2nd gen H1 receptor antagonists (antihistamine)? What is the major difference between these two generations?

1st gen: Diphenhydramine (Benadryl) - Much more *SEDATING as it PENETRATES the CNS, and SHORTER acting* 2nd gen: - Loratidine (Claritin) - Cetirizine (Zyrtec)

This non-invasive test can help you Dx cystic fibrosis, where a normal value is around 60 mEq/L in children and *around 80 mEq/L in adults*

2 positive SWEAT CHLORIDE TESTs

What is the relationship between 2,3-BPG and oxygen affinity of hemoglobin?

2,3-BPG favors the DE-OXYGENATED form of hemoglobin, so it REDUCES hemoglobins affinity for O2!!

TEST: Resistance against Linezolid is due to point mutations of the ____ _____

23S rRNA (requires two or more copies with mutations)

Cufuroxime is a (2nd/3rd) gen cephalosporin with activity against what bacterial species?

2nd gen, against HAEMOPHILUS INFLUENZAE

The ALVEOLAR period is from ____ weeks to ___.

32 weeks to 8 years Breathing movements occur just before birth 90-95% survival rate

*****RECALL: What is the general structure of hemoglobin? Which chromosomes house the genes of the hemoglobin?

4 heme-globin subunits, 2α and 2β *Housed at the 16 and 11 chromosome* respectively

The surfactant pool is around ___ - ____ in PRETERM

4-5mg/kg

The dome of the diaphragm extends up to the ___ ICS on the right side and the ___ rib on the left side

4th ICS on the right side (it's HIGHER!) 5th rib on the left side

TEST: The horizontal fissure of the R lung lies around the level of the ___ rib

4th rib

What is a normal A-a gradient range?

5-10 mmHg

Generally, most COPD pts present in the ___ decade with a ____ cough or ___ ___ ___

5th decade with PRODUCTIVE cough or ACUTE CHEST ILLNESS

Chest wall tends to expand in volume up to about ____% of the total lung capacity (pneumothorax).

75

When checking inspiration, the diaphragm should be at the __ ___ posterior rib or at the __ ___ anterior rib

8-10 POSTERIOR rib 5-6 ANTERIOR RIB

RECAP: If the FEV1/FVC ratio is less than ____%, then you probably have an obstructive defect

< 70%

TEST/AGAIN: The FEV1/FVC ratio must be what to Dx an obstructive disorder?

< 70%!!

A patient with an FEV1/VC % ratio = 68% they have ____. We then look at the FEV1 and it is at 89%, this makes a patient with a ___ ____ defect

<70 = OBSTRUCTION Then go to the FEV1 = 89% = MILD obstructive defect

If the TLC is < 80% then there is a ____ ventilatory defect. If the TLC is > 80% predicted but the FV is <80% then the pt ___ ___ ___ a restrictive defect If the TLC is >120% predicted then there is an ___ ___ ___.

<80 = RESTRICTIVE >80 TLC/<80 FVC = NO restrictive defect TLC>120 = OBSTRUCTIVE ventilatory defect

TEST: In a person with a LOW risk of TB, an induration of ___ mm and above indicates latent TB

>15mm induration

The ventilation system in humans can be divided into two main zones. What are they, and what parts does each zone include?

1) CONDUCTING zone: Trachea to bronchioles 2) RESPIRATORY zone: Bronchioles to alveoli

TEST: What are the 3 main types of obstructive lung disease? List any subtypes if present.

1) COPD - Emphysema - Chronic bronchitis 2) ASTHMA - *reversible* airflow obstruction 3) BRONCHIECTASIS (dilation of bronchi)

RECAP: List whether each of the following increases susceptibility to TB: 1) Coal Workers Pneumonia 2) Silicosis 3) Asbestos-related diseases

1) CWP - NO 2) Silicosis - YES 3) Asbestosis - NO

What are three ways to measure functional residual capacity (FRC)?

1) Helium dilution (closed-circuit) 2) Nitrogen washout (open-circuit) 3) Body Plethysmography (body box)

RECAP: List the two major things in RBCs that are vital for gas transport

1) Hemoglobin 2) Carbonic anhydrase

List four species of fungi that itraconazole is effective at treating

1) Histoplasmosis 2) Blastomycosis 3) Sporotrichosis 4) Paracoccidioidomycosis

TEST: List the two key features of the alveoli that make it a very efficient gas exchanger

1) Huge absorptive surface 2) Minimal air-to-blood diffusion distance!

What are some types of bronchodilators?

Beta 2 agonists (LABAs, SABAs) Anticholinergics PDE inhibitors

Ipratropium can be used in combination with this class of anti-asthmatics, prolonging the duration of it

Beta2 agonists!

RECALL: Where exactly are the oxygen binding sites in hemoglobin?

Between the Fe in HEME and the histidine imidazole groups in GLOBIN

What are some of the characteristic findings on physical exam for interstitial lung disease?

Bibasilar rales *Clubbing of fingers (rare in sarcoidosis)* Signs of pulmonary HTN and cor pulmonale - advanced progression *Rales/Crackles*

RECALL: The thyroid cartilage is big in the (front/back) and small in the (front/back). What attaches to the posterior surface of the thyroid cartilage?

Big in the FRONT, small in the BACK The VOCAL CORDS attach to the posterior surface of the thyroid cartilage!

What are some of the common findings of ACUTE hypersensitivity pneumonitis on high resolution CT?

Bilateral AIR SPACE CONSOLIDATION (opacities) with a GROUND GLASS opacity - Also, small ill-defined centrilobular NODULES

TEST: Binding of O2 to Hb (increases/reduces) the affinity of Hb for CO2. What is this phenomenon called?

Binding of O2 to Hb REDUCES the affinity of Hb for CO2 - Called the HALDANE EFFECT (deoxyhemoglobin shows an increased capacity to bind and carry CO2)

TEST: MOA of Rifampin Is it bactericidal or bacteriostatic?

Binds to DNA-dependent RNA polymerase, prevents TRANSCRIPTION BACTERICIDAL

RECALL: What does Heparin do?

Binds to antithrombin III and inactivates XII, XI, X, and II

Histamines are biological _____ found throughout the periphery (such as where?) and in select areas of the ______. They are an ________ mediator, along with _______ and ________

Biological AMINES found throughout periphery (skin, bronchioles, intestinal mucosa) and select areas of the BRAIN (posterior basal hypothalamus and reticular formation) They are INFLAMMATORY mediators, along with PROSTAGLANDINS and LEUKOTRIENES

CWP - Coal workers pneumoconiosis is also called ___ ___ __ and has what findings? Simple: Complicated:

Black lung disease Asymptomatic Anthracosis SIMPLE cwp: little pulm dysfunction COMPLICATED: progressive massive fibrosis

An air filled sac with NO vasculature is known as a ____

Bullae (left side shows no vasculature)

One advanced generation cephalosporin is _______, which inhibits cell wall synthesis by interfering with _________ cross-linking. It has a high affinity to _____-associated ______ (binding protein). Bacteria that express _____ __-_______ greatly reduces the activity of this drug.

CEFTAROLINE, inhibits cell wall synthesis by interfering with PEPTIDOGLYCAN cross-linking. It has a high affinity to MRSA-associated PBP2a Bacteria that express AmpC Beta-lactamase reduces activity of Ceftaroline

A CF gene mutation can lead to recurrent ________, which can then progress into ___________ and finally into chronic _________ failure

CF gene mutation --> recurrent BRONCHITIS --> BRONCHIECTASIS (CF) --> Chronic RESPIRATORY FAILURE

With the (lung/chest wall), as you increase in pressure you decrease in volume

CHEST WALL - Opposite for lung (increase in pressure, increase in volume)

AGAIN: Inhaled corticosteroids are for (acute/chronic) asthma exacerbations

CHRONIC

Classically, Bird-Fanciers lung is a form of (acute/chronic) hypersensitivity pneumonitis with the most responsible agents being what?

CHRONIC, as it is due to chronic exposure to the birds / down comforter (which is what bird BREEDERS do) - Most responsible agent = Avian proteins and urine and feces

TEST: 90% of COPD is caused by what?

CIGARETTE smoking! - Pipe and cigar smokers have lower mortality than cigarette smokers because of the acidity of the smoke <25% of smokers will NEVER develop COPD

A series of x-ray views taken from many different angles which are then processed to create cross-sectional images is called a ___ This type of imaging can be used for ___ and____

CT (computerized tomography) Used for phones and soft tissue

TEST: What is the imaging of choice for many clinicians for making the diagnosis of PE? If the patient cannot tolerate this imaging modality, what is your next choice?

CT angiogram! (95% sensitive) However, if the patient cannot tolerate the contrast dye, the next choice is a VENTILATION PERFUSION SCAN (V/Q scan)

****TEST: If a PE is likely after assessing the clinical probability, then you go straight to doing a ____ ______ which will _____ the presence of a PE

CT angiogram, which will CONFIRM the presence of a PE!

Voriconazole predominantly inhibits CYP (3A4/2C19)

CYP2C19

TEST: What is the black box warning with systemic corticosteroids?

Can lead to DEATH when switching from chronic SYSTEMIC steroids to INHALANTS

TEST: Systemic corticosteroids can suppress the _____ axis, which if stopped abruptly can lead to what?

Can suppress the HPA axis (hypothalamic-pituitary-adrenal axis), as corticosteroids negatively feedback on the hypothalamus and anterior pituitary. When stopped abruptly, it can lead to ADRENAL INSUFFICIENCY and ultimately DEATH as the body can't make up for the lack of steroids at a good rate

What kind of fungal infections is amphotericin effective to treat?

Candidiasis Cryptococcal meningitis Invasive aspergillosis Zygomycosis Coccidioidomycosis Blastomycosis Histoplasmosis

List some of the side effects of systemic corticosteroids

Candidiasis (Thrush) Bone resorption Osteonecrosis (femoral head most often) Increased infections Mental disturbances, insomnia Peptic ulcers

In a patient with TB we can see?

Cavitary Lesion

TEST: MOA of Ethambutol Is it bactericidal or bacteriostatic?

Cell WALL inhibition by inhibiting ARABINOSYLTRANSFERASE BACTERIOSTATIC!

List which TB drug targets: - Transcription - Cell wall synthesis - Cell membrane

Cell WALL: INH and Ethambutol Cell MEMBRANE: Pyrazinamide Transcription: Rifampin

Sarcoidosis has the highest incidence in what two populations?

African Americans and Northern Europeans (Swedish population)

After the lysosomal enzymes kill and digest the microbe in the fused ________, it leaves a _______ _____ that will empty its contents outside the cell by _________

After the lysosomal enzymes kill and digest the microbe in the fused PHAGOLYSOSOME, it leaves a RESIDUAL BODY that will empty its contents outside the cell by EXOCYTOSIS

How does mycobacterium tuberculosis evade normal macrophage antimicrobial mechanisms?

After they get engulfed into a phagosome, mycobacterium PREVENTS MATURATION of the phagosome so that it resembles early ENDOSOMES. This prevents phagosome fusion with the macrophage's lysosome!

TEST: List some of the side effects of leukotriene receptor antagonists/ synthesis inhibitors

Agitation, aggression, tremors, anxiety, restlessness, insomnia, hallucinations, depression, suicidal thoughts

What occurs at the conducting zone of the ventilation system?

Air TRANSPORT, WARMING, HUMIDIFICATION, and particle FILTRATION *anatomic dead space*

A pneumothorax is an accumulation of ___ in the ___ ___ The types are 1- 2- 3-

Air in the plural space types: Spontaneous Iatrogenic Traumatic

RECALL: What are the ABCDEs of reading CXRs?

Airway Breathing tube Cardiac size Diaphragm Everything else

When is wheezing no longer heard?

Airway narrowing is SEVERE

Describe the chance in airway resistance depending on the lung volume

Airway resistance peaks around 2 liters and is lowest at lung volumes over 8 liters.

RECALL: What are leukotrienes?

An eicosanoid that is an INFLAMMATORY mediator *produced from arachidonic acid*

*******What change is considered a positive response to bronchodilators in a patient with a new obstructive defect?

An improvement in FEV1 or FVC by *at least 12%, or 200mL from baseline*

What muscarinic antagonist is used to dilate the pupils during an eye exam?

Atropine!

TEST: List the equation for calculating the DLCO from the volume and pressure of CO in alveolar gas

DLCO = Volume of CO in alveolar gas / Partial pressure in alveolar gas

TEST: During inspiration, the diaphragm (relaxes/contracts) and both the ______ _____ pressure and the pressure inside the ______ decrease

Diaphragm CONTRACTS, and both the PLEURAL CAVITY pressure and the pressure inside the LUNGS decrease (Pi)

TEST: Define the diffusing capacity and describe what its significance is

Diffusing capacity determines the overall *ability of the lung to transport gas into and out of the blood* - Significance: By evaluating the diffusing capacity of the lung for carbon monoxide (DLCO), it *provides a quick way for evaluating RESTRICTIVE and OBSTRUCTIVE lung disease*

What index on spirometry measures the ability of lungs to transport inhaled gas from alveoli to pulmonary capillaries?

Diffusion Capacity of lungs for CO (DLCO) *gas exchange capabilities of the lung*

Centriacinar emphysema affects the respiratory (alveoli/bronchioles), while panacinar emphysema affects the respiratory (alveoli/bronchioles)

Centriacinar: Affects BRONCHIOLES Panacinar: Affects ALVEOLI (and ducts) Panacinar emphysema extends all the way to the terminal blind alveoli, whereas centriacinar stays centrally (before the alveoli)

Simple CWP, if left untreated, can develop into what?

Centrilobular emphysema

The opening of the nasal cavity to the pharynx can be seen posteriorly by what structure?

Choanae

The typical chronic bronchitis patient is described as a _____ ________, while the typical emphysema patient is described as a _____ ________

Chronic bronchitis patient: BLUE BLOATER Emphysema patient: PINK PUFFER

TEST: A 55 year old man with an *increasingly persistent cough that has lasted for several years presents with complaints of wheezing and chest tightness that is worse than usual. He is a chronic two pack a day smoker for 20 years.* The cough produces an excessive THICK, WHITE mucus. Lab values showed elevated RBC counts and normal WBC counts. *Blood gases showed low O2 and high CO2. PFTs show a marked decrease in FEV1*. CXR shows hyperinflation, depressed diaphragms, and an increased retrosternal space. What is top on your differential?

Chronic bronchitis!

TEST: What two symptoms can distinguish between chronic and acute hypersensitivity pneumonitis?

Chronic usually LACKS a FEVER or CHILLS

List what you would normally see on CXR for a patient with Caplan's Syndrome?

Circular opacities bilaterally distributed at the periphery of the lungs (appear in crops)

TEST: List the classic triad of symptoms of asthma, and list a symptom that is very classical of asthma and may often be the only symptom in patients.

Classic triad: 1) Persistent wheezing 2) Chronic episodic dyspnea 3) Chronic NONproductive cough Really classical symptom: NIGHT-TIME COUGH

What is a common physical finding of bronchiectasis?

Clubbing of the fingers (Pulmonary osteoarthropathy)

****What are some things that occur in the late stage of silicosis?

Coalescing nodules may cavitate, and regional lymph nodes may calcify *(EGGSHELL CALCIFICATION)* Just like in complicated CWP, it can also cause *progressive massive fibrosis*

On a CT, if you find characteristics of UIP you __ ___ need to biopsy

DO NOT neet to biopsy

TRUE or FALSE: The terminal bronchioles are the same as the respiratory bronchioles

FALSE! The terminal bronchioles are a part of the LOWER CONDUCTING portion, while the respiratory bronchioles are even smaller and are a part of the RESPIRATORY portion

TRUE or FALSE: The bronchi contains cartilage rings like the trachea does

FALSE! Their "cartilage" rings has been replaced by irregularly shaped cartilage PLATES

TRUE or FALSE: The true vocal cords contain numerous serous glands

FALSE! They exist throughout the lamina propria of the larynx EXCEPT at the true vocal cords!

TEST/TRUE or FALSE: CWP may increase the susceptibility to tuberculosis

FALSE!!!!!

TRUE or FALSE: A Hampton's Hump and Westermark sign is diagnostic of PE

FALSE, they are SUGGESTIVE but NOT diagnostic

TRUE or FALSE: Histamines are only found in mast cells

FALSE, they are also located in basophils, in *enterochromaffin-like cells in the stomach*, and in brain tissue MAINLY in the MAST cell

TRUE or FALSE: Type III hypersensitivity reactions do not involve antibodies

FALSE, COMPLEMENT activation occurs by SOLUBLE antigen:antibody complexes, usually IgG (ADCC)

TRUE or FALSE: Fluconazole is metabolized by CYP450 enzymes in the liver

FALSE, Fluconazole is EXCRETED UNCHANGED

TRUE or FALSE: The partial pressure of CO2 in the dead space is greater than that in the alveoli

FALSE, PCO2 is much lower in the dead space because there is no gas exchange with the blood, so the CO2 that is in the blood can't get into the dead space (but can be exchanged in the alveoli!)

TRUE or FALSE: Salmeterol can be administered orally or by inhalation

FALSE, Salmeterol can only be administered by inhalation - Albuterol however can be administered ORALLY!

TRUE or FALSE: Balanced forces at the surface generate surface tension

FALSE, UNBALANCED forces generate surface tension

TRUE or FALSE: Under normal conditions, oxygen gas exchange is diffusion limited

FALSE, although it is transferred much more slowly than something like N2O, it is still transferred fast enough so that there is normally no diffusion limitation - *So oxygen under normal conditions is PERFUSION LIMITED*

TRUE or FALSE: Idiopathic interstitial pneumonias are considered infections

FALSE, although pneumonias are often thought of as infections, this class of diseases do NOT appear to result from infection

TRUE or FALSE: Antibodies are produced and effective at controlling M. tb infections

FALSE, although they ARE produced they ARE NOT effective in controlling tuberculosis as they do not provide any protection against the disease

TRUE or FALSE: Natural killer (NK) cells are phagocytic

FALSE, although they still contribute to the innate defense system

TRUE or FALSE: Increases in either pulmonary artery or pulmonary venous pressure can increase vessel distention which increases vascular resistance

FALSE, an increase in vessel distention --> DECREASE in vascular resistance

TRUE or FALSE: An increased A-a gradient implies hypoxemia

FALSE, as A-a gradients increase with age!

TRUE or FALSE: Findings of erythema, warmth, pain, and swelling in the lower extremity is indicative of a DVT

FALSE, as these may not be present all the time in a patient with DVT

TRUE or FALSE: In asthma, you expect the FEV1/FVC ratio to be greater than 70%

FALSE, because asthma is an OBSTRUCTIVE disease you would expect *FEV1/FVC to be BELOW 70%*

TRUE or FALSE: Anemia is a type of hemoglobinopathy

FALSE, because hemoglobinopathy refers to genetic defects --> abnormal Hb STRUCTURE or development of subunit expression Anemia, on the other hand, is caused by a *DECREASED AMOUNT of hemoglobin*

TRUE or FALSE: Pleural disease is common in ILD

FALSE, but if you do find it it is suggestive of connective tissue disease

TRUE or FALSE: Viral coating inhibitors are effective against all three types of influenza ciruses

FALSE, effective just against A And really, they should be called viral UNCOATING inhibitors because they are preventing the virus from uncoating and releasing into the host cell

TRUE or FALSE: You do not want to give either doxycycline or azithromycin with food

FALSE, food does not interfere with DOXYCYCLINE absorption

TRUE or FALSE: According to Fick's Law of Diffusion, gases can diffuse through a thicker medium more efficiently

FALSE, gases can diffuse through a THINNER medium more efficiently

****TRUE or FALSE: Inspired oxygen in the blood gets to the tissues by active transport

FALSE, it *PASSIVELY DIFFUSES* across alveolar and tissue cell membranes

TRUE or FALSE: Anatomic dead space volume decreases slightly during inspiration

FALSE, it INCREASES slightly during inspiration because the airways EXPAND - And remember, anatomic dead space is the volume in the airways from the trachea to the bronchioles

TRUE or FALSE: As alveolar ventilation rate decreases, alveolar composition becomes more similar to that of inspired air

FALSE, it becomes similar to the composition of mixed systemic VENOUS blood

TRUE or FALSE: Sarcoidosis only forms in the lungs

FALSE, it can also form in lymph nodes, lungs, liver, eyes, skin, spleen, bones/joints, sinuses, skeletal muscles, kidneys, heart, repro organs, salivary glands, and the nervous system

TRUE or FALSE: Smooth muscle contraction occurs only when there is an influx of extracellular calcium through gated Ca channels

FALSE, it can also occur by calcium release from the SARCOPLASMIC RETICULUM (SR)

TRUE or FALSE: Sarcoidosis often presents with hypocalcemia

FALSE, it commonly presents with HYPERcalemia/-caliuria granulomas secrete a PTH like hormone

TRUE or FALSE: You could give cromolyn for someone who is having an asthma attack

FALSE, it doesn't immediately relieve symptoms of asthma!

TRUE or FALSE: Stimulating H2 receptors decreases gastric acid secretion

FALSE, it increases it! - Therefore, H2 antagonists are used to INHIBIT acid secretion

TRUE or FALSE: Beryllium is an organic dust

FALSE, it is a MINERAL dust

TRUE or FALSE: Since O2 is the final electron acceptor in the ETC, the PO2 in mitochondria is high

FALSE, it is actually about 2 mmHg in the mitochondria because the oxygen is constantly being hydrogenated --> WATER (so it can generate ATP!)

TRUE or FALSE: The Red-Man syndrome seen with vancomycin is an allergic reaction

FALSE, it is direct toxicity to the mast cells that is causing them to release histamines (not due to an allergen)

TRUE or FALSE: The alveolar membrane across which oxygen and CO2 diffuse is thicker than the interalveolar septum

FALSE, it is half as thick since it only consists of thin cytoplasm of the type I alveolar cell

TRUE or FALSE: Azithromycin is mainly used to treat CF as an antibiotic

FALSE, it is mostly used to treat the INFLAMMATION

TRUE or FALSE: Prednisone is not safe during pregnancy

FALSE, it is safe during pregnancy because the fetal barrier can convert the ACTIVE form (prednisolone) back to the inactive form (prednisone)

TRUE or FALSE: The alveolar dead space occurs in zone 2

FALSE, it occurs in zone 1 where the collapse of vessels are more likely leading to no blood flow and thus, no gas exchange at those alveoli

TRUE or FALSE: Albuterol takes about 30 minutes for it to start working

FALSE, it takes about 5 minutes (remember, Albuterol is SHORT-acting)

TRUE or FALSE: Cystic fibrosis is a kids disease

FALSE, it used to be but now there are more adults with CF than kids (part of that has to do with kids surviving longer)

TRUE or FALSE: Unlike turbulent flow, laminar flow can be heard with a stethoscope

FALSE, laminar flow = SILENT - Turbulent flow can be HEARD

TRUE or FALSE: Larger inhaled particles are more likely to appear in pulmonary fluids

FALSE, larger particulates tend to persist in the lung TISSUE for years

TRUE or FALSE: Early stage silicosis presents with shortness of breath

FALSE, late stage silicosis does

TRUE or FALSE: Myoglobin has a higher P50 than hemoglobin

FALSE, myoglobin has a *LOWER P50 than hemoglobin as myoglobin has a HIGHER AFFINITY for oxygen*

TRUE or FALSE: Breathing movements occur only after birth

FALSE, natural breathing motions occur in the womb so they can prepare the muscles for breathing outside of the womb

TRUE or FALSE: Itraconazole penetrates into the CNS

FALSE, neither the parent drug nor its active metabolite can penetrate into CNS

*******To make a Dx of an obstructive defect, what ratio do you look at and what does the ratio have to be?

FEV1/FVC ratio LESS than 70% predicted!!

TEST: What is the clinical PFT value required to diagnose an obstructive lung disease?

FEV1/FVC ratio < 70% FEV1 is the forced expiratory volume in the first second, and this should be DECREASED as the air is trapped inside the alveoli and cannot leave during expiration.

In all stages of COPD, what PFT index criteria remains the same, and which one changes with different stages?

FEV1/FVC ratio criteria stays the same, at < 70% (obstructive disease) The FEV1 changes with the different stages: - Stage I: Greater than 80% - Stage II: 50-80% predicted - Stage III: 30-50% predicted - Stage IV: <30% predicted, OR <50% predicted + chronic respiratory failure/right HF

Complicated CWP can result in a possible progressive massive ______

FIBROSIS Complicated CWP is actually called Progressive Massive Fibrosis (PMF)

Fetal hemoglobin has a (greater/lesser) affinity for oxygen relative to maternal hemoglobin. Fetal hemoglobin shows a dissociation curve similar to that in adults when there is no _______ added. Why?

Fetal hemoglobin has a *GREATER affinity for oxygen relative to maternal hemoglobin* Fetal hemoglobin shows a dissociation curve similar to that in adults when there is no 2,3-BPG added. This is *because fetal hemoglobin is INSENSITIVE to 2,3-BPG!!* (so it shows the shape of an adult Hb with NO 2,3-BPG, which increases the affinity for O2)

What is the most common method for obtaining a diagnosis of sarcoidosis?

Fiberoptic bronchoscopy (biopsy) Cannot do this on Hilar Lymph nodes so we use *EBUS* *Sarcoidosis is a Dx of EXCLUSION*

What are the 4 components of a berylliosis granulomatous inflammation?

Fibroblasts Lymphs Histiocytes GIANT cells

In sarcoidosis what things are seen on the image?

Fibrosis Adenopathy *Hilar adenopathy*

**What virulence factor does Bordatella use to attach to ciliated epithelial cells?

Filamentous HEMAGGLUTININ

TEST: List what is responsible for filtering, moistening, and warming the air that is inhaled.

Filtering is done by VIBRISSAE, a special short thick hair in the nasal cavity that entrap large particulates Moistening is done by GOBLET CELLS in the sinuses Warming is done by the VASCULAR LAMINA PROPRIA, which can be attributed to the rich venous networks that can SWELL with blood.

What three categories can crackles be categorized in?

Fine Medium Coarse

List what develops from the first three divisions of the laryngotracheal tube.

First division: L and R main bronchi Second division: Secondary bronchi Third division: Tertiary bronchi

First line defenses against M. tuberculosis infections from alveolar ________ are often unsuccessful in controlling the infection, so they release _______ to induce ________ formation, which prevents the _____ of infection

First line defenses against M. tb infections from alveolar MACROPHAGES are often unsuccessful in controlling the infection, so they release TNF-alpha to induce GRANULOMA formation, which prevents the SPREAD of infection

TEST: The first test you would do in diagnosing ILD is _______, and the next test you would get is what?

First test: CXR Second test: High resolution CT

A spirogram often involves what two graphs/relationships?

Flow-volume Volume-time

The larynx is formed by irregular shaped ___ and ____ cartilages. This is an organ of ____

Hyaline and elastic cartilage Organ sound

What causes the increased airway resistance in asthma?

Hyper-reactive airways that CONSTRICT in response to STIMULI.

Obstructive lung disease can ultimately progress to what?

Hypercapnic respiratory failure (too much CO2 due to obstruction!)

TEST: The prognosis of hypersensitivity pneumonitis is very (good/poor)

GOOD! Majority of patients experience near total recovery

TEST: What glomerulonephritic disease is an example of a type II hypersensitivity reaction? What is the "antigen" in this example?

GOODPASTURE'S Syndrome (anti-GBM) - Antigen = glomerular basement membrane (GBM)

Hb Changes from the tissue to the lung....because of the presence of? What effect does this have?

H+, CO2 and Low O2 Changes the effect in AFFINITY of Hb for O2

H. influenzae are gram (positive/negative) bacteria (with/without) a capsule (careful, trick question)

H. influenzae are gram NEGATIVE bacteria that MAY or MAY NOT be encapsulated: - Encapsulated species --> meningitis and sepsis - Non-encapsulated --> upper respiratory tract infections (NON-INVASIVE)

TEST: List the common side effects of Pyrazinamide

Hepatotoxicity Hyperuricemia ARTHRALGIAS (shoulders especially)

TEST: Spirometry is used to classify patients as having one of what three patterns?

Normal Airflow obstruction Airflow restriction

A normal D-Dimer means your chance of having a PE is ___

Normal = low risk (95% predictive value)

TEST: What are the common CXR findings for a patient with emphysema?

Normal heart size and lung parenchyma, a widened RETROSTERNAL airspace, and LOW FLAT DIAPHRAGM

COPD is characterized by airflow obstruction that is not fully ________

Not fully REVERSIBLE

Zanamivir is not recommended in patients who have what? Why?

Not recommended in patients who have an UNDERLYING airway disease like COPD, because it can cause WHEEZING and BRONCHOSPASM

What are the three types of air flow?

Note: Breath sounds heard with a stethoscope reflect the turbulent airflow.

List some of the major classification of interstitial lung disease

Occupational/environmental (like asbestos) Drugs and poisons Connective tissue disease Idiopathic Infection IATROGENIC

TEST: The olfactory zone contains an outer layer of olfactory ________ and an inner layer of ________ _______ that contains ________ glands

Olfactory zone contains: - Outer layer of olfactory EPITHELIUM - Inner layer of LAMINA PROPRIA (submucosa), contains BOWMAN'S glands

Briefly describe how the QuantiFERON TB gold test works

One time blood draw, and the blood is treated with M. tb antigens. The supernatants is collected and measured for IFN-gamma - Production of IFN-gamma indicates previous exposure to M. tb

TEST: Only the physically (bound/dissolved) oxygen contributes to the partial pressure of O2 in blood, but the partial pressure determines how much combines with ________

Only the physically DISSOLVED oxygen contributes to the partial pressure of O2 in blood, but the partial pressure determines how much combines with HEMOGLOBIN

AGAIN: In acute exacerbations of COPD, you want to give the patient what?

Oral of IV STEROIDS!

TEST: What are the two layers that make up the pleural cavity? What is contained within the pleural cavity?

Outer PARIETAL pleura Inner VISCERAL pleura Contained within the pleural cavity is PLEURAL FLUID, a serous fluid which is made of WATER!

RECALL: Arachidonic acid can be converted to either ________ or ________, and inhibition of either one will lead to (more/less) production of the other

PROSTAGLANDINS or LEUKOTRIENES, inhibiting either one --> MORE production of the other one (as it will lead to a buildup of arachidonic acid)

__________ ______ is a potential adverse reaction of Ciprofloxacin, while prolonged use can lead to a ruptured _______ _____

PSEUDOMEMBRANOUS COLITIS Prolonged use --> ruptured ACHILLES TENDON

TEST: What is the most common organism in the adult CF population to cause an infection?

PSEUDOMONAS aeruginosa

TEST: What is considered the gold standard in diagnosing a PE? This test is more ______ than a CT angiogram, but what is the downside to this?

PULMONARY angiogram This test is more SPECIFIC than a CT angiogram, but it is much more INVASIVE - This is why when you think of diagnosing PE, *you will do a CT angiogram*

What is Boyle's Law?

PV=nRT otherwise known as PV=C (the C means we are holding the Temp constant)

TEST: Pulmonary vascular resistance is _____ of systemic vascular resistance

PVR is 1/10 of SVR!

Which two cytokines are released by CD4+ Th2 cells in response to an allergen that then cause production of IgE?

IL-4 and IL-5

TEST: Neutrophils migrate to the lungs in response to what cytokine?

IL-8

TEST: Decreasing the radius of the tube will significantly (increase/decrease) the resistance of the tube to laminar flow

INCREASE!

TEST: (Increased/decreased) temperature and (increased/decreased) 2,3-BPG, relative to normal, will decrease the affinity of hemoglobin for oxygen

INCREASED temperature INCREASED 2,3-BPG

TEST: Most of the back of the lung will be the (superior/inferior) lobe

INFERIOR lobe

List the major artery that supplies the inferior surface of the diaphragm

INFERIOR phrenic A.

TEST: Side effects with (oral/inhaled) corticosteroids are minimal

INHALED (as it is more localized!)

The use of ___ _____ should NOT be used as primary therapy or monotherapy in COPD

INHALED CORTICOSTEROIDS

The respiratory epithelial cells play a role in the (innate/adaptive) immune system against respiratory pathogens

INNATE

Crackles are heard during ____ >>>>____

INSPIRATION >>>>EXPIRATION

TEST: When the V/Q ratio is greater than 1.0, the alveolar gas tension becomes more similar to that of what?

INSPIRED AIR! - V/Q > 1 means that *the alveoli is ventilating more*, so it makes sense that the alveolar gas tension is closer to that of inspired air

RECALL: Hypersensitivity pneumonitis is a form of what?

INTERSTITIAL LUNG DISEASE! From a chemical

Antihistamines bind and prevent agonist activity, these are specifically called....

INVERSE Agonists

TEST: What is the definition of bronchiectasis, and what is it characterized by?

IRREVERSIBLE dilation and destruction of one or more bronchi with inadequate clearing and POOLING OF MUCUS - Characterized by PERSISTENT MICROBIAL INFECTION and/or inflammatory response

What is the difference between inspiratory reserve volume (IRV) and inspiratory capacity (IC)?

IRV = Maximum amount of air that can be inhaled AFTER a normal tidal volume inspiration (excludes tidal volume) IC = Maximum amount of air that can be inhaled from the END of a tidal volume (includes tidal volume) - IC = IRV + TV

AGAIN**** Under what condition (PFT values) could you rule out a restrictive defect?

If the FVC is <80% BUT the TLC > 80% (TLC should also be less than 80% to be considered a restrictive defect)

When would you give oxygen to a patient who has asthma? What is the goal oxygen saturation?

If the patient ALSO has HYPOXIA! Goal: O2 sat > 90%

TEST: When would a patient with sarcoidosis not require treatment with corticosteroids?

If the patient is in stage 1 with or without erythema nodosum and with NORMAL lung function (normal or near normal PFTs) Remember: stage 1 does NOT involve the lungs! So you can have a skin finding and still be in stage 1

When is arterial blood gas (ABG) recommended in patients with asthma? Why?

If their FEV1 < 1L or PEF< 200 L/min - You want to make sure the patient doesn't have worsening pCO2 levels, as that is often a sign of HYPERCAPNIA --> RESPIRATORY FAILURE

AGAIN: How do you determine "reversibility" of obstructive disorders?

If there is a POSITIVE response to bronchodilators - Increase of FEV1 > 200mL and >12% from baseline

_______________ pressure is always smaller than alveolar pressure, if not, the lung will be Collapsed.

Intrapleural

What embryonic pharynx structure sits in between the two arytenoid swellings?

Laryngiotracheal groove!

In an asthmatic patient, the lumen of the bronchus is filled with ___ ___ and there is a ___ ____ with many inflammatory cells.

Lumen = STEAKY MUCUS WIDENED SUBMUCOSA

Over 90% of TB infections are (active/latent), but an infection that depletes _____ __ cells can lead to ________ of the infection. Name one major viral infection that can lead to this.

Over 90% of TB infections are LATENT, but an infection that depletes CD4 T-cells can lead to (ENDOGENOUS) REACTIVATION of the infection - Example: HIV infections!!

The ABCD formulation of amphotericin B causes (more/less) infusion-related reactions than C-AMB, but is (more/less) nephrotoxic than C-AMB

MORE infusion-related reactions, but LESS nephrotoxic than C-AMB

Acute hypersensitivity pneumonitis is the ___ ___ and easily recognized form. Sx's occur __-___ hours after exposure and resolve within __ -___ hours after removal from exposure

MOST CLASSIC 4-8 hours Resolves in 24-48 hrs Can be confused with VIRAL/Bacterial infection

RECALL: Neutrophils have a (single/multi)-lobed nucleus and makes up roughly how much of the WBC population? When neutrophils migrate out of circulation in response to cytokines and chemokines, it is called _______.

MULTI-LOBED nucleus (polymorpholeukocytes) and makes up 60-75% of the total WBC population DIAPEDESIS

The INTRAPULMONARY bronchus is characterized by ____ of ____

PLATES of CARTILAGE Lymph nodes = BALT = Bronchial Associated lymphoid tissue

TEST: Linezolid may cause ________, which includes what kind of disorders?

MYELOSUPPRESSION (bone marrow): - Anemia, leukopenia, pancytopenia, thrombocytopenia

Hemoglobin delivers O2 to _______ in muscle

MYOGLOBIN

Alveolar macrophages, which are (long/short) lived, differentiate from ________ that are usually present in the _______

Macrophages, which are LONG lived, differentiate from MONOCYTES that are usually present in the CIRCULATION

TEST/RECAP: What are the 5 main structures that form from the embryonic pharynx, and what arch does each structure associate with? Together, these structures of the embryonic pharynx ultimately forms what part of the adult respiratory system?

Main structures of the embryonic pharynx: 1) Epiglottis swelling - 4th arch 2) Thyroid cartilage - 4th arch 3) Arytenoid swelling - 6th arch 4) Laryngiotracheal groove/ridge - 6th arch 5) Cricoid cartilage - 6th arch Together, these form the UPPER RESPIRATORY TRACT in the respiratory system

A squeaky creaky and grating or rasping sound is a ___ ____ ____ This sound is made by ___ or ____ pleural surfaces

PLEURAL FRICTION RUB Made: ROUGH or INFLAMED pleural surfaces rubbing together during respiration

TEST: Minute ventilation rate, also called minute respiratory ________, is calculated by what equation?

Minute respiratory VOLUME (Ve' or Vt') Vt' = Vt x f Vt = Tidal volume f = Respiratory frequency, or the number of breaths per unit time (usually per minute)

TEST: When the V/Q ratio is less than 1.0, the alveolar gas tension becomes more similar to that of what?

Mixed systemic VENOUS BLOOD (in the RIGHT VENTRICLE) - V/Q <1 means *the alveoli is PERFUSING more*, and that blood came from the pulmonary artery which is from the right ventricle!

TEST: What MAIN special cell type is contained within the epithelium of the olfactory zone, and what is their main function? What are three other cell types found within the olfactory epithelium?

Modified BIPOLAR cells called OLFACTORY cells that are used for SMELLING Other cell types: - Supportive cells (closer to apical surface) - Basal cells - Brush cells

In the pseudoglandular period of lung development, there are (lots of/no) alveoli and (poor/rich) vasculature

NO alveoli and POOR vasculature

TEST: If you see a test question that says the patient has shortness of breath and a calf clot, does the patient have a PE?

NO!! We worry about THIGH CLOTS or higher, but calf clots DO NOT cause a PE (calf thrombosis is NOT a part of the DEEP VENOUS thrombosis)

Methemoglobin is a __ ___ hemoglobin.

NON-Functional Hb Hb with Fe+3

TEST: What histopathology finding is commonly seen in hypersensitivity pneumonitis (at least 67-90% of cases)

NON-caseating granulomas

TEST: Pneumoconiosis is a form of lung reaction that (is/is not) neoplastic, and is most often a reaction against inhaled __________ such as what?

NON-neoplastic lung reaction to inhaled PARTICULATES, including: - *Mineral dusts* (coal, mines, silica, asbestos) - *Organic dusts* (moldy hay, bird droppings, hemp) - *Chemical fumes and vapors* - *Urban Exposure*

What is the general goal of oxygen therapy in terms of PO2 and pH?

PO2 > 55 mmHg without drop of pH < 7.25

What is the PO2 in: - Dry air - Inspired bronchial air - Alveolar air - Systemic arterial blood - Mixed venous blood

PO2 in: - Dry air ~ 160 mm Hg - Inspired bronchial air ~ 150 mm Hg - Alveolar air ~ 100 mm Hg - Systemic arterial blood ~ 100 mm Hg - Mixed venous blood ~40 mm Hg

RECALL: What law describes resistance of laminar flow through a tube?

POISEUILLE Law: R = 8nL / (pi * r^4) n = viscosity of fluid L = length of tube r = radius of tube R = Resistance

TEST: Which azole has the widest spectrum of antifungal activity? This azole is the only agent with significant activity against what?

POSACONAZOLE - ONLY agent with significant activity against ZYGOMYCOSIS and MUCORMYCOSIS

The ___ ____ Muscle pulls medially on the muscular process, which results in the vocal process to be pulled OUTWARD. What is this m. used in?

POSTERIOR CRICOARYTENOID M. Normal breathing if rotated outward

TEST: What is a very common skin test that is an example of a type IV hypersensitivity reaction?

PPD test (for TB testing!)

TEST: Neutrophils contain both _______ and _______ granules in their cytoplasm. List examples of each of these granule types.

PRIMARY or AZUROPHILIC granules: - alpha-defensins - Lysozyme - MYELOPEROXIDASE SECONDARY granules: - Lactoferrin

RECALL: What drug can increase systemic drug concentrations of Oseltamivir? (this drug can increase systemic drug conc of many antibiotics and other acidic drugs)

PROBENECID

TEST: Chronic bronchitis is diagnosed if there is a ________ cough for at least ___ consecutive months in at least ___ consecutive years

PRODUCTIVE cough for at least THREE consecutive months in at least TWO consecutive years

In general, beta2 agonists are not intended for ________ of asthma

PROPHYLAXIS

TEST: Sarcoidosis usually presents with what hallmark finding in the involved organs? What other disease also has this finding?

NONCASEATING GRANULOMAS! - Other disease that has this is hypersensitivity pneumonitis!!

TEST: What is the effect of gravity on alveolar pressure?

NONE! Gravity does NOT affect the air pressure within the alveoli

Tracheal/Bronchial sounds are ___. These sound are heard over the ___/____ and are ___ in quality. Tracheal/Bronchial sounds are ____ or more high pitched than vesicular sounds and the ___ is ___ the inspiratory phase....or slightly longer.

NORMAL Over the STERNUM/TRACHEA TUBULAR quality LOUDER than vesicular Expiratory phase is EQUAL to the inspiratory

In an ASTHMATIC pt. plain films can be ___ in up to 75% of all pts. Key findings are:

NORMAL in 75% Key findings: *PULMONARY HYPERINFLATION BRONCHIAL WALL THICKENING*

The _______ are the part of the nasal cavity that communicates with the external environment, while the _______ communicates with the nasopharynx posteriorly

NOSTRILS (nares) anteriorly CHOANAE posteriorly (posterior nasal apertures)

NSIP is ____ What categories does it fall into?

NSIP - Nonspecific Interstitial pneumonia This is UIP but less severe Cellular or Fibrotic type Cellular = >10 yr survival Fibrotic survival = 5 yrs

Do smokers or non smokers develop pneumonitis more often?

Non smokers > Smokers.

Just below the posterior 1/3 of the tongue, and associated with the FOURTH arch is what embryonic structure of the pharynx?

Epiglottis swelling --> Epiglottis

TEST: What are some of the typical symptoms of asthma?

Episodic RECURRENCE of: - Coughing (particularly *at NIGHT* or early morning) - WHEEZING - Breathlessness - Chest tightness *again, the sx's wax and wain and they worsen at night*

How can a diagnosis fee made for pneumoperitoneum

Erect chest radiograph - most useful Abdominal radiograph Decubitus film (left side down) Computerized tomography

Even though the lung buds are an outpouching of the _____ region developmentally, it ends up in the _____ region

Even though the lung buds are an outpouching of the NECK region developmentally, it ends up in the CHEST region

Again: TRUE or FALSE: Smokers are more likely to develop hypersensitivity pneumonitis than non-smokers

FALSE!

TRUE or FALSE: A calf clot can cause a PE

FALSE!

TRUE or FALSE: A specific diagnosis can be made with spirometry alone

FALSE!

TRUE or FALSE: Alveolar gas composition is the same as the composition of inspired gas int he conducting airways

FALSE!

TRUE or FALSE: CWP may increase the incidence of cancer

FALSE!

TRUE or FALSE: Exercise induced asthma is a form of allergic asthma

FALSE!

TRUE or FALSE: Ipratropium can cross the blood brain barrier

FALSE!

TRUE or FALSE: Many of the ILDs are responsive to pharmacological therapy

FALSE!

TRUE or FALSE: Since hypersensitivity pneumonitis is a type of hypersensitivity reaction, eosinophilia is common

FALSE!

TRUE or FALSE: There are goblet cells within the olfactory epithelium

FALSE!

TRUE or FALSE: You expect to find infiltrates on CXR for asthma

FALSE!

*****TRUE or FALSE: Macrophages are not a type of antigen-presenting cell

FALSE! *They can present Ag to T-cells and initiate ADAPTIVE immunity*

TEST/TRUE or FALSE: As you move up in altitude, the partial pressures of N2, O2, and CO2 will rise

FALSE! - As altitude increases, barometric or atmospheric pressure DECREASES - Since the %gas for each gas remains the same, this will produce a SMALLER partial pressure for each gas (since Pgas = %gas x Ptotal)

TRUE or FALSE: Bordatella pertussis species attaches and invades the ciliated epithelial cells in the upper respiratory tract

FALSE! Bordatella pertussis species DO NOT INVADE! (only attach) Attach with *Filamentous hemagglutinin*

TRUE or FALSE/TEST: A normal CXR excluses the diagnosis of hypersensitivity pneumonitis

FALSE! Hypersensitivity pneumonitis can present with a normal CXR!! *LOW sensitivity*

TRUE or FALSE: Non-caseating granulomas is specific for sarcoidosis

FALSE! It is most commonly seen in sarcoidosis, but other diseases with this include: - hypersensitivity pneumonitis - TB

TRUE or FALSE: During expiration, the pleural pressure is positive so it can drive air out of the lungs

FALSE! Pleural pressure is NEVER positive during the respiration cycle, or else the lungs will collapse!

TRUE or FALSE: Nonallergic asthma is still considered a type I hypersensitivity reaction

FALSE! *Non-allergic* The IgE levels are normal in this case

TRUE or FALSE: Exercise can cause oxygen gas exchange to be diffusion limited for normal lungs

FALSE! Alveolar PO2 is normal during exercise!

TRUE or FALSE: Oseltamivir is effective against H1N1

FALSE! H1N1 = 100% resistant - But novel H1N1 (swine flu) is sensitive still

TRUE or FALSE: Cyanosis is an early sign of asthma

FALSE! It is a late stage sign

TRUE or FALSE: The diaphragm develops after the development of the alveoli

FALSE! The diaphragm develops relatively early on, at about the 8th week, while the alveoli start to develop at around the 32nd week

TRUE or FALSE: The lung buds develop from the pharyngeal arches

FALSE! The lung buds develop AROUND the pharyngeal arches as an EXTENSION of the embryological pharynx, but they are NOT formed directly from them

MethylXanthenes, because of ____ are not currently used clinically as often.

Toxicities

Identify the

Trachea Carina

the structures within the larynx are a pair of vocal chords is known as the ___ ____. These function to ___ ___ and____ ____

Vocal fold Control air and produce sound

Define percent oxygen saturation

Volume of O2 bound to Hb / Hb carrying capacity of O2

TEST: Define tidal volume (TV)

Volume of air inspired and expired during NORMAL quiet breathing

TEST: Define residual volume (RV)

Volume of air remaining in the lungs at the END of MAXIMUM EXPIRATION

Charcot-Leyden Crystals histo

Triangular pale pink in upper right.

TEST: What is the difference in epithelium that covers the true versus the false vocal cords?

True vocal cords: STRATIFIED SQUAMOUS False vocal cords: RESPIRATORY epithelium

Voriconazole is a derivate of _________, and like it it has good ____ penetration. However, unlike it it is extensively _______ in the liver

Voriconazole is a derivative of FLUCONAZOLE, and like it it has good CNS penetration - Unlike fluconazole in that it is extensively METABOLIZED in the liver (remember, fluconazole is excreted unchanged)

What is the equation for DLCO?

V'CO / PACO V'CO = Ventilation rate of CO

The V/Q ratio is a major determinant of _______ PO2 and PCO2 values, which then determine the systemic _______ PO2 and PCO2 values

V/Q = major determinant of ALVEOLAR PO2 and PCO2 values, which then determine the systemic ARTERIAL PO2 and PCO2 values V = Ventilation of ALVEOLI Q = Perfusion of PULMONARY BLOOD FLOW

AGAIN: V/Q values (>/<) 1.0 are the most common cause of arterial hypoxemia

V/Q levels < 1

TEST: If the A-a gradient turns out increased, what are the three possible causes of hypoxemia?

V/Q mismatch (responds to 100% oxygen) Shunt Impaired diffusion (decreased DLCO)

AGAIN: From the equation for alveolar ventilation rate, it depends on what two primary factors?

VA (which is tidal volume - dead space volume) f (respiratory frequency) You get a greater VA' if you have a greater tidal volume or a greater respiratory frequency

TEST: What is the equation to calculate alveolar ventilation rate?

VA' = VA x f VA = (tidal volume - dead space volume) f = Respiratory frequency, or the number of breaths per unit time (usually per minute) Another way of saying this is: VA' = Vt' - Vd'

AGAIN: What is the equation for alveolar ventilation rate?

VA' = Vt' - Vd' or VA x f Vt' = Minute ventilation rate (tidal volume rate) Vd' = Dead space ventilation rate VA = Tidal volume - dead space volume f = respiratory frequency

TEST: The main nerve that innervates the larynx is the ______ nerve, and it gives off a major branch called the _______ laryngeal n. What are the two main branches of this, and which one is the sensory branch?

VAGUS N., gives off the SUPERIOR LARYNGEAL NERVE, which has two main branches: 1) Internal laryngeal n. - SENSORY 2) External laryngeal n.

TEST: What antibiotics are used when the respiratory infection is from MRSA?

VANCOMYCIN or LINEZOLID

What structure sits right behind the pharynx, and what layer separates the two?

VERTEBRA Pharynx and bone are separated by the BUCCOPHARYNGEAL FASCIA!!

Complaince of the lung is (volume/pressure) dependent and it is the inverse of ______ _____ of the lung

VOLUME dependent - Greater compliance at low lung volume Inverse of the ELASTIC RECOIL of the lung

TEST: Define vital capacity (VC)

VOLUME of air that can be *EXHALED from the lungs after a MAXIMUM INSPIRATION*

TEST: The primary measurement in spirometry is either ______ or _____

VOLUME or FLOW

Type I hypersensitivity reactions, also called a ______ and ______ reaction, is characterized by the involvement of _____ cells and _____ (antibody)

WHEAL and FLARE reaction - Characterized by the involvement of MAST cells and IgE

A HIGH PITCHED continuous sound that is generated by airflow through NARROWED airways are ___ These are ___ in nature and indicate an ___ ____

WHEEZES Musical in nature Indicate AIRFLOW OBSTRUCTION

(male/women) tend to develop interstitial lung disease associated with connective tissue disorders, except _______ _______

WOMEN, except in RHEUMATOID ARTHRITIS (where males are more affected)

Define alveolar pressure (Palv)

Water pressure (cm H2O) held within the alveoli

Question 2

What is it?

Flip card for case:

What should we do for this person? ADMIT TO ICU

Sound heard mainly in EXPIRATION?

Wheeze

A continuous high pitched whistling sound in the airway during breathing is ?

Wheezing Commonly *caused by asthmatic attacks*

When are flow-volume loops obtained?

When a patient does an expiratory forced vital capacity (FVC) followed by an inspiratory forced vital capacity (inhales quickly and deeply to TLC)

When is oxygen uptake into blood diffusion limited?

When alveolar PO2 is LOW! - Such as at HIGH-altitude or hypoventilation

When the pleural cavity is ruptured/damaged, the intrapleural pressure (increases/decreases) and may exceed the ________ pressure, causing _______ on the lungs known as a ___________

When pleural cavity is ruptured/damaged, intrapleural pressure INCREASES and may exceed the ATMOSPHERIC pressure, causing COMPRESSION on the lungs known as a PNEUMOTHORAX

Under what conditions would you get "alveolar" dead space? Would you get this under normal conditions?

When the alveoli are NOT/POORLY PERFUSED, so there is NO GAS EXCHANGE occuring - Therefore, in a normal person, this dead space is either ZERO or very small as there should be very few non-perfused alveoli

TEST: Describe the oxygen DEPENDENT mechanism of killing microbes by macrophages

When the microbe gets phagocytized, the plasma membrane which forms the phagosome gets disrupted. On this plasma membrane is an NADPH OXIDASE that when stimulated will be activated to generate SUPEROXIDE from OXYGEN (reduction). This superoxide then undergoes dismutation to form H2O2, which INHIBITS THE GROWTH OF INTRACELLULAR PATHOGENS

TEST/TEST: What defect causes a congenital herniation of the diaphragm? What lung defect can this cause?

When the pleuroperitoneal membrane (folds) fail to close adequately, it causes herniation of the stomach into the (Left) pleural cavity!! Eventually, so many guts and stuff herniate up into the LEFT pleural cavity (most common) that it causes a HYPOPLASTIC LUNG!!

When does zone 1 of an upright lung exist?

When there are ABNORMAL conditions causing CONTINUOUS VESSEL COLLAPSE!

How does legionella pneumophila evade normal macrophage antimicrobial mechanisms?

When they get engulfed by macrophages in a phagosome, they recruit RIBOSOMES onto the phagosomal membrane and makes it look like ROUGH ER. The host cell does not recognize this as a phagosome, and thus it cannot fuse with the lysosome to degrade its contents (the pathogen)

What on earth is an EPP?

When you try to force an expiration, your ribs push on your lungs and increase pleural pressure. This causes an "EPP" or "Equal Pressure Point" just beyond the alveoli that is basically a little banded area of restriction. This means that a person's driving pressure will always be the same, because of that EPP. In fact, their Driving Pressure= alveolar pressure minus the pleural pressure.

When would you think secretions if you hear crackles?

When you wake the pt up in the morning. Have them cough and listen again...they then disappear.

Are azole antifungals considered teratogenic?

YES! They are contraindicated in pregnancy!!

At Functional Residual Capacity, the alveolar pressure is ___

ZERO! (No air movement) - And since this is at the end of expiration, the inward pull of the lung is balanced by the outward pull of the chest wall

Like ________, Zileuton can significantly increase warfarin PT times

Zafirlukast Both are substrates for CYP2C9, inhibiting it from metabolizing warfarin

TEST: What is a leukotriene synthesis inhibitor, and what is its mechanism?

Zileuton MOA: *Inhibits 5-lipooxygenase, inhibiting the production of leukotrienes*

What are some of the main side effects of anticholinergics?

Paradoxical bronchconstriction GLAUCOMA Bitter tase DRY mouth Urinary retention CONSTIPATION

If the patient has three or more rents fractured in two or more segments this can cause a ____ Motion of the chest wall, this is known as a ___ ___

Paradoxical motion *Flail chest*

What is the orientation of the olfactory cilia?

Parallel to the surface of the olfactory epithelium

Wheezes have which two subcategories?

Polyphonic Monophonic

The nasal cavity/sinuses communicate with the nasopharynx inferiorly through openings called what?

Posterior NARES

RECALL: What do the right and left recurrent laryngeal nerves each loop under as it ascends?

Right: SUBCLAVIAN vein Left: AORTIC arch

RECALL: How many secondary bronchi are on the right versus the left side? These correspond to the number of ______ on each lung.

Right: THREE secondary bronchi (forming THREE LOBES of the R lung) Left: TWO secondary bronchi (forming TWO LOBES of the L lung)

TEST: Selective beta2 agonists can either be _____-acting or _____-acting

SHORT-acting (SABAs) or LONG-acting (LABAs) SABA = Short-acting beta2 agonist

******TEST: What kind of curve represents the kinetics of oxygen loading on hemoglobin? Why does it show this shape?

SIGMOID shape, as hemoglobin demonstrates COOPERATIVITY *O2 on then O2 off* (ALLOSTERIC properties)

MONOPHONIC wheezes are ___ pitch and usually heard in ___ ___ of the chest The term ___ is used to describe the inspiratory wheezing like sound that results from narrowing of the ___ or large ___ or the ___

SINGLE pitch ONE area of the chest STRIDOR from narrowing of the TRACHEA or LARGE BRONCHI...or the LARYNX

The forced vital capacity is usually measured at ____ seconds

SIX seconds

Anticholinergics are (faster/slower) acting than but acts for a (shorter/longer) duration than SABAs

SLOWER acting but LONGER duration

RB-ILD is associated with __ and appears in the ___-___ decade. What lobes affected? tx?

SMOKERS 4/5th decade Upper Lobes Tx: Prednisone and smoking cessation

The number one treatment for COPD is what?? (think of what the number one cause is)

SMOKING CESSATION!!!

The posterior surface of the trachea is lined by _______ ______. Why?

SMOOTH MUSCLE, because the esophagus sits right behind the trachea and when you swallow food, it pushes against the trachea. The smooth muscle is there to prevent the trachea from collapsing and cutting off the airway.

The oropharynx spans from the ____ ______ to the _____ bone

SOFT PALATE/UVULA to the HYOID bone

List some of the common connective tissue diseases that are associated with interstitial lung disease

Scleroderma SLE Rheumatoid arthritis Primary Sjogrens syndrome Mixed CT disease Polymyositis-Dermatomyositis

What is the genetic abnormality associated with primary ciliary dyskinesia?

Short dynein arms, decreasing the effectiveness of the cilia to clear mucus

RECALL: Between macrophages and neutrophils, which one is short-lived? Which one presents antigens? Which one has myeloperoxidase?

Short lived: NEUTROPHILS Presents Ag: MACROPHAGES Contains myeloperoxidase: NEUTROPHILS

TEST: The first line treatment of all stages of COPD is what? How does it work?

Short-acting BRONCHODILATORS - Relaxes the airway smooth muscle, reducing the hyperinflation and reducing the dyspnea symptoms

A type of PECTORILOQUY, when the pt says '99' the sounds should ___ but in a consolidated lung sound the sound will ___

Should DIMINISH Diseased lung = Louder sounds This is called BRONCHOPHONY

Beta lactam antibiotics should either be ____ dose or given with ___-________ inhibitors

Should be either HIGH dose or given with BETA-LACTAMASE inhibitors (which are penicillin-binding proteins in the periplasmic space that destroy -lactam antibiotics)

What does it mean if something is pre-viable?

Something that CANNOT survive OUTSIDE the womb

What is a VESICULAR breath sound? Where are they? How do they sound?

Sound heard during auscultation in the PERIPHERAL LUNG FIELD of a HEALTHY person SOFT/Muffled More intense and longer in INSPIRATION

TEST: There are specialized ________ _______ cells for ______ in the roof of each nasal cavity/superior concha

Specialized SENSORY BIPOLAR cells (OLFACTORY cells) for OLFACTION in the roof of each nasal cavity/superior concha

Which longitudinal pharyngeal muscle is innervated by a different nerve than the one that innervates the others?

Stylopharyngeus, innervated by CN IX The other two longitudinal pharyngeal muscles are innervated by the VAGUS nerve (CN X)

What is the major difference between the superior bronchus of the right lung versus the left lung?

Superior bronchus on the LEFT lung has a LINGULAR branch with 2 additional divisions, for a total of 4 divisions. Apicoposterior, anterior Superior/inferior lingual

RECAP: List the most common CXR findings of stages 1-3 sarcoidosis

Stage 1: Bilateral hilar LAD without lung involvement (asymptomatic) Stage 2: Bilateral hilar LAD with lung involvement Stage 3: Lung involvement WITHOUT bilateral hilar LAD

What is an acute unremitting attack of asthma called, *not responsive to bronchodilators* and which can be fatal?

Status asthmaticus

In status asthmaticus, patients cannot breath (in/out). As a result, the lungs become (over/under)-inflated and press against the surrounding chest ____. The lungs on gross autopsy are (red/pink), which is different from most lungs. Moreover, we can see mucous ________ which is a characteristic gross feature of status asthmaticus.

Status asthmaticus: - Patients cannot breathe OUT - Lungs become OVERinflated, pressing against surrounding chest WALL - PINK lungs - Characteristic mucous PLUGGING

List some drugs that decrease in concentration when administered with Rifampin

Steroids, Protease inhibitors, Warfarin, Antiepileptics, Methadone, Morphine, Digoxin

Lines that are visible on xray

Still follow ABCD... ETT = endotracheal tube SG = Pulmonary artery catheter P = Chest tube M = mitral valve

What is the embryological pharynx structure that forms the mouth?

Stomodeum

What kind of epithelium lines the mucosa layer of the pharynx?

Stratified squamous epithelium

Streptomycin, like _______, can cause ______ loss that can become permanent

Streptomycin, like VANCOMYCIN, can cause HEARING loss that can become permanent

TEST: A diaphragmatic hernia is most likely to occur at the ___ hiatus. Why?

T10, or ESOPHAGEAL hiatus - This is the most likely site of a diaphragmatic hernia because the seal at this hiatus is naturally weaker than the other ones, as the esophagus needs to be able to slide up and down when swallowing food

The tracheal bifurcation is found at the ___ ___ vertebral junction

T4-T5

What type of pneumothorax can be identified clinically?

TENSION pneumothorax NEVER get a film for this.

How many constrictor muscles of the pharynx are there? Are they circular or longitudinal, and what are they?

THREE CIRCULAR muscles: - Superior, middle, and inferior constrictor muscles

How many elevator muscles of the pharynx are there? Are they circular or longitudinal, and what are they?

THREE LONGITUDINAL muscles: 1) Stylopharyngeus 2) Salpingopharyngeus 3) Palatopharyngeus

There are ___ stages of sarcoidosis, with about half of all cases presenting in stage ___ and are usually (symptomatic/asymptomatic). TEST: What is the most common CXR finding?

THREE stages - About half of cases present in stage ONE - Usually ASYMPTOMATIC Most common CXR finding: *Bilateral hilar LAD WITHOUT lung involvement*

How many types of influenza viruses are there? Which two cause significant disease in humans? Which one of these two is limited to humans? Which one can cause severe epidemics?

THREE types, A/B/C, but only A and B cause significant disease in humans - Type B is limited to humans (usually milder) - Type A can cause severe epidemics (and pandemics), and can infect OTHER SPECIES

If you see a dot over a letter, that usually implies that it is that variable per unit _____

TIME

Spirometry measures how an individual inhales or exhales volume of air as a function of ______

TIME!

Beta lactams are a class of antibiotics that exhibit (time/concentration) dependent killing

TIME-dependent killing

TEST: Total lung capacity is the volume of air in the lungs after maximum __________. TLC is the sum of what FOUR terms?

TLC = volume of air in the lungs after maximum INSPIRATION TLC = IRV + TV + ERV + RV

In obstructive lung disease, although the total lung capacity (TLC) may be _______, the ______ volume and functional _______ capacity are increased

TLC may be NORMAL RESIDUAL volume and function RESIDUAL capacity (FRC) are INCREASED

_____-____ inhibitors have been effective at treating sarcoidosis

TNF-ALPHA inhibitors Steroids

TEST: How is Nystatin given? Why? What is it used to treat?

TOPICALLY, as it is NOT ABSORBED SYSTEMICALLY from the skin, vagina, or GI tract Used to treat CANDIDIASIS in the skin, vagina, and GI tract

What muscle lines the posterior area of the trachea where the cartilage fails to connect?

TRACHEALIS smooth muscle!

TEST/TEST: What can be a clinical problem of having the developing trachea descend parallel with the developing esophagus?

TRACHEOESOPHAGEAL FISTULAS! - An abnormal connection between the esophagus and the trachea!!

TRUE or FALSE: Almost all chemotherapy forms can cause some form of interstitial lung disease

TRUE

TRUE or FALSE: The CXR is often normal for a patient with PE

TRUE

TRUE or FALSE: The CXR may be normal in ILD

TRUE

TRUE or FALSE: Lung biopsy can rule out any neoplastic or infectious processes of ILD

TRUE Biopsies are from 2 sites

T/F: Long acting beta agonists are not indicated for tx of asthma.

TRUE LABA are NOT indicated for asthma tx.

TEST/TRUE or FALSE: Atopic and occupational asthma is IgE mediated and involves Th2 cells releasing cytokines that stimulate eosinophils

TRUE!

TRUE or FALSE: SMALLER inhaled particles achieve toxic levels more readily than larger inhaled particles

TRUE!

TRUE or FALSE: 100% of azithromycin concentration in the serum reaches the lungs

TRUE!

TRUE or FALSE: A lack of response to the bronchodilator does not preclude a clinical response

TRUE!

TRUE or FALSE: Alveolar macrophages are a part of the first line innate defense mechanism against respiratory infections

TRUE!

TRUE or FALSE: By the end of inspiration, the pressure outside and the pressure inside the lungs are equal to zero

TRUE!

TRUE or FALSE: CWP may increase the incidence of chronic bronchitis/emphysema

TRUE!

TRUE or FALSE: Carbon monoxide has a much greater affinity for hemoglobin than oxygen

TRUE!

TRUE or FALSE: Doxycycline has good penetration into lung tissue but not pleural fluid

TRUE!

TRUE or FALSE: Emphysema causes bronchiole wall destruction but not fibrosis

TRUE!

TRUE or FALSE: If you prevent a DVT, you will prevent PEs

TRUE!

TRUE or FALSE: Imidizaoles have the same MOA as triazoles

TRUE!

TRUE or FALSE: Inhaled dust particles may initiate a host immune response to SELF proteins

TRUE!

TRUE or FALSE: Obstructive defects typically have a normal chest X-ray

TRUE!

TRUE or FALSE: Respiratory epithelial cells forms a first line innate defense mechanism against respiratory infections

TRUE!

TRUE or FALSE: Sarcoidosis can lead to blindness if left untreated

TRUE!

TRUE or FALSE: The 6th pharyngeal arches aren't actually physical arches even though we have mesoderm associated with them

TRUE!

TRUE or FALSE: The R lung has 3 lobes while the L lung has 2 lobes

TRUE!

TRUE or FALSE: The developing esophagus is also embedded in splanchnic mesoderm

TRUE!

TRUE or FALSE: The majority of interstitial lung disease is idiopathic

TRUE!

TRUE or FALSE: The smooth muscle of the bronchi is discontinuous

TRUE!

TRUE or FALSE: There are no goblet cells in the respiratory bronchiole epithelium

TRUE!

TRUE or FALSE: Tiotropium has been shown to increase the risk of CV events in chronic COPD users

TRUE!

TRUE or FALSE: You expect to see hypertrophied smooth muscle associated with the airways in an asthmatic patient

TRUE!

TRUE or FALSE: More women die today from COPD than men

TRUE! - Much of this has to do with women in developing countries who cook in poorly ventilated homes

TEST/TRUE or FALSE: The rate of decline of pulmonary function is similar in those who have never smoked and those who stopped smoking

TRUE! - The RATE OF DECLINE of (FEV1) lung function will be the SAME in all patients, In SMOKERS = LESS RESERVE CAPACITY but their rate of decline is the same as someone who has never smoked (this is why you want people who smoke to stop, because they're adding on decreased reserve capacity in their lungs in addition to the regular decline in function)

TRUE or FALSE: Pulmonary function tests allow you to quantify how severe a disease is

TRUE! Also - Follow disease over time/response to tx.

TRUE or FALSE: Neutrophils are not a type of antigen-presenting cell

TRUE! Therefore, they CANNOT initiate adaptive immunity

TRUE or FALSE: Asthma can cause bronchitis

TRUE! (known as asthmatic bronchitis)

TRUE or FALSE: Wheezing is an uncommon symptom of hypersensitivity pneumonitis

TRUE! Common symptoms include fever, cough, chills, and SOB

TRUE or FALSE: Emphysema increases lung compliance

TRUE! Emphysema destroys the alveolar septal tissue that normally OPPOSES lung expansion, so it indeed does increase COMPLIANCE (distensibility)

TRUE or FALSE: You never give inhaled steroids in acute exacerbations of COPD

TRUE! For acute exacerbations, give oral or IV steroids, but never INHALED!

****TRUE or FALSE: Mutations in the IFN-gamma gene is often linked to increased susceptibility to M. tb infections

TRUE! Remember, *IFN-gamma is CRITICAL for controlling M. tb infections* (and IFN-gamma is produced by Th1 CD4 cells)

TRUE or FALSE: Occupational exposure to asbestos is linked to cancer

TRUE! (think asbestos commercials and mesotheliomas)

TEST/TRUE or FALSE: Pleural pressure is always smaller than alveolar pressure

TRUE! If not, the lung will collapse!

TRUE or FALSE: You cannot make a diagnosis of ILD on spirometry alone

TRUE! You must get a BIOPSY! However, biopsy may not be as useful nowadays....yet still required for an accurate diagnosis

TRUE or FALSE: A short course of steroids is just as beneficial as a long course of steroids for COPD exacerbations

TRUE!! - There is no difference in the % exacerbations, deaths, infections, etc... 5 days over 14 day course

TEST/TRUE or FALSE: Silicosis increases the susceptibility to tuberculosis

TRUE!!!!!!!!!!!!!!!!!!!!!!!!!!!!!!!!!!!!!!!!!

TEST: Patient shows: - FVC = 69% - FEV1 = 51% - FEV1/FVC = 63.72 TRUE or FALSE: The patient has an obstructive lung disease

TRUE, BUT!!!!! - You cannot rule out a RESTRICTIVE lung disease (since FVC is less than 80%) until you get LUNG VOLUMES!

TRUE or FALSE: CD4 T cells make up the majority of the T cell population

TRUE, about 2/3 of the T cell population are CD4 T cells

TRUE or FALSE: In an acute asthma attack, you expect pCO2 levels in the blood to be low

TRUE, although this is an obstructive disease and air is trapped, *hyperventilation occurs in an acute asthma attack so it will lead to LOW CO2 levels*

TRUE or FALSE: Longer distances increase flow resistance

TRUE, as distance and resistance are directly proportional

TRUE or FALSE: H1 histamine antagonists have anti-inflammatory activity

TRUE, as it suppresses NF-kB activation and thus transcription of pro-inflammatory cytokines

TRUE or FALSE: Exercise increases the diffusion capacity

TRUE, because exercise increases the number of perfused pulmonary capillaries and it increases the membrane surface area!

TRUE or FALSE: Salmeterol is not intended for acute asthmatic attacks

TRUE, because it has a SLOW onset (~20 min)

TRUE or FALSE: Malnutrition may worsen CF lung disease. Why or why not?

TRUE, because it has been shown that malnutrition tends to develop more respiratory infections! - This is why many patients in CF may be on parenteral nutrition (especially if they are malnourished), as it can transiently improve respiratory function!

TEST: The olfactory cell projects olfactory _______ to the apical surface, which contain numerous ______ that sense odorants. The base of the olfactory cell gives rise to a long ______ that ultimately joins others to form the olfactory ______, which are largely (myelinated/unmyelinated).

The olfactory cells projects olfactory DENDRITES to the apical surface, which contain numerous CILIA that sense odorants The base of the olfactory cell gives rise to a long AXON that ultimately joins others to form the olfactory NERVES, which are largely UNMYELINATED

What is the issue with this patient?

look at the arrow Lung cancer

Major resistance is at the _________-sized bronchi. Most of the pressure drop occurs at the _________ division.

medium seventh

Driving Pressure= _______ pressure minus the ______ pressure.

alveolar pleural

The phospholipid "surfactant" is produced by __________ cells. What is the main function of surfactant?

alveolar type II Surfacant lowers surface tension by reducing attractive forces of hydrogen bonding .

When you find a SOLITARY pulmonary nodule, what needs to be done?

can be malignant or benign NEED to compare to OLD FILMS

Aifrflow obstruction in asthma is often ____

reversible.

Surface tension measurements show that the surface tension of LUNG extract is area _____________, while water and detergent are area ______________. Of those three substances, _______ has the highest surface tension.

dependent independent Water

Inspiration is active and uses ______________ and _________ muscles.

diaphragm external intercostals

In a Bronchial alveolar lavage, you see this image...this iron substance is a?

ferreginous body

The inflation (inspiration) and deflation (expiration) curves are not the same, which is called ______________.

hysteresis

IF the posterior cricoarytenoid m is rotated inward, this allows for

speaking.

Restful breathing works on the _____________, most compliant part of the curve. With a deep inspiration, the lung moves toward the ___________ part of the curve, and thus it has reduced compliance.

steepest flatter

What are the two factors affecting lung compliance?

the elastic tissue of the lung and the chest wall; surface tension of the fluid that lines the inside wall of the alveoli.

A MULTIDETECTOR CT allows ___ ___ in a shorter period

thinner slices

Compliance of the lung is _____________-dependent (nonlinear). What does that mean?

volume It is greater at low lung volume and lower at high lung volume.

Hb saturates at PO2 = ____

~100mmHg

***AGAIN: What is the PO2 in the final destination of inspired air?

~2mmHg, in the MITOCHONDRIA

What is the PCO2 in the alveolus? What about in systemic arterial blood? Are they the same?

~40 mmHg in either the alveolus or the arterial blood - They ARE the same because the *CO2 had to diffuse into the alveoli FROM the blood!*

RECALL: What is the partial pressure of water vapor at a body temperature of 37 degrees C?

~47 mm Hg

RECALL: What is Patm at sea level?

~760 mmHg

At what vertebral level does the esophagus start?

~C6

What ratio is really useful in characterizing lung disease?

FEV1/FVC ratio

The -mab drugs are used to focus on ___ ____

MONOCLONAL ANTIBODIES

****Only ___% of the PO2 in inspired air can actually dissolve in blood through direct diffusion With hemoglobin, now ___% of the total blood volume is oxygen

*0.3% (0.003 mL O2 per every 100 mL blood)* With hemoglobin, now *20% of the total blood volume is oxygen!* - This is *~70x the amount of oxygen dissolved in blood through direct diffusion*

List at least one advantage and one disadvantage to using body plethysmography to measure lung volumes?

*Advantage*: Rapid, more accurate than gas techniques in obstructive airway disease *Disadvantage*: Expensive, more complex than gas techniques, unable to use in claustrophic or severely obese patients

What lab finding is often elevated in patients with sarcoidosis (50-80% of patients)? Is this a specific finding?

*Elevated angiotensin converting enzyme* (ACE) - NOT specific however, because there are many other causes of an elevated ACE (hyperthyroidism, chronic renal failure, diabetes, etc...)

What are the cells in the bronchus that create the mucus? The smooth muscle around the bronchus will show?

*Goblet cell* metaplasia Smooth muscle hyperplasia

Chronic bronchitis is caused by repeated and frequent exposure to what?

*Inhaled irritants, like CIGARETTE SMOKE and urban smog.* These toxins *irritate the airway MUCOSA, causing HYPERSECRETION and hyperplasia of mucous-secreting glands* (and Goblet cells) which --> narrowing of airway --> OBSTRUCTION!

What is the benefit of administering alpha adrenergic agonist nasal decongestants orally versus topically? List at least three alpha adrenergic agonists, and whether they stimulate alpha 1 and alpha 2 receptors

*Orally*: Prolonged effects, less potent, no REBOUND congestion 1) Phenylephrine - alpha1 2) Ephedrine - alpha1 3) Oxymetazoline - alpha2

TEST: What are the typical patterns in lung VOLUMES of restrictive vs obstructive defects?

*Restrictive: TLC < 80% predicted* Obstructive: TLC > 120% predicted TLC = Total lung capacity

What is D-dimer?

*Specific degradation product* released into circulation when cross-linked fibrin undergoes fibrinolysis

Smoking is associated with the interstitial lung disease of ___ ____ ____ and ___ as well as ____ It is NOT associated with _____ or ____

*Yes*: Langerhan Cell Histiocytosis DIP RB-ILD *NO*: Hypersensititivity pneumonitis Sarcoidosis

Where is this pneumonia located?

- R Lower lobe location. Silhouette sign indicates the R hemidiaphragm

List some of the causes of bronchiectasis

- Allergic bronchopulmonary aspergillosis - Cystic fibrosis - Kartagener syndrome (combo of primary ciliary dyskinesia, bronchiectasis, sinusitis, and situs inversus) - Necrotizing pneumonia (staphlyococcus, TB) - Bronchial obstruction (tumor, foreign body, mucus, etc...)

As inspiration begins, your alveolar pressure changes from ___ to ____ due to the depression/contraction of the diaphragm. Also, the Pi (intrapleural pressure) also decreases from ___ to ____.

0 to -1 -5 to -7

During expiration, there is a decrease in alveolar size and therefore the alveolar pressure goes from ____ to ____. What happens to the intrapleural pressure? And of course, barometric pressure is ___. Cuz there ain't no storm comin'.

0 to 1 it stays about the same (somewhere around -5 to -8) because although the chest wall is moving inward, so are the alveoli. 0

RECALL: What is the normal respiratory exchange ratio at room temperature? What is the normal PAO2 breathing room air?

0.8 Normal PAO2 = 150 - (40/0.8) = 100 mmHg PIO2 = 150 PaCO2 = 40

*****TEST: For small solutes or gases in water, list how long it would take for them to diffuse if the diffusion distance between the capillaries and the target tissue is: - 1 nm - 1 micrometer - 1 mm - 1 m

1 nm takes about 1 nsec 1 micrometer takes about 1 msec 1 mm takes about 1000 sec 1 m takes about 78 years! Note: "Small" solutes or gases = O2, CO2, glucose, etc...

What are the 4 commercial formulations for amphotericin B?

1) C-AMB (conventional) 2) L-AMB (liposomal) 3) ABLC - Lipid complex 4) ABCD - Colloidal dispersion

What are the two main TYPES of nasal decongestants used?

1) *Alpha adrenergic agonists* - They CONSTRICT dilated blood vessels DECREASING the volume of nasal mucosa and thus DECREASING the resistance to airflow 2) *Corticosteroids* - reduces inflammation

TEST: There are 4 factors of the inhaled particulate that influence the development of pneumoconiosis. What are they?

1) AMOUNT of dust RETAINED in the lung 2) SIZE and SHAPE of particles (1-5 microns are really dangerous) 3) Particle SOLUBILITY and reactivity 4) Effects of OTHER present IRRITANTS (ie smoking)

List some of the other mechanisms of beta2 agonists in treating asthma symptoms

1) Activates Ca-sensitive K channels --> K EFFLUX --> Hyperpolarization and RELAXATION 2) Inhibits degranulation of mast cells --> decrease histamine release 3) Stimulating beta2 receptors --> DECREASED ACh release --> DILATION

TEST: IFN-gamma is essential for controlling M.tb infections. List THREE main mechanisms of how this cytokine can help control tuberculosis infections

1) Activates macrophages to produce more REACTIVE OXYGEN INTERMEDIATES (superoxide, H2O2, Hydroxyl radicals, singlet oxygens) 2) Promotes fusion of phagosome containing microbes with lysosomes 3) Activates expression of iNOS (inducible nitric oxide synthase) in macrophages, which synthesizes REACTIVE NITROGEN INTERMEDIATES

Besides pertussis toxin, list two other toxins that are produced by Bordatella pertussis and what the function of each toxin is

1) Adenyl cyclase "toxin" - Inhibits antimicrobial mechanisms of macrophages (when they take it up) 2) Tracheal Cytotoxin - Induces NO production from the ciliated epithelial cells, which in turn KILLS those cells!!

What are two most common developmental defects of the lung?

1) Agenesis 2) Hypoplasia

What are the 2 main portions of the respiratory system?

1) Air conducting portion: External to lung & Within the lung 2) Respiratory portion

TEST: The DLCO depends on what 3 primary factors?

1) Alveolar-capillary membrane 2) Hemoglobin concentration 3) Cardiac output

TEST: What are the two viral coating inhibitors?

1) Amantadine 2) Rimantadine (analog of #1)

TEST: What are the two polyene macrolides?

1) Amphotericin B 2) Nystatin

List five classes of drugs that can be used to treat the SYMPTOMS of a respiratory illness

1) Antihistamines 2) Nasal decongestants 3) Expectorant 4) Cough suppressant 5) Mucolytics

What are the two primary types of asthma? Each type is primarily associated with that?

1) Atopic or EXTRINSIC - *Type I hypersensitivity reaction (allergy) with strong familial tendencies* 2) Nonatopic or INTRINSIC - Associated with *VIRAL infection in patients with NO FAMILY HISTORY of allergies (w/ normal IgE levels)*

TEST: List the three inhaled corticosteroids presented in class

1) Beclomethasone 2) Budesonide 3) Fluticasone

TEST: List three things present in the *epithelial lining fluid* of the inner lung that helps fight infections

1) Beta-defensins (secreted by respiratory epithelial cells) 2) Glutathione - antioxidant 3) IgA - secretory Ab

RECAP: List the MOA of the following drugs that inhibit protein synthesis: 1) Macrolides 2) Doxycycline 3) Linezolid 4) Streptomycin

1) Binds to tRNA site on 50S subunit 2) Binds to ACCEPTOR site on 30S subunit 3) Binds to the P site of the 50S subunit 4) Binds to the 30S subunit

TEST/TEST: List what part of the diaphragm is formed by the following: 1) Body wall mesoderm 2) Septum transversum 3) Pleuroperitoneal membrane 4) Mesoesophagus

1) Body wall mesoderm --> MUSCULAR part 2) Septum transversum --> CENTRAL TENDON 3) Pleuroperitoneal membrane --> CENTRAL TENDON 4) Mesoesophagus --> CRURA (smooth muscle)

RECAP: List the main factors that allow each of the following bacteria to colonize the host respiratory epithelial cells: 1) Bordatella pertussis 2) Mycoplasma Pneumoniae 3) Haemophilus Influenzae

1) Bordatella - Filamentous hemagglutinin 2) Mycoplasma - Cytoadhesins on tapering end with sialic acid from host 3) Haemophilus - IgA proteases that degrade IgA, preventing it from blocking colonization

TEST: List three examples of microbes that have developed pathogenic mechanisms to compensate against the host respiratory epithelial innate defense. List the major pathology that each microbe causes.

1) Bordatella pertussis - Whooping Cough 2) Mycoplasma Pneumoniae - Atypical pneumonia 3) Haemophilus influenzae - Meningitis and sepsis in CHILDREN

TEST: List the three physiologic effects of histamine in the ALLERGIC response TEST: How does this translate symptomatically? What are the characteristic symptoms?

1) Bronchoconstriction 2) Vasodilation 3) Increase capillary PERMABILITY Translates into the TRIPLE RESPONSE of Lewis (FLARE & WHEAL reaction) 1) Red spot 2) Bright flair 3) Wheal

TEST: What are the 4 main layers of the pharynx from most SUPERFICIAL (vertebra) to deep (inner lining)? Which layer do you see as a clinician?

1) Buccopharyngeal fascia (outside) 2) Pharyngeal muscles 3) Pharyngobasilar fascia 4) Pharyngeal mucosa - This is the layer that you will see as a clinician, as this is the closest layer to FOOD passage

RECAP: List whether each of the following occurs primarily in the upper or lower lobes of the lung: 1) Coal Workers Pneumonia 2) Silicosis 3) Asbestos-related diseases

1) CWP- UPPER lobe and upper parts of the lower lobe 2) Silicosis - UPPER lobe 3) Asbestos-related diseases - LOWER lobe

TEST: Which two species of fungi are especially resistant to azole antifungals?

1) Candida Krusei 2) Mucormycosis

TEST: What are the two main types of emphysema, what is each type mainly associated with, and does each type primarily affect the upper or lower lobes?

1) Centriacinar (85% of cases) - Associated with SMOKING, primarily affecting the UPPER lobes 2) *Panacinar* - Associated with *alpha1-antitrypsin deficiency*, primarily affecting the LOWER lobes

TEST: List two common microscopic findings of asthma

1) Charcot-Leyden crystals - formed from major basic protein 2) Curschmann spirals - sloughed epithelial cells in a mucous cast in the shape of the airway

List at least 3 major uses of spirometry

1) Checking the severity of pulmonary impairment 2) Checks and monitors response to therapy 3) Monitoring the effects of environmental/occupational/drug exposures 4) Assess PreOp risk 5) Assess Disability and Impairment

List the three most common mechanisms for tetracycline resistance

1) Chemical modification 2) Efflux (genes) of the antibiotic out of the cell 3) Decreased binding of antibiotic to the target receptor due to RIBOSOMAL PROTECTION

TEST: List at least 6 cell types found within the PSCC of the respiratory zone of the nasal cavity

1) Ciliated columnar cells - Main function: cilia clears air from particulates 2) Goblet cells (glandular simple columnar) 3) Basal cells (small cuboidal cells) 4) Granulated cells 5) Endocrine cells 6) Brush cells (non-ciliated)

TEST: What are the 3 main types of pneumoconiosis?

1) Coal-worker's 2) Silicosis 3) Asbestos-related

TEST: What are two mast cell stabilizers, and what is their MOA?

1) Cromolyn sodium 2) Nedocromil (OPHTHALMIC only) MOA: Inhibits Cl transport needed for Ca-induced histamine release *(PREVENTS Mast Cell DEGRANULATION)*

There are three forms of silicon dioxide, what are they and which one is more fibrogenic? Which one is the most common form?

1) Crystalline - More fibrogenic 2) Amorphous 3) Quartz - Most common form

The rate of gas transfer from the air to the blood is primarily dependent on what two major processes?

1) DIFFUSION 2) PERFUSION (of blood)

What are five classes of long-term asthma control medications?

1) Daily INHALED corticosteroids 2) Leukotriene modifiers 3) Long-acting inhaled beta2 agonists 4) Cromolyn and Nedocromil 5) Methylxanthines

What are two possible physical signs of asthma to look for, especially in children?

1) Dark rings under eyes (allergic SHINERS) 2) Dark transverse crease on nose (allergic salute)

TEST: List four factors that can DECREASE the DLCO

1) Decreased pulmonary capillary blood volume 2) Decreased hemoglobin concentration in the blood 3) Decreased alveolar-capillary membrane surface area 4) Increased membrane thickness

List the three antihistamines discussed

1) Diphenhydramine 2) Cetirazine 3) Loratadine

List three methods for CO2 transport in blood

1) Dissolved CO2 (minimal, ~5% of total) 2) Carbamino compounds (~10% of total) 3) BICARBONATE ions (~85% of total)

RECAP: What are the two ways for oxygen to be transported in blood?

1) Dissolved as a gas in solution 2) Bound to hemoglobin

List at least four mechanisms of corticosteroids

1) Downregulates proinflammatory cytokines 2) Upregulates genes for annexin-1, which inhibits PLA2 (enzyme that increases arachidonic acid and inflammatory prostaglandins) 3) Induces apoptosis in Th2 cells 4) Drug-receptor complex inactives pro-inflammatory TFs

TEST: What are the two causes of the elastic forces of the lung?

1) Elastic TISSUE of the lung/thoracic wall 2) Surface tension of fluid that lines the inside walls of the alveoli

TEST: List the two types of fibers that surround the openings of the alveolar sacs and alveoli, and list what the main function of each is.

1) Elastic fibers - Allows alveoli to EXPAND UPON INSPIRATION and to PASSIVELY contract upon expiration 2) Reticular fibers - Prevents OVER-DISTENTION of the alveoli

To make a Dx of asthma, the medical history, PE, and spirometry must demonstrate what?

1) Episodic symptoms of airflow OBSTRUCTION or airway hyper-responsiveness 2) Airflow obstruction that is partially reversible 3) Excluded alternative Dx

RECAP: The first two things you look at in a PFT are what? Then what do you look at?

1) FVC 2) FEV1 3) FEV1/FVC ratio

Give two well-known examples of hypersensitivity pneumonitis

1) Farmer's lung - Repeated inhalation of heat loving bacteria in moldy hay 2) AC lung - Contaminated air from AC or humidifier with circulating antigens

TEST: What are the four azole antifungals discussed?

1) Fluconazole 2) Itraconazole 3) Voriconazole 4) Posaconazole

TEST: List at least five things you can measure from measuring expiratory flow

1) Forced vital capacity (FVC) 2) Forced expiratory volume (FEV) 3) Forced expiratory flow (FEF) 4) Peak expiratory flow (PEF) 5) Maximum voluntary ventilation (MVV) 6) Slow vital capacity (SVC)

TEST: What are the three primary spirometric indices?

1) Forced vital capacity (FVC) 2) Forced expiratory volume in 1 second (FEV1) 3) Ratio of FEV1/FVC

TEST: What are the four different classes of histamine receptors, and where is each class primarily found?

1) H1 receptors - on vascular ENDOTHELIAL cells, smooth muscle cells 2) H2 receptors - on PARIETAL cells in gastric mucosa, and on cardiac muscle cells 3) H3 receptors - in CNS 4) H4 receptors - on MAST CELLS, T cells, basophils, and eosinophils

TEST: What are the two broad classes of azole antifungals? Which one is more systemic? Which one has more of an impact on human sterol synthesis?

1) Imidazoles - These have MORE of an impact on human sterol synthesis 2) Triazoles - These are SYSTEMIC, and so they are metabolized more slowly and have LESS effect on human sterol synthesis (beneficial)

TEST: What are the three types of side effects associated with the use of amphotericin B?

1) Immediate SYSTEMIC effects - CYTOKINE STORM 2) Renal effects - Renal ischemia 3) Hematologic toxicitiy - Anemia secondary to decreased EPO production

List the THREE things that has to occur at birth to establish respiration

1) Increased surfactant production 2) Lungs go from being primarily secretory to being a gas exchange organ 3) Mature pulmonary circulation is established

TEST: What are two mechanisms of macrolide resistance?

1) Inducible expression of Erm methytransferase 2) Peptide-mediated resistance

TEST: In order to make the diagnosis of bronchiectasis, the patient must have what two components?

1) Infection 2) Obstruction

Airway resistance is primarily due to what two things?

1) Internal friction between gas molecules 2) Friction between the gas molecules and the WALLS of the airways

TEST: What are the 3 major destinations of dust particles once inhaled?

1) Interstitium (by crossing epithelium and getting absorbed) 2) Lymphatic system (drained or migrated) 3) Blood

TEST: What are the two inhaled anticholinergics?

1) Ipratropium 2) Tiotropium (Spiriva) Note: Both are derivatives from atropine

TEST: List the five most common drugs for TB, and whether each drug is a first-line or second-line agent

1) Isoniazid (INH): FIRST-line 2) Rifampin (RIF): FIRST-line 3) Pyrazinamide (PZA): FIRST-line 4) Ethambutol (EMB): FIRST-line 5) Streptomycin (SM): SECOND-line

TEST: What are the three types of air flow?

1) Laminar 2) Transitional 3) Turbulent

TEST: List two respiratory infection causing bacteria that have developed to evade the macrophage defense mechanisms

1) Legionella pneumophila 2) Mycobacterium Tuberculosis

RECAP: List some of the functions of surfactant

1) Lowers surface tension of alveoli and lung, which increases lung compliance and DECREASES WORK of breathing 2) Promotes stability of alveoli so they don't COLLAPSE 3) Prevents transudation of fluid into alveoli

TEST: What are the four categories of information that can be obtained with PFTs?

1) Lung VOLUMES (allowing us to get Vmax) 2) Flow RATES (measure max flow) 3) DIFFUSING capacity 4) Max inspiratory and expiratory pressures (measures applied strength of resp muscles)

TEST: What are 4 possible antibiotic regimes for community acquired pneumonia?

1) Macrolide 2) Doxycycline 3) Respiratory fluoroquinolone 4) Beta lactam + macrolide/doxycycline - Decreases mortality and/or hospital stay!! Beta lactam = amoxicillin/ 3rd gen cephalosporin

TEST: What are the two antibiotics you give for patients with community acquired pneumonia (CAP) who are previously healthy AND have no risk factors for drug resistant strep species?

1) Macrolide (Azithro/clarithro/erythro-micin) 2) Doxycycline

List the FEV1 ranges for the following degrees of severity: 1) Mild 2) Moderate 3) Moderately severe 4) Severe 5) Very severe

1) Mild = >70% 2) Moderate = 60-69% 3) Moderately severe = 50-59% 4) Severe = 35-49% 5) Very severe = <35%

What are three main ways to describe ventilation rates?

1) Minute ventilation rate (Ve' or Vt') 2) Dead space ventilation rate (Vd') 3) Alveolar ventilation rate (VA') Note: I'm using the apostraphe as the dot over the variable

TEST: List the three regions of the pharynx, and what kind of epithelium lines each part.

1) Nasopharynx - Pseudostratified ciliated columnar (PSCC) 2) Oropharynx - Stratified squamous epithelium (SSE) 3) Laryngopharynx - non-keratinized SSE

What are the four different types of bronchiectasis?

1) Normal 2) Cylindrical 3) Varicose 4) Cystic

Which TWO laryngeal muscles make up the laryngeal inlet sphincter?

1) Oblique artenoid muscles 2) Thyroarytenoid muscles

What are the two major types of ventilatory defects?

1) Obstructive ventilatory defect 2) Restrictive ventilatory defect

Linezolid is effective against gram (positive/negative) bacteria. List at least 3 advantages of this antibiotic.

1) Oral prep with 100% bioavailability 2) Extensive distribution 3) Full penetrance to lungs 4) No interaction with CYP enzymes

TEST: What are the two main functions of the larynx?

1) Organ of PHONATION 2) Close TRACHEA when swallowing to prevent food from aspirating into the lungs/airway

TEST: What are the two neuraminidase inhibitors?

1) Oseltamivir 2) Zanamivir

TEST: What are the two main mechanisms that macrophages use to kill intracellular organisms?

1) Oxygen dependent 2) Oxygen independent

List two granules that NK cells secrete to kill the infected target cells, and describe what each granule does.

1) Perforin - Creates small pores on the cell surface 2) Granulysin - Enters the cells through those pores and damages the DNA of the target celsl

List the two nasal decongestants discussed

1) Phenylephrine 2) Pseudoephedrine

TEST: What are the 4 main parts of the developing diaphragm?

1) Pleuroperitoneal fold 2) Mesentery of body wall 3) Mesentery of Esophagus (Mesoesophagus) 4) Septum transversum

TEST: What are three main classes of anti-fungal drugs?

1) Polyenes 2) Azoles 3) Echinocandins

List the two oral corticosteroids (systemic), and which one is an active drug and which one is inactive

1) Prednisone - INACTIVE pro-drug 2) Prednisolone - ACTIVE

TEST/TEST: On a histological level, what are 4 developmental periods of lung maturation? Which stages are viable, and which stages are pre-viable?

1) Pseudoglandular period (6-16 wks) - PRE-VIABLE 2) Canalicular period (16-26 wks) - PRE-VIABLE, but DEPENDS on time of birth 3) Terminal sac period (26 wks to birth) - VIABLE 4) Alveolar period (32 wks to 8 years) - VIABLE

List at least three major complications of emphysema

1) Pulmonary hypertension (losing alveolar septae causes loss of alveolar capillaries, leading to an increased pressure in the existing vessels) 2) Cor pulmonale 3) CHF 4) Pneumothorax 5) Respiratory acidosis

What are the two major physiologic determinants of alveolar gas composition?

1) Ratio of VA' / Q' (pulmonary cap blood flow) 2) Respiratory exchange ratio, R

TEST: Give two mechanisms to why pulmonary vascular resistance decreases with an increase in blood flow

1) Recruitment: Opening of previously collapsed capillaries 2) Distention: Increasing diameter of open capillaries

TEST: Which three PFTs cannot be measured using simple spirometry? Which one actually needs to be measured experimentally, and using what techniques?

1) Residual volume (RV) 2) Functional residual capacity (FRC) 3) Total lung capacity (TLC) FRC needs to be measured using HELIUM DILUTION or BODY PLETHYSMOGRAPHY - From this, RV and TLC can be calculated

TEST: The nasal cavity is lined by two main "zones". What are these two zones, and list what areas of the nasal cavity each zone covers.

1) Respiratory zone - Lines the nasal septum, and all surfaces of the nasal cavity EXCEPT the roof, upper portion of nasal septum, and superior concha 2) Olfactory zone - Lines the SUPERIOR parts of the nasal septum and nasal cavity, superior concha

RECAP: List the most common 4 drug regimen as part of the INITIAL phase of TB treatment?

1) Rifampin 2) Isoniazid 3) Pyrazinamide 4) Ethambutol Can stop Ethambutol if isolate is susceptible to #1-3

TEST: List the two major side effects, as well as the major caution with 1st gen H1 antagonists

1) SEDATION 2) Anticholinergic effects (dry mouth, constipation, blurred vision) Major caution: *NARROW ANGLE GLAUCOMA, as it can increase intraocular pressure via relaxation of ciliary muscles*

TEST: What are the two extremes of V/Q inequality? Briefly define what each means and what the V/Q value will be

1) SHUNT - Perfusion of lung without VENTILATION (V/Q = 0) 2) DEAD SPACE - Ventilation of lung without PERFUSION (V/Q = infinite) - The gas enters and leaves lung without contacting blood

What are the two geometric forms of asbestos, and which one is more flexible?

1) Serpentines - more FLEXIBLE (curled structure, more soluble) 2) Amphiboles - more STIFF and straight

What are three classes of quick-relief medications for asthma?

1) Short-acting beta2 agonists 2) Inhaled anticholinergics 3) SYSTEMIC corticosteroids

What are the three major types of pulmonary function tests (PFTs)?

1) Spirometry (most important) 2) Lung volumes 3) Diffusion capacity

RECAP: List whether each of the following TB drugs is bactericidal or bacteristatic 1) Streptomycin 2) Isoniazid 3) Rifampin 4) Ethambutol 5) Pyrazinamide

1) Streptomycin - STATIC 2) INH - CIDAL 3) Rifampin - CIDAL 4) Ethambutol - STATIC 5) Pyrazinamide - CIDAL RIP = Cidal

List three arteries that supply the superior surface of the diaphragm

1) Superior phrenic A. 2) Pericardiacophrenic a 3) Musculophrenic a.

TEST: What are 4 major subtypes of CD4 T cells? List the major cytokines that each subtype produces, and what their major function is.

1) Th1: produces IFN-gamma - enhances functions of macrophages (INTRAcellular defense) 2) Th2: produces IL-4, 5 - Induces B cell maturation and Ab production 3) Th17: produces IL-17, induces inflammation at high conc (EXTRAcellular defense) 4) Treg: produces IL-10, suppresses immune response

NOT TESTED: What are two methylxanthines, and what is their main MOA?

1) Theophylline 2) Aminophylline MOA: Inhibits PDE III and IV, which prevents degradation of cAMP --> more cAMP --> PKA --> inhibited MLCK --> no smooth muscle contraction --> BRONCHODILATION

TEST: List the four main factors that affect the RATE of gas diffusion?

1) Thickness (the thicker the membrane is, the slower the diffusion) 2) Surface area (more SA, quicker diffusion) 3) Diffusion constant (based on solubility and MW) 4) Pressure difference across membrane

TEST: List at least 4 uses of Diphenhydramine (Benadryl)

1) Type I IgE-mediated hypersensitivity 2) Motion sickness 3) Night time sleep aid 4) Antitussive 5) Topical antipruritic 6) AntiEMETIC (anti-D2 receptor)

TEST: What are the two types of cells that are contained within the alveolar epithelium? What percentage of the epithelial cells does each cell type make up, and what percentage of the alveolar surface does each cell type cover?

1) Type I alveolar cells - 40% of total alveolar epithelial cells - Covers 90% of alveolar surface 2) Type II alveolar cells - 60% of total alveolar epithelial cells - Covers only 10% of the alveolar surface!!

TEST: What are the 5 most common interstitial lung diseases to know about?

1) Usual interstitial pneumonia (UIP) 2) Nonspecific interstitial pneumonia (NSIP) 3) Connective tissue disease 4) Hypersensitivity pneumonitis 5) Sarcoidosis

TEST: What are the two most important idiopathic interstitial pneumonias (lung disease)? What is the main distinguishing factor between the two?

1) Usual interstitial pneumonia (UIP) 2) Nonspecific interstitial pneumonia (NSIP) Main difference: *NSIP is more TREATABLE because it is more cellular*, UIP has a WORSE prognosis and is associated with fibrosis and scarring

TEST: What are the two major classes of anti-influenza agents? What is each class targeting, and what is the normal function of each of these targets?

1) Viral coating INHIBITORS - Targets the M2 protein on the surface of the virus, which is a PROTON CHANNEL that is in the viral envelope and is critical for VIRAL REPLICATION (by viral uncoating and release into host cells) 2) Neuraminidase inhibitors - Targets Neuraminidase, which normally cleaves sialic acid groups from the host glycoproteins to release the virus from the host cell (required for VIRAL REPLICATION)

What are the top three causes of wheezing?

1- ASTHMA 2- COPD 3- FOREIGN BODY OBSTRUCTION

TEST: In simple CWP, there are macules that are 1-2 (cm/mm) large with even larger coal nodules. The lesions are most heavily involved in what parts of the lung?

1-2 MM large macules Most heavily involved in the UPPER lobes and UPPER parts of the lower lobes *upper and lower lobes affected*

What are the FIVE factors that affect airway resistance?

1. Airway diameter: Decreases airway diameter increases resistance. 2. Turbulent Flow increases airway resistance. 3. Medium-sized airways provide greater resistance to flow than the small airways do. 4. Lung Volume: High lung volume has low resistance and low lung volume increases airway resistance. 5. Density & Viscosity of Gas: Increased density & viscosity increase airway resistance.

Any factor that decreases airway diameter, or increases turbulence will increase airway resistance. Based on that principle, fill in the blanks: 1. Rapid breathing ________ airway resistance. 2. Asthma _________ airway resistance. 3. Emphysema ___________ airway resistance.

1. increases (because air velocity and hence turbulence increases) 2. increases (there is narrowing of the airways, same with parasympathetic stimulation) 3. increases (there is decreased small airway diameter during forced expiration)

What is the ideal V/Q ratio?

1.0

****What enzyme converts prednisone to prednisolone? Where is this enzyme located?

11-beta hydroxysteroid dehydrogenase (11B-HSDI) in the LIVER *Pic is IMPORTANT*

Reducing r by 50% will increase R ____ fold

16 fold

In terms of notation, "A" means ______ while "a" means _______ (for this class)

A = ALVEOLAR a = ARTERIAL

What constitutes a pulmonary "lobule"?

A terminal BRONCHIOLE and the associated region of pulmonary tissues that it supplies (alveolar ducts, alveolar sacs, and alveoli)

What does a flow-volume loop allow you to see?

A visual picture of the airflow throughout the breathing cycle, and it allows IDENTIFICATION of OBSTRUCTION on either inspiration OR expiration

What is the systemic approach to reading an Xray...

ABCDEF

TEST: Mycobacteria tuberculosis are an _____ _____ bacteria that can be grown in specialized media such as _________-_______

ACID FAST bacteria - Grown in specialized media such as LOWENSTEIN-JENSEN media Recall: Acid fast implies that the bacteria resists de-colorization with dilute acid after staining, so they retain the stain color after the washing out

AGAIN: Systemic corticosteroids are used for (acute/chronic) asthma exacerbations

ACUTE

Erythrocyte sedimentation rate (ESR) may be elevated in (acute/subacute/chronic) hypersensitivity pneumonitis

ACUTE

Long acting beta agonists (LABAs) are contraindicated in what kind of asthma?

ACUTE asthma! - Also, LABAs without ICS use (inhaled corticosteroids) is also contraindicated in the Tx of asthma!

Respiratory (acidosis/alkalosis) is a common finding in patients with hypersensitivity pneumonitis

ALKALOSIS

Mast cell stabilizers can be used for asthma bronchospasms and _____ _______

ALLERGIC RHINITIS **Prevents EXERCISE-induced asthma

Hemoglobin changes confirmation as oxygen binds, exhibiting _________ properties. What exactly in hemoglobin is actually changing shape?

ALLOSTERIC properties - The *PLANE of HEME is actually bending with the binding/unbinding of oxygen*, so that it shows cooperative kinetics (one binds, and the next one will bind even tighter)

_______ vessels are mainly the capillaries and are exposed to alveolar pressure

ALVEOLAR (versus extra-alveolar)

All the expired CO2 comes from where?

ALVEOLAR gas

The majority of (amantadine/rimantadine) is excreted in the urine unchanged

AMANTADINE For Rimantadine, <25% is excreted unchanged

TEST: MOA of Streptomycin Is it bactericidal or bacteriostatic?

AMINOGLYCOSIDE - Inhibits protein synthesis by binding to 30S subunit BACTERIOSTATIC

Drug induced histamine release can cause an __________ reaction

ANAPHYLACTOID (burning, itching, intense warmth)

TEST: What is the physiologic dead space?

ANATOMIC + ALVEOLAR dead space

TEST: Most urban dwellers will develop ________, which is a milder and asymptomatic form of coal worker's pneumoconiosis, due to the constant exposure of air pollutants (from carbon emissions for example)

ANTHRACOSIS Macrophages with carbon in image

Dendritic cells (DCs) are considered the most efficient ______ ______ cells

ANTIGEN-PRESENTING cells - They still phagocytose the Ag!

Sarcoidosis can affect any ___ and usually affects people aged __ - ___. This disease is usually ___ on presentation

ANY ORGAN People 20-40 Usually asymptomatic

TEST: Intravascular pressures increase progressively from the ____ of the lung to the ____ of the lung

APEX to BASE - So that at the apex, there is hardly any blood flow and at the base, there is a lot of blood flow (so high intravascular pressure)

TEST: The _______ and _______ cartilage is primarily from neural crest associated with pharyngeal arch 6

ARYTENOID and CRICOID CARTILAGES

A shipbuilder/plumber or someone in BYC during 9/11 presents with Pulmonary sxs BIBASILAR FINE END INSPIRATORY CRACKLES HONEYCOMBING on imaging Ferruginous body (dumbbell asbestos fiber on histo)

ASBESTOS EXPOSURE

TEST: Echinocandins are active against ________ and ______ species

ASPERGILLUS and CANDIDA species

TEST: About 20% of asthmatics are sensitive to what common NSAID? Why?

ASPIRIN - Possibly due to a buildup of arachidonic acid from the inhibition of COX enzyme, which now causes more production of LEUKOTRIENES!

List some of the reasons of an INCREASED DLCO

ASTHMA (may be normal) Pulmonary hemorrhage POLYCYTHEMIA SHUNTS (L to R)

AGAIN: LABA by itself is contraindicated in treatment of (asthma/COPD)

ASTHMA! - In COPD, you CAN give LABA by itself as a first-line treatment, but this doesn't happen very often (you would supplement the LABA with something else) - However, in asthma, LABA by itself without steroids is an absolute contraindication

Isoniazid can lead to a transient (symptomatic/asymptomatic) elevation of ___/____ levels in the first 4-8 weeks of therapy

ASYMPTOMATIC elevation of AST/ALT

Vancomycin is known to cause ________ impairment that may be permanent

AUDITORY impairment (OTOTOXIC)

TEST: Cystic fibrosis shows what type of inheritance pattern? What gene does it affect, and what chromosome is this located on?

AUTOSOMAL RECESSIVE - Affects the transmembrane conductance regulator gene (CFTR), located on chromosome 7 - This causes a defect in CHLORIDE ION CONDUCTANCE, resulting in a buildup of MUCUS that can't be cleared

The primary treatment of hypersensitivity pneumonitis is what?

AVOID THE ANTIGEN!!! "every-time I go to the silo I get ...." helps differentiate between Pneumonia etc.

What is the pathogenesis of emphysema?

Abnormal permanent enlargement of the air spaces distal to the terminal bronchioles accompanied by destruction of alveolar WALLS

What is the normal value of the ratio of physiologic dead space volume (Vd) to tidal volume (Vt)?

About 0.3

TEST: By what week of development is the fetus actually viable?

About 24 weeks - Before 24 weeks, there isn't enough vasculature for the fetus to survive!

What is the main mechanism of resistance in C. albicans against azole antifungals?

Accumulation of mutations in ERG11, the gene coding for 14-alpha-sterol demethylase - This causes accumulation of 14-alpha methylsterols! Other possible MOAs of resistance: - Increased production of C14-alpha-sterol demethylase - Mutation of C5,6 sterol reductase gene ERG3

The major parasympathetic NT is __________, which can cause broncho(spasms/dilation)

Acetylcholine, which can cause bronchoSPASMS (via smooth muscle constriction)

How is Bordatella acquired and spread?

Acquired and spread by INHALATION of aerosol particles or droplets

Isoniazid is a prodrug that is activated by what enzyme? What gene is this enzyme coded on? On the other hand, isoniazid is inactivated by ________

Activated by CATALASE-PEROXIDASE, coded by the KatG gene Inactivated by ACETYLATION

TEST: Activating H1 histamine receptors causes vaso(constriction/dilation) and broncho(constriction/dialation) Briefly describe the mechanism behind this

Activating H1 histamine receptors: - VasoDILATION - BronchoCONSTRICTION Mechanism: Histamine binds H1 receptors, causes PLC --> IP3 and DAG --> Ca release from SR --> activates MLCK and PLA2 1) MLCK phosphorylates myosin, triggering BRONCHOCONSTRICTION 2) PLA2 increases NO and PGI2 (VASODILATORS)

List the three broad classes of hypersensitivity pneumonitis

Acute Subacute Chronic

ARDS stands for?

Acute respiratory distress syndrome

What laryngeal muscles are vocal fold ADDUCTORS? What laryngeal muscle is a vocal fold ABDUCTOR?

Adductors: - TRANSVERSE arytenoid m - LATERAL cricoarytenoid m Abductor: - POSTEROR CRICOARYTENOID m

What tumor can grow into the airway and cause obstruction of the airway and lead to stasis of secretions in the airway ? What can this ultimately lead to?

Adeno-Carcinoma Lead to BACTERIAL INFECTION Leads to destruction of the airway

TEST: As a caution, albuterol can delay ______

Albuterol can delay LABOR!! (through relaxation of uterine smooth muscle)

TEST: What is the only SABA discussed in class?

Albuterol!

What are some possible risk factors that exacerbate asthma?

Allergens Respiratory infections Exercise and hyperventilation Weather change Sulfur dioxide Food, additives, and drugs

Allergic asthma occurs more frequently in (adults/children), while nonallergic asthma occurs more frequently in (adults/children)

Allergic asthma: More frequent in CHILDREN Non-allergic: More frequent in ADULTS

****What is the only known genetic abnormality that leads to COPD (emphysema)?

Alpha-1 antitrypsin (AAT) deficiency - Major role is inhibition of neutrophil protease

What is the definition of hypersensitivity pneumonitis? (what are they caused by)

Also called extrinsic allergic alveolitis - Caused by a wide variety of *INHALED organic dusts or chemicals producing an immunologically mediated inflammatory response* of the alveoli and terminal bronchioles

Alveolar air (does/does not) have the same gas concentrations as atmospheric air. List at least 3 reasons why.

Alveolar air DOES NOT have the same gas concentrations as atmospheric air, because: 1) Alveolar air is PARTIALLY replaced by atmospheric breath during each breath 2) Oxygen is constantly absorbed into blood 3) CO2 is constantly diffusing into alveoli 4) Air becomes humidified as it is inspired!

List some things that decrease the DLCO

Alveolar fibrosis (restrictive) Space occupying tumors Loss of lung tissue (emphysema))

What component of the immune system acts as a *first line protective barrier in the lungs against inhaled particulates?*

Alveolar macrophages

AGAIN: What is the normal PAO2 and PACO2 at sea level?

Alveolar oxygen = 100 mmHg Alveolar CO2 = 40 mmHg

TEST: Why do changes in alveolar ventilation rate affect alveolar air composition?

Alveolar ventilation brings FRESH AIR into the alveoli, so that as alveolar ventilation increases the air composition becomes similar to that of humidified inspired air! (you are breathing in more fresh and thus inspired air)

TEST: What gradient can you use to diagnose hypoxemia?

Alveolar-arterial gradient (A-a gradient): - PAO2 - PaO2

What are the two things we absolutely need to breath? (think small structures)

Alveoli and capillaries!! - Alveoli provide the respiratory gas exchange surface, and capillaries provide the pathway of blood that carries the gasses!

TEST: Out of all the drug induced interstitial lung disease, which drug is the MOST common? What are some other drugs that can induce interstitial lung disease?

Amiodarone Others: - Nitrofurantoin - Gold - Dilantin - Bleomycin - Cytoxan

The amount of dust retained in the lungs is determined by the _______ of the dust in air, the _______ in the lungs, and how well the lungs can ______ the debris

Amount of dust retained in lungs is dependent on: - CONCENTRATION in air - DURATION in the lungs - How well the lungs can CLEAR the debris

TEST/AGAIN: The anatomic dead space is the volume in the ________ airways, which spans from the ______ to the _______. This is on average about ______ mL in a normal adult

Anatomic dead space = VOLUME in the CONDUCTING airways, from the TRACHEA to the BRONCHIOLES. This is ~150 mL in a normal adult

Flip card for case:

Answer: - - - - - - - B

How does the nasal cavity proper communicate anteriorly? Posteriorly? Where does the nasal cavity proper communicate laterally?

Anterior: Through anterior nares Posterior: to the nasopharynx via CHOANAE Laterally: to the paranasal sinuses and nasolacrimal duct

AGAIN: What is the most innocuous (harmless) coal-induced pulmonary lesion?

Anthracosis! Urban living

The apex of the lung sits on the (midclavicular/midaxillary) lung

Apex is at the MIDAXILLARY lung

Which second gen H1 antagonist is: - Appetite stimulating - Hydroxyzine metabolite

Appetite stimulating: LORATADINE (Claritin) Hydroxyzine metabolite: CETIRIZINE (Zyrtec)

TEST: As alveolar ventilation increases, the alveolar PO2 (increases/decreases) and the alveolar PCO2 (increases/decreases)

As alveolar ventilation INCREASES: - Alveolar PO2 INCREASES - Alveolar PCO2 DECREASES As the alveoli are ventilating more, they receive more oxygen and get rid of more CO2 (since all alveolar CO2 will get breathed out)

Diffuse pulmonary fibrosis due to inhalation of excess asbestos fibers is linked to what disease?

Asbestosis! Roofing/shipyards

Which fungal infection can obstruct the airway?

Aspergilloma

AN 18 yo black man presents with wheezing and Difficulty breathing. Attacks were intermittent and not related to any circumstance. X-ray of the chesst was normal. Lung function showed that there was a MARKEDLY DECREASED FEV1 but improved after he inhaled β-ADRENERGIC AGONIST...

Asthma

Wheezes are connected to

Asthma COPD Foreign Body

What are the four items usually indicated under obstructive lung disease?

Asthma Emphysema Chronic Bronchitis Cystic Fibrosis

Asthma is a chronic _________ disorder that causes (hypo/hyper)-responsive airways leading to airflow _________ that usually is (reversible/irreversible)

Asthma = *Chronic INFLAMMATORY disorder* that causes *HYPERresponsive airways leading to airflow OBSTRUCTION* (narrowing) that *usually is REVERSIBLE* *thick, sticky mucous plugs seen in asthma pt.*

Asthma is a chronic _________ disorder that causes (hypo/hyper)-responsive airways leading to airflow _________

Asthma = Chronic INFLAMMATORY disorder that causes *HYPERresponsive airways leading to airflow OBSTRUCTION (narrowing)*

****What is the primary difference between asthma and emphysema?

Asthma is REVERSIBLE, and the PFTs SHOULD return back to normal for asthma (but not for emphysema)

Status asthmaticus is essentially a prolonged ___ ______.

Asthmatic attack!

TEST: The diaphragm gives rise to three openings at its (anterior/posterior) surface. List the vertebral levels and the major structure that passes through each of these openings.

At POSTERIOR surface: 1) T8 - IVC 2) T10 - Esophagus 3) T12 - Aorta

Emphysema often doesn't present with symptoms until ____ of the functional capacity is lost

At least 1/3

TEST: In order to make a Dx of a restrictive lung disease, how many tests do you need to do, and what are they?

At least TWO tests: 1) Spirometry 2) Confirm with LUNG VOLUMES

Give the regular RR range at rest and during exercise

At rest: 10-20 times/min During exercise: 40-45 times/min

TEST: At the end of expiration, the muscles of respiration are (tense/relaxed). The lung has a natural (outward/inward) elastic recoil, while the chest wall has a natural (outward/inward) elastic recoil

At the end of expiration, the muscles of respiration are RELAXED. The lung has a natural INWARD elastic recoil, while the chest wall has a natural OUTWARD elastic recoil. These BALANCE each other out!

TEST: Voriconazole is the azole of choice for invasive ________, with a common side effect of what?

Azole of choice for invasive ASPERGILLOSIS Common side effect: VISUAL DISTURBANCES (blurry or changes in vision color)

Which leukotriene is responsible for producing mucosal edema? Which leukotrienes are potent spasmogens?

B4, as it increases VASCULAR PERMEABILITY *C4, D4, and E4*

TEST: What is the MOA of Ampicillin and amoxicillin?

BACTERICIDAL, binds and inhibits penicillin-binding proteins to inhibit CELL WALL SYNTHESIS

RECALL: What is the MOA of macrolides?

BACTERIOSTATIC, inhibiting protein synthesis by binding reversibly to the 50S ribosomal subunit (specifically the tRNA binding site)

TEST: What is the MOA of Doxycycline?

BACTERIOSTATIC, inhibiting protein synthesis by binding to the 30S bacterial ribosome, preventing access of aminoacyl tRNA to the ACCEPTOR site

A common finding of casts in the bronchus which are spiral shaped mucus plugs from subepithelial mucous gland ducts or bronchioles are known as?

BAL - CURSCHMANN's SPIRAL

A heavy exposure to dust or fumes of BERULLIUM causes ___ and is most common in ___ and ____ industries

BERYLLIOSIS Aerospace / Nuclear Industries

TEST: What is a potential adverse reaction of Ceftriaxone with prolonged use?

BILIARY STASIS

A farmer is breeding birds. His or her hypersensitivity pneumonitis is occuring in the summer. This is most likely (farmer's/bird-breeder's) lung.

BIRD-BREEDERS - Remember, farmer's lung typically occurs in the WINTER! *Down comforters can also cause HP*

TEST: Complicated CWP shows multiple _______ scars that are greater than ___ cm in size

BLACKENED scars that are greater than 2 cm in size

TEST: Chronic bronchitis patients are referred to as ____ _______, largely due to the fact that they are often _________

BLUE BLOATERS, due to the fact that they are often HYPERCAPNIC (can't breath out due to airflow obstruction, builds up CO2)

TEST: How do you calculate the physiologic dead space? Give the equation (you need to know this one).

BOHR METHOD: *Vd /Vt = (PaCO2 - PeCO2) / PaCO2* Vd = Dead space volume Vt = Tidal volume (volume exhaled with each breath) PaCO2 = Partial pressure of CO2 in arterial blood PeCO2 = Partial pressure of CO2 in expired air

What arteries supply the bronchi and lung tissues? Why are these arteries here?

BRONCHIAL arteries - These are here because the oxygen within the bronchi can't actually pass through the cartilage, so without these arteries the bronchi and lung tissue would die!

TEST: Obstruction of the airway by mucus in chronic bronchitis can lead to _________ or ________, a decrease in a part of the lung volume

BRONCHIECTASIS (dilation of bronchi caused by destruction) or ATELECTASIS (decrease in part of lung volume)

Essentially, methylxanthines are potent _________

BRONCHODILATORS

In patients with COPD, they may not respond to what treatment? Why is this clinically relevant?

BRONCHODILATORS! - But, you would still TREAT them anyways!

When the pneumonia is focused diffusely throughout the lung it is known as?

BRONCHOPNEUMONIA

TEST: In general, all beta2 selective agonists can induce paradoxical _________

BRONCHOSPASMS!

AGAIN: Short acting beta agonists (SABAs) are the primary treatment for what symptom of asthma?

BRONCHOSPASMS, causing INTERMITTENT asthma *SABA's Tx of Choice*

What blood has LOST 2,3 BPG?

Banked Blood Affinity shifts left...need to add blood infusion SLOWLY to allow BPG to build up.

Which cell type in the respiratory zone has the ability to differentiate into other cell types?

Basal cells

How come when you exercise and the cardiac output increases that mean pulmonary artery pressure doesn't increase with it?

Because as pulmonary blood flow increases, the pulmonary vascular RESISTANCE DECREASES, keeping the pulmonary artery pressure relatively the same Remember: - Vascular resistance = (input pressure - output pressure) / blood flow

Why aren't the reactive oxygen intermediates created by macrophages enough to kill all of the ingested microbes? Besides these intermediates, macrophages can also generate what other intermediates that are efficient in controlling intracellular pathogens?

Because many microbes can synthesize SUPEROXIDE DISMUTASE to DEGRADE superoxides, as well as CATALSE to degrade H2O2!! This is why macrophages also generate reactive NITROGEN intermediates (like NITRIC OXIDE)!!

AGAIN: As the air passes from the atmosphere into the lungs, how come there is a drop from ~150 to ~100 mmHg for the PO2?

Because the *air inside the lungs is HUMIDIFIED, so the PO2 accounts for the PH2O in the lungs* (which is about 50 mmHg)

RECALL: If we are using the ALVEOLAR gas equation, why do we use the partial pressure of CO2 in ARTERIAL blood?

Because the CO2 rapidly diffuses from blood to alveoli! So we are approximating PACO2 by using PaCO2, which is much easier

Why is spirometry alone not enough to Dx restrictive defects?

Because there could be a decrease in FVC due to restriction, OR due to a consequence of AIR TRAPPING due to airflow obstruction

What are two commonly prescribed nasal steroids?

Beclomethasone diproprionate Flunisolide For LONG term control

TEST: In a flow-volume loop, ________ is below the X-axis and _______ is above

Below the X-axis: INSPIRATION Above the X-axis: EXPIRATION

TEST: Below the epiglottis, the mucosa forms 2 pairs of folds separated by what? List what each pair of folds form in the larynx.

Below the epiglottis, the mucosa forms 2 pairs of folds separated by a LARYNGEAL VENTRICLE: 1) Upper pair --> FALSE vocal vords 2) Lower pair --> TRUE vocal cords

TEST: Patients with benign ______ ______ should be cautioned before using anticholinergics. Why?

Benign prostatic hyperplasia, as anticholinergics can cause URINARY RETENTION

TEST: What is the mechanism of anticholinergic drugs?

Blocks the M2 and M3 muscarinic receptors in the bronchi, preventing the effects of ACh and thus bronchospasms (promotes bronchodilation)

TEST: What is the mechanism of leukotriene receptor antagonists?

Blocks the cysteinyl LT-1 receptor, decreasing: - Mucous production - Eosinophil infiltration - Hyperplasia

In a chest XR, what is the silhouette sign?

Blurred edge on Lower R edge, pneumonia blocks where the atria of the heart are viewed. Anterior R middle lobe

In a typical xray how do the following appear? Bone Calcification Soft Tissue Fluid Fat Gas

Bone - White/grey Calcificaiton - same density as bone Soft tissue - Grey Fluid - Same as soft tissue Fat - Dark grey but lighter than soft tissue Gas - black

Where should the lungs be auscultated? Anterior lobes are? Posterior?

Both anteriorly and posteriorly Anterior = Upper and middle lobe Posterior = Lower lobe

A severe type of emphysema, preferentially found in the UPPER LOBE and often associated with a pneumothorax is called a?

Boullous emphysema

Bowman's glands within the ______ ______ of the olfactory zone secretes fluid that traps _______-______ proteins, which have a high affinity for _______ molecules. This fluid is bathing the olfactory _______ that extended from the olfactory ______, the ends of the olfactory dendrites. This allows the proteins in the fluid to bind to receptors on the olfactory ______ and ultimately send signals for smell!

Bowman's gland within the LAMINA PROPRIA of the olfactory zone secretes fluid that traps ODORANT-BINDING proteins (OBPs), which have a high affinity for ODORANT molecules. This fluid is bathing the olfactory CILIA that extended from the olfactory KNOB, the ends of the olfactory dendrites. This allows the proteins in the fluid to bind to receptors in the olfactory CILIA --> SMELL signals!!

According to ___ law, the ___ of the gas is inversely related to its ____ when what remains constant?

Boyle's LAW The PRESSURE of the gas is inversely related to its VOLUME when TEMPERATURE and GAS molecules are constant If volume decreases the pressure increases

RECALL: Brachial grooves, also called pharyngeal ______, are lined with (endoderm/ectoderm). Branchial pouches on the other hand are (endodermal/ectodermal) outpouches of the (rostral/caudral) foregut that are located between ________ branchial arches

Branchial grooves = pharyngeal CLEFTS, lined with ECTODERM (grooves between arches) Branchial pouches are ENDODERMAL outpouches of the ROSTRAL foregut that are located between ADJACENT branchial arches

A thickening of the bronchi due to chronic inflammation is?

Bronchiectasis *Seen in Chronic BRONCHITIS*

TEST: A 52 year old woman has had increasing *frequency of intermittent bouts of cough, fever, and copious amounts of foul-smelling purulent sputum* over the past 3 years. She was treated with antibiotics for bronchitis 6 times in the previous 18 months. She now has increasing dyspnea and orthopnea, and after failing to take the prescribed antibiotics she dies. Autopsy shows markedly distended peripheral bronchi, primarily in the LOWER lobes, as well as bronchial wall destruction. What is top on your differential?

Bronchiectasis!

Part of the pathogenesis of bronchiectasis is destruction of the airways due to an excessive _______, which then causes those bronchi to be ________ and thus less able to clear _______. This builds up and increases the risk of further ______ and _____ the airways

Bronchiectasis: destruction of airways due to an excessive INFECTION, which then causes those bronchi to be DILATED and thus less able to clear SECRETIONS. This builds up and increases the risk of further INFECTION and OBSTRUCTION of the airways

What technique allows one to get a biopsy of ILD?

Bronchoalveolar lavage (using a bronchoscope to aspirate a sample for cytology and culture)

Patients with a new obstructive defect on spirometry should have what test, and why?

Bronchodilator test, to determine their responsiveness as a treatment!

C-AMB is complexed with a bile salt, ________. This forms a _______ solution, and should not be used with ______ filters and ________ in the infusion line because they can cause the solution to ________

C-AMB is complexed with DEOXYCHOLATE. This forms a COLLOIDAL solution and should NOT be used with: - INLINE filters and ELECTROLYTES in the infusion line because they can cause the solution to AGGREGATE

The cricoid cartilage is found a the ___ vertebra

C6

What is the normal activator of the myosin light chain kinase in the context of muscle contraction?

CALCIUM! - When it enters smooth muscle cells and binds to CALMODULIN, which then activates MLCK

TEST/TEST: In which developmental period of lung maturation does SURFACTANT first get produced? What does this mean about the viability of the fetus?

CANALICULAR period, around 20 weeks This means that birth after 21 weeks COULD mean survival, but the chances are still pretty low - 10% at 22 weeks - 50% at 23 weeks

RECALL: What enzyme converts carbon dioxide and water to bicarbonate?

CARBONIC ANHYDRASE (via carbonic acid)

The conducting zone structures contains ________ that holds the tube system open and ____ ______ that controls the tube diameter

CARTILAGE - holds tube system open SMOOTH MUSCLE - controls tube diameter

TEST: Patients with invasive aspergillosis who fail to respond to Amphotericin B and itraconazole is put on what drug as salvage therapy?

CASPOFUNGIN

TEST: Terminal bronchioles are lined by what kind of epithelium? What cells are located within this epithelium, and what is their primary function?

CILIATED CUBOIDAL epithelium Cells = CLARA cells - Main function: Contains secretory granules (Clara cell secretory protein) that DETOXIFY against oxidative pollutants (inhaled into the lungs) and inflammation. They also secrete components of SURFACTANT.

AGAIN: The helium dilution technique is a (open/closed) loop

CLOSED (there are no leaks in the system, and helium can't get out)

2nd Gen drugs here have little or no ___ penetration These antihistamines are ___ acting

CNS = Low drowsiness LONGER acting

What is the problem with moving CO2 as a waste in blood directly, without any carriers or modification? TEST: What is the solution?

CO2 is still extremely INSOLUBLE in aqueous solution, so it cannot be transported efficiently to excrete it by itself Solution: Convert CO2 into a highly soluble BICARBONATE ion

Iatrogenic PTX's are a ___ of a diagnostic or therapeutic intervention

COMPLICATION - central line

The lower conducting portion contains the ________ zone, which spans from the ______ to the terminal _______

CONDUCTING ZONE, which spans from the TRACHEA to the terminal BRONCHIOLES

Wheezing is generally more ___ while crackles are more ___

CONTINUOUS Crackles: DISCONTINUOUS

Chronic hypersensitivity pneumonitis is due to _______ or (high/low) intensity exposure for _____ to ______

CONTINUOUS or LOW intensity exposure for MONTHS to YEARS

Increase in resistance of the peripheral airways can exaggerate the dynamic compression of airways, such as in the airway disease called _______________.

COPD

If a pt is leaning forward with their arms outstretched, a possible cyanosis could be presenting with?

COPD Leaning forward helps with ventilation

Ipratropium is used to treat bronchospasms associated with what?

COPD (chronic bronchitis and emphysema) and asthma

Anticholinergics 2

COPD DRUG OF CHOICE = ANTICHOLINERGICS

RECALL: Emphysema and chronic bronchitis are both under what overarching diagnosis?

COPD!

Air trapping, a hyperinflated lung, and a flattened diaphragm are common signs of what?

COPD!!

COPD vs ASTHMA facts:

COPD: - Mid life - slow progressing sxs - LONG smoking Hx

*****The most important thing when reading a film is to make sure you have the ___ ____ What are some things that can be checked?

CORRECT PATIENT Check name, medical record # , DOB, Date of the film

Dextromethorphan is used to suppress __ and acts on the ____ receptor in the brain as an ____

COUGH suppressant acts on NMDA receptor as an ANTAGONIST (cough center)

After looking at the correct patient identification info, you check ___ which is the AREA you want to see Next is to check ____ which is over/under penetration

COVERAGE = area you want to see EXPOSURE = over/under penetrated

What membrane joins the thyroid and cricoid cartilages? What is clinically relevant about this?

CRICOTHYROID membrane - Can do emergency tracheotomies here!

RECALL: What drug inhibits mast cell degranulation and acts as a mast cell stabilizer? What is its MOA?

CROMOLYN SODIUM - MOA: Inhibits Cl transport needed for Ca-induced histamine release

What substance allows for easy clinical measurement of diffusion capacity? Why?

Carbon monoxide, as its gas exchange is DIFFUSION-LIMITED and is the most SENSITIVE to abnormalities of the alveolar-capillary membranes

The cardiac notch on the (right/left) lung leaves a __________ recess

Cardiac notch on the LEFT lung leaves a COSTOMEDIASTINAL recess

Isoniazid can cause peripheral _______, which can be prevented by giving the patient vitamin ____

Cause PERIPHERAL NEUROPATHY, prevented with vitamiin B6

What are some of the passive influences on pulmonary vascular resistance?

Changing cardiac output Changing lung volume Changing the HYDROSTATIC pressure of the blood vessels

TEST: Restrictive defects are characterized by a reduced (flow/volume), and it can be indicated on spirometry by what value, and what is the limit?

Characterized by a reduced VOLUME, and it can be indicated by: 1) FVC < 80%, or 2) TLC < 80% (>120% = Obstruction)

Before considering a step up in asthma tx's, what needs to be done?

Check inhaler techniques and adherence.

AGAIN: Classic cystic fibrosis is characterized by what?

Chronic BACTERIAL INFECTION

What are some of the common diseases that fall under COPD?

Chronic bronchitis Emphysema Bronchiectasis

elastic properties of both the lung and chest wall determine their _____ _____. At the end of expiration, (FRC), the inward recoil of the lung is ____ by the ____ Spring of the chest wall The chest wall tends to expand at volume up to ___% the total lung capacity.

Combined compliance FRC = functional residual capacity the lung is *balanced by the outward spring of the chest wall* 75%

What is a POLYPHONIC wheeze? What diseases experience Polyphonic wheezing?

Combo of different musical pitches that START and STOP and DIFFERENT TIMES during the expiratory cycle SMALL AIRWAY obstruction characteristic ASTHMA/COPD

What is the one major function of histamine in the CNS?

Contributes to the SLEEP/WAKE rhythm cycle - Which is why antihistamines often have a sedative effect

List some of the general principles of treating CF (3)

Controlling the infections Treating the inflammation Treating the airway obstruction

What space contains pleural cavity but no lung?

Costodiaphragmatic recess - The lung can't get into this tiny space between the diaphragm and the ribs

What are some of the pulmonary symptoms of CF?

Cough Sputum production Rhinitis with/without sinusitis Hemoptysis WHEEZING Dyspnea Chest pain

List a few of the most common symptoms of asthma

Cough, dyspnea, wheezing, chest tightness

Again: Sound heard mainly in INSPIRATION?

Crackle

What are the three main ADVENTITIOUS breath sounds?

Crackles Wheezes Pleural rub

What muscle forms the upper esophageal sphincter?

Cricopharyngeus muscle

TEST: What major immunologic event triggers histamine release?

Cross linkage of IgE on the mast cell surface with the antigen (via second exposure)

Asthma treatment it's focused I'm two things. The frequency and intensity of the patients symptoms and functional limitations is known as ___ ____ while the likelihood of untoward events (exacerbations) is ___

Current Impairment Risk

What are the two main types of bronchiectasis?

Cystic fibrosis, and non-cystic fibrosis

QUIZ: A 49 year old African American woman comes to the physican because of shortness of breath, cough, and chest pain, Chest x-ray shows hilar LAD and biopsy of the lymph node is consistent with a diagnosis of sarcoidosis. Which of the following histologic findings in the biopsy specimen best confirms the diagnosis? A) Aschoff bodies B) Calcifications C) Caseous necrosis D) Epitheloid histiocytes

D) Epitheloid histocytes

TEST: If a PE is unlikely after assessing the clinical probability, then what test do you do? What criteria determines whether further evaluation is required?

D-dimer assay (to EXCLUDE PE) - But if D-dimer is >500 ng/mL, then you still need to do a CT angiogram!

What is the other disease that goes away after smoking is stopped?

DIP

Diffusion of gases across the alveolar-capillary membranes requires gas molecules to _______ in the substance of the membranes

DISSOLVE

TEST: The OVERALL pathogenesis of Bordatella pertussis results in what?

DEATH of ciliated epithelial cells --> DECREASED CILIARY ACTIVITY

AGAIN: Increases in blood flow and intravascular pressure (increase/decrease) the pulmonary vascular resistance. Why?

DECREASE, due to RECRUITMENT and DISTENTION of pulmonary capillaries!

Generally, DLCO is (increased/decreased) in restrictive diseases

DECREASED Normal = 25 ml/min/mmHg

TEST: For restrictive lung disease, you would expect a (increased/decreased) TLC, FVC, and FEV1/FVC ratio (can be different for all three)

DECREASED TLC and FVC INCREASED FEV1/FVC ratio

TEST: (Increased/decreased) pH and (increased/decreased) PCO2, relative to normal, will decrease the affinity of hemoglobin for oxygen

DECREASED pH and INCREASED pCO2 will DECREASE the affinity of hemoglobin for oxygen

As surface tension increases, the size of the alveoli (increases/decreases)

DECREASES (because the fluid lining the walls of the alveoli is pulling and becoming tighter, drawing the alveolar walls inwards)

TEST: Surface tension (increases/decreases) the compliance of the lung and maintains ________ of the lung

DECREASES compliance of the lung and maintains HYSTERESIS of the lung

As pulmonary blood flow increases, pulmonary vascular resistance (increases/decreases)

DECREASES!

AGAIN: Type IV hypersensitivity reactions is a ______ reaction that involves the activation of ___-cells of the immune system

DELAYED type reaction (2-3 days), involves activation of T-cells of the immune system

Sound travels faster through ___ ___

DENSER MATERIAL (Tissue)

TEST: A (phosphorylated/dephosphorylated) myosin light chain KINASE leads to muscle contraction. Explain.

DEPHOSPHORYLATED myosin light chain KINASE, as MLCK is INACTIVE when phosphorylated. - And if MLCK is inactive, it can't phosphorylate myosin to permit contraction! Therefore, MLCK needs to be DE-phosphorylated to stay active and allow muscle contraction!!

TEST: Between DIFFUSION-limited and PERFUSION-limited gas exchange, which one has abnormal PO2 levels in the alveoli?

DIFFUSION-limited

Phenylephrine causes eye pupils to (dilate/constrict)

DILATE (it acts as a MYDRIATIC)

List the major structural change seen in emphysema

DILATION of the airspaces (with loss of the alveolar walls)

TEST: If you suspect a patient has PE but their D-dimer is negative, the patient (does/does not) have PE

DOES NOT! (you do not have a DVT, and you do not have a PE) *The role of the D-dimer test is to rule OUT PE* The D-dimer test is highly SENSITIVE but NONSPECIFIC screening test

TEST: What is the only side effect of tiotropium?

DRY MOUTH (Xerostomia) Bitter taste

_______ compression of the airways limits air flow during forced expiration, and the flow is determined by ________ pressure

DYNAMIC compression of the airways limits air flow during forced expiration, and the flow is determined by TRANSPULMONARY pressure (alveolar - intrapleural) - This is different from normal breathing, which is driven by pressure gradient between alveoli and atmosphere

______ is usually the first symptom of asbestosis

DYSPNEA (on exertion)

Two very common symptoms of most interstitial lung diseases are what?

DYSPNEA (progressive) and a NONPRODUCTIVE cough

TEST: What is a dead space? How many of these dead spaces are there in our ventilation system, and what are they?

Dead space = area that *does NOT participate in gas exchange* THREE dead spaces in ventilations system: 1) ANATOMIC (fixed) dead space (~150ml) 2) ALVEOLAR dead space 3) PHYSIOLOGIC dead space

Around what weeks do the lung buds start forming?

End of the 4th - Early 5th week - Right after gastrulation

What are the two major effects of surface tension in regards to the lungs?

Decreases compliance of the lung Maintains hysteresis of the lung

TEST: What is the MOA of alpha adrenergic receptor agonists?

Decreasing resistance to airflow and decreasing volume of nasal mucosa by ACTIVATING alpha receptors in VENOUS CAPACITANCE vessels in nasal tissues

The pathogenesis of asbestosis depends on ________ interaction, which in turn will release _______ and ________ mediators

Depends on MACROPHAGE interaction, which in turn will release CHEMOTACTIC and FIBROGENIC mediators

TEST: What is Caplan's Syndrome? What is the classical finding of this that will help make your Dx?

Developing Rheumatoid pneumoconiosis, which is a combination of *rheumatoid arthritis and pneumoconiosis* Classical finding: RHEUMATOID NODULES (can be small for pneumoconiosis, or large for massive fibrosis) with OR without clinical rheumatoid arthritis

TEST: ...What does DIFFUSION limited gas exchange mean? What is one example of this?

Diffusion-limited gas exchange refers to *diffusion of a substance from the air to the blood being limited by the AMOUNT of DIFFUSION* - Example: Carbon monoxide NEVER equilibrates into the capillary blood, as CO is bound immediately to hemoglobin. The only way to get more CO exchange into blood is by increasing the DIFFUSION (by increasing surface area, or having a thinner barrier, etc...)

What physical sign on the fingers do you often see with CF?

Digital CLUBBING

CRACKLES are ___. These sounds involve ____ _____ and are common during ____ The old school term for this was ____

Discontinuous involve: SMALL AIRWAY Common during INSPIRATION ***RALES***

TEST: MOA of Pyrazinamide Is it bactericidal or bacteriostatic?

Disrupts M. tb cell MEMBRANE BACTERICIDAL

Atropine has several side effects, so in order to get away from those, we can use two other drugs, why do these not have as severe side effects?

Do not cross epithelial layer in the lung POOR SYSTEMIC ABSORPTION Lasts 4-6 hrs LONGER with B2- adrenergic agonist

What noninvasive test is very specific and sensitive at picking up DVTs? What are some limitations to this test?

Doppler ultrasound Limitations: Patient obesity, edema, and tenderness, as well as any casts or immobilization devices

The only peptide mucolytic agent approved for use in United States is ____ and it is used for treatment of __ ____

Dornase Alfa CF lung disease

List three drugs that increase azole concentrations by decreasing P450 enzymes. List three drugs that decrease azole concentrations by increasing P450 enzymes

Drugs that INCREASE azole conc: 1) Cyclosporine 2) Warfarin 3) Quinidine Drugs that DECREASE azole conc: 1) Rifampin 2) Isoniazid 3) Phenobarbital

What is meant by "dynamic compression of airways"?

Dynamic compression of the airways results when intrapleural pressure equals or exceeds alveolar pressure, which causes dynamic collapsing of the lung airways. It is termed dynamic given the transpulmonary pressure (alveolar pressure − intrapleural pressure) varies based on factors including lung volume, compliance, resistance, existing pathologies, etc.

AGAIN: What are some of the classic symptoms of bronchiectasis?

Dyspnea Chronic cough (with large amounts of FOUL-SMELLING purulent SPUTUM) Hemoptysis Fever (following infection)

TEST: Genetic testing for cystic fibrosis should NOT be offered to which of the following? A) A 32 year old couple planning to have pregnancy B) A 22 year old with positive family history of CF C) A 30 year old female with a partner diagnosed with CF D) A 30 year couple seeking prenatal testing E) 30 year old female smoker with uncle diagnosed with CF

E) 30 year old female smoker with uncle diagnosed with CF - The uncle is NOT a FIRST-DEGREE RELATIVE!!!!!!!

TEST: There are two stages of asthma, an ______ and a ______ stage

EARLY and a LATE stage

TEST: What is the early stage of asthma caused by, and what are the subsequent effects?

EARLY stage is due to *DEGRANULATION of mast cells* due to secondary exposure of the same or cross-reacting antigen to *PRESENSITIZED IgE-coated mast cells.* This causes the release of: - *Histamines* (bronchospasm) - *Leukotrienes* (inflammation, mucus) - *Platelet activating factor* (causes more release of histamine and serotonin from platelets) - *Mast cell TRYPTASE* (inactivates vasoactive intestinal peptide, a bronchodilator) All of these cause or promote BRONCHOCONSTRICTION!

List what goes on in the early and late phases of chronic bronchitis

EARLY: *Hypersecretion of mucus in LARGE airways* by submucosal glands LATE: Hypersecretion of mucus in SMALL airways by GOBLET CELLS

_________ are rarely indicated in acute PE

EMBOLECTOMY

TEST: The pressure outside and the pressure inside the lungs are equal to zero during what parts of the respiratory cycle?

END OF EXPIRATION and INSPIRATION (no air movement)

TEST: The lung is derived from (ectoderm/mesoderm/endoderm)

ENDODERM!!

TEST: In what compartment of the DC is the phagocytosed antigen processed and associated with MHC class II?

ENDOSOMAL compartment (complex Ag protein is broken down and associated with MHC II, and the entire complex is transported to cell surface of DCs)

The ________ fascia lines the parietal pleura, and at the apex it thickens to become ______ Fascia. What is clinically significant about this?

ENDOTHORACIC fascia lines the parietal pleura, and at the apex it thickens to become SIBSON'S Fascia. - This is clinically relevant because when you're doing surgery up in the neck, and you poke a hole in this fascia, you poke a hole in the underlying parietal pleura. This introduces AIR into the parietal cavity --> PNEUMOTHORAX!!

TEST: The ______ and ______ cartilage is primarily from neural crest _________ in pharyngeal arch 4

EPIGLOTTIS THYROID CARTILAGE Primarily from neural crest MESENCHYME in pharyngeal arch 4

The laryngopharynx spans from the ________ down to the ________ cartilage of the larynx It is the part of the throat that connects to the ________

EPIGLOTTIS down to the CRICOID cartilage of the larynx It is the part of the throat that connects to the ESOPHAGUS

List one physiological antagonist of histamines

EPINEPHRINE!

TEST: What establishes the driving pressure in the airways and divides it into downstream and upstream segments? When is this established?

EQUAL PRESSURE POINT (EPP) - This is the point where the *alveolar pressure is EQUAL to the pleural pressure* - This is established at the *PEAK EXPIRATORY flow when the pressure inside airway = pressure outside airway*

TEST: A classical skin finding of Lofgren's Sydrome is _______ _____ with _________ (type of pain) and bilateral ________ lymphadenopathy (triad of findings)

ERYTHEMA NODOSUM POLYARTHRALGIA Bilateral HILAR LAD Lofgren's Syndrome is just a type of sarcoidosis

TEST: Rapid infusion of vancomycin may cause ________ reactions with extreme _________ of the skin, sometimes called "___-____" syndrome

ERYTHEMATOUS reactions with extreme FLUSHING of the skin, sometimes called "RED-MAN" syndrome

Which macrolide was first isolated? What modifications have been made to the subsequent macrolides that made them safer?

ERYTHROMYCIN was isolated first Clarithromycin has higher bioavailability, while azithromycin resulted in better GI TOLERABILITY as erythromycin is well known for causing GI disturbances

The lung buds form and descend together with the developing _________

ESOPHAGUS

RECALL: The adult pharynx ultimately continues as the _______ around what cervical vertebra?

ESOPHAGUS, around C6

The positive portion of the flow-volume loop is (inhalation/exhalation)

EXHALATION!

AIP presents with ___ onset of resp. sx's and is characterized by ___ ___ respiratory failure

EXPLOSIVE onset Rapid Progressive failure 33% mortality in hospital

Each alveolus has a thin wall with capillaries lined by ________ cells. This forms part of the air-blood barrier and is called what?

Each alveolus has a thin wall with capillaries lined by ENDOTHELIAL cells. This forms part of the INTERALVEOLAR septum!

What does "combined compliance" mean?

Elastic properties of both the lung and chest wall determine their combined compliance. At the end of expiration (Functional Residual Capacity, FRC), the inward recoil of the lung is balanced by the outward spring of the chest wall.

TEST: A 42 year old male presents to the clinic complaining of increasing dyspnea on exertion. He has a history of smoking 2 packs/day for 25 years and his dyspnea improves very little after using inhalers. He denies an excessive cough, sputum production, wheezing, or chest pain. On physical exam, his chest is BARREL-SHAPED with an increased AP diameter. The breath sounds are diminished and there is a increased expiratory phase on auscultation. CXR shows a flattened diaphragm with an enlarged retrosternal space. What is top on your differential?

Emphysema!

Dextromethorphan can cause ___ so there is potential for ____ Metabolized how?

Euphoria = abuse Metabolized by liver via P450

70% of all patients with CF will have a deletion of the _____ locus on chromosome ___

F508 locus on chromosome 7

TEST: Exacerbating symptoms of asthma, like an increase in the use of _______ or frequent nighttime or daytime symptoms in a patient who was otherwise stable on baseline asthma medication should be treated with what?

Exacerbating symptoms like an increase in the use of SABAs or frequent nighttime or daytime symptoms in a patient who was otherwise stable on baseline asthma meds should be treated with a SHORT COURSE of ORAL CORTICOSTEROIDS

Tiotropium is excreted _______ and is actively (secreted into/reabsorbed from) the urine

Excreted RENALLY - Actively SECRETED into the urine

Rifampin is excreted in the _____ and can possibly be _______ in the ____

Excreted in the BILE and can possibly be RECIRCULATED in the LIVER

What are some of the triggers for nonallergic asthma?

Exercise Cold air Drugs GERD VIRAL INFECTIONS

List the expectorant and cough suppressant discussed

Expectorant: Guafenesin *Cough suppressant:* Dextromethorphan

TEST: During expiration, the diaphragm (relaxes/contracts), the rib cage volume (increases/decreases) and both the ______ _____ pressure and the pressure inside the ______ increase

Expiration: - Diaphragm RELAXES - Rib cage volume DECREASES - Both the PLEURAL CAVITY and pressure inside the LUNGS INCREASE

TEST/AGAIN: The embryological pharynx will extend a ridge that becomes the ______ and the ______, while the pharynx itself will continue as the ________

Extend a ridge that becomes the TRACHEA and the LUNGS (laryngotracheal tube), while the pharynx itself will continue as the ESOPHAGUS

Respiration can be divided into external and internal respiration. What does each exactly mean?

External: Gas exchange between air and blood Internal: Gas exchange between blood and tissues

RECALL: Each branchial arch is covered by _______ externally and lined by _______ internally, with a core formed by _________

Externally: ECTODERM Core: MESENCHYME Internally: ENDODERM

****TRUE or FALSE: Antibiotics are the treatment of choice for asthma

FALSE!

TRUE or FALSE: Most people who develop hypersensitivity penumonitis will suffer irreversible damage to the lungs

FALSE, only a small percentage of people will! Generally, a person must be exposed repeatedly over time before sensitivity and resultant disease develop (causing complement activation, granuloma formation, fibrosis, etc...)

TEST/TRUE or FALSE: Oxygen requires hemoglobin in order to dissolve in blood

FALSE, oxygen CAN actually directly dissolve in plasma, but VERY LIMITED as gases do not dissolve well in aqueous solutions

TRUE or FALSE: Viability of the fetus means survival on its own when it is born

FALSE, since it is a fetus, it still needs INTENSIVE THERAPY!!

TRUE or FALSE: Myoglobin also shows a allosteric curve

FALSE, since myoglobin has a much *higher affinity for O2, it shows a HYPERBOLIC curve*

TRUE or FALSE: Mycoplasmas are penicillin sensitive

FALSE, since they don't have cell walls penicillins have little-no effect on them! (remember, penicillins prevent bacterial wall synthesis)

TRUE or FALSE: Macrophages are capable of producing hypochlorous acid

FALSE, since they lack the myeloperoxidase enzyme

TRUE or FALSE: Just like in asthma, antibiotics are not helpful in COPD

FALSE, studies have shown that antibiotics MAY be beneficial for COPD exacerbations! *unlike asthma*

TEST/TRUE or FALSE: Surfactant decreases lung compliance

FALSE, surfactant decreases surface tension which should INCREASE lung compliance!

TRUE or FALSE: Beclomethasone can be given to patients 12 months of age or older

FALSE, that is Budesonide (Beclomethasone can be given to patients 5 YEARS of age or older)

TRUE or FALSE: The R lung has a cardiac notch

FALSE, the L lung does!

TRUE or FALSE: The physical exam is the most important in diagnosing interstitial lung disease

FALSE, the history is the most important!

TRUE or FALSE: The intrapleural pressure at the end of expiration is more negative than during inspiration

FALSE, the intrapleural pressure is more negative DURING INSPIRATION so air can flow IN to the alveoli - At the end of expiration, intrapleural pressure is still negative but not as much

TRUE or FALSE: The middle lobe of the right lung extends fully to the back

FALSE, the middle lobe does not extend fully to the back!

TRUE or FALSE: The lung goes all the way down to the diaphragm

FALSE, the pleural cavity does however!

TRUE or FALSE: Fick's Law of Diffusion states that the rate of diffusion of a gas is directly proportional to the distance of diffusion

FALSE, the rate of diffusion is INVERSELY proportional to the distance of diffusion (so that the longer the distance, the slower the diffusion)

TRUE or FALSE: Corticosteroids help alleviate asthma symptoms by causing bronchodilation

FALSE, the therapeutic effects are due to its anti-inflammatory properties

TRUE or FALSE: Like fluconazole, Echinocandins penetrate the CNS well

FALSE, they LACK CNS penetrability

TRUE or FALSE: Since bronchioles are very small, they have very high resistance

FALSE, they actually have very LITTLE resistance because there is SO MANY of them in PARALLEL (much like capillaries)

TRUE or FALSE: The pharyngeal constrictor muscles are *smooth muscles*

FALSE, they are SKELETAL muscles as swallowing is a VOLUNTARY action!!

TRUE or FALSE: The antibodies involved in type II and III reactions are IgE

FALSE, they are either IgG or IgM

TRUE or FALSE: The soft palate and uvula are a part of the oropharynx

FALSE, they are parts of the NASOPHARYNX - The oropharynx is from the UVULA to the HYOID bone You can also say that the oropharynx is UNDERNEATH the soft palate

TRUE or FALSE: Azoles are only fungicidal

FALSE, they can also be fungistatic depending on their SYSTEMIC CONCENTRATIONS

TRUE or FALSE: The trachea has 16-20 C-shaped rings of hyaline cartilage that opens anteriorly

FALSE, they open POSTERIORLY to keep the trachea open

TEST: What is the most common form of ACUTE hypersensitivity pneumonitis?

FARMER'S lung

T/F - Corticosteroids treat UIP?

FASE< No therapy is helpful

The forced expiratory flow (FEF) measures the airflow ______ through an exhale, and is a good index of _____ airway disease such as ______ and ________

FEF measures the airflow HALFWAY through an exhale, and is a good index of SMALL airway disease such as ASTHMA and EMPHYSEMA Highly Variable Depends on FVC

RECAP: To determine the severity of an obstructive defect, what value on PFT do you look at?

FEV1

TEST: What PFT index is the best predictor of mortality in CF? Give some of the ranges of this index and the corresponding mortalities

FEV1 > 60% predicted, ALL alive for next 4 years < 35% predicted, 40% will die in next 4 years

A pulmonary function test is necessary for dx and severity of obstruction...what is used to make a COPD dx?

FEV1/FVC FEV1 = Severity

RECALL: What law relates the rate of gas diffusion to various variables? What is the equation for this?

FICK'S LAW of DIFFUSION V'gas = rate of gas diffusion, is proportional to [A x D x (delta P)] / T A = Surface area D = Diffusion constant Delta P = Change in partial pressure from side 1 to side 2 of the membrane T = Thickness of membrane

Diffusing capacity is based on _____ Law, which states what?

FICK'S Law of Diffusion: Amount of gas diffused is proportional to: - (Area x Diffusion constant x Pressure difference before and after medium) / Thickness

TEST: Most TB is curable, but for active TB you must use a ____ or more drug even for the simplest regimen

FOUR or more

TEST: How many stages of COPD are there, and what are they?

FOUR stages: Stage I: Mild (MAJORITY) Stage II: Moderate Stage III: Severe Stage IV: Very severe

A PE is likely if the Well's Score is greater than ____

FOUR!!

What is the equation for calculating FRC using the body plethysmography technique?

FRC (which is V2) = (P4 x deltaV) / (P3 - P4) P4 = mouth pressure after inspiration in box P3 = mouth pressure before inspiration in box Delta V = change in volume inside box before and after inspiration

What is the equation for calculating FRC using the helium dilution method?

FRC (which is V2) = V1 (C1-C2) / C2 V1 = volume of helium in the spirometer C1 = Initial He concentration V2 = FRC C2 = Final He concentration (measured by spirometer)

TEST: Define functional residual capacity (FRC), and how is it different from residual volume (RV)?

FRC = Volume of air remaining in the lungs at the end of a TIDAL volume expiration - Whereas residual volume (RV) is the volume remaining in the lungs just at the end of MAXIMUM expiration *FRC = Expiratory reserve volume (ERV) + RV*

AGAIN: Pulmonary vascular resistance is lowest near the ______ _______ capacity

FUNCTIONAL RESIDUAL CAPACITY

TEST: Amphotericin B is a ________ with a ______ spectrum of activity. What is its MOA?

FUNGICIDAL with a BROAD spectrum of activity - MOA: Binds to ERGOSTEROL in fungal cell membrane, creating pores --> membrane depolarization and cell LEAKAGE

List some of the genetic risk factors for pulmonary emboli formation

Factor V Leiden (MOST COMMON) Protein S, C deficiency Prothrombin G20210A mutation Anticardiolipin Ab syndrome Lupus anticoagulant

True or false... CT scans are not used with IV contrast

False Can be used in conjunction with IV contrast to visualize lumen and the vessels of the heart and lungs

******If FEV1/FVC ratio is (less than/greater than) 80%, and/or FVC is less than ___%, you probably have a restrictive defect

For a RESTRICTIVE defect: - FEV1/FVC GREATER THAN 80% - FVC < 80%

What are the holes in the thyrohyoid membrane for?

For passing of the internal branch of the superior laryngeal N. (coming off of vagus n.)

When are IV antibiotics indicated in treating CF?

For pulmonary exacerbations!

Classically, Farmer's lung is a form of (acute/chronic) hypersensitivity pneumonitis and is more common in the (summer/spring/winter). Why?

Form of ACUTE hypersensitivity pneumonitis, more common in WINTER because the hay is stored in years with excessive rain in the spring/summer

TEST: Laryngeal seromucous glands are found throughout the ________ ______, except at the level of the ____ _____ _____.

Found throughout the LAMINA PROPRIA, EXCEPT at the level of the TRUE VOCAL CORDS

TEST: Within the total lung capacity (TLC), there are four ______ and four _______ that can be calculated. What are they?

Four VOLUMES: 1) Inspiratory reserve volume (IRV) 2) Tidal volume (TV) 3) Expiratory reserve volume (ERV) 4) Residual volume (RV) Four CAPACITIES: 1) Vital Capacity (VC) 2) Inspiratory capacity (IC) 3) Expiratory capacity (EC) 4) Functional residual capacity (FRC)

How can you tell the difference between an INFLATED lung and a pt with EMPHYSEMA?

Free floating alveolar septa.

List the order of macrolides from most to least likely to cause p450 interactions

From MOST to LEAST likely: - Erythro > Clarithro > Azithro

What are the four paranasal sinuses?

Frontal, ethmoidal, sphenoidal, and maxillary sinuses

TEST: Fluconazole is idea for treating fungal _________. Why?

Fungal MENINGITIS, because it penetrates WELL into CSF (and sputum, urine, and saliva)

TEST: After dividing to form the bronchi, further divisions of the laryngotracheal tube will form the _______ until the last division, which will form the _______

Further divisions of the tube will form the BRONCHIOLES until the last division, which will form the ALVEOLI

What genetic defect that can cause CF is a TREATABLE one? What is the caveat to this?

G551D - However, treating this does not REVERSE any damage that has already happened

What occurs at the respiratory zone of the ventilation system?

GAS EXCHANGE!

The region between the two vocal cords is called the ______, and the only time you close this space is when you are (eating/talking). Why?

GLOTTIS Only time it is closed is *when you are TALKING, as this allows the air going through them to cause VIBRATIONS in the cords to produce different sounds*

TEST: What cells are present between the respiratory epithelial cells in the inner lining of the lungs, and what is their primary function in terms of fighting infection?

GOBLET cells - Produces MUCOUS that can TRAP particulate matter entering the lungs (this allows the beating cilia on the epithelial surface to SWEEP and EXPEL the mucous bound particles up and out by coughing)

TEST: 90% of TB infected individuals will lead to _______ formation. How does this ultimately lead to a LATENT infection?

GRANULOMA formation eventually --> CASEUM (cheese-like structure) formation - The environment of the caseum is too hostile for the bacteria, with low pH and low oxygen tension. While some of the bacteria will be killed/inhibited from growing, many can remain dormant while the caseum HEALS by FIBROSIS --> LATENT TB!

TEST: On inspiration, the pressure outside the lungs are (greater/lesser) than the pressure inside the lungs

GREATER - And will move towards being equal with the pressure inside (max inspiration)

****TEST: The lower the P50, the (greater/lower) the affinity for oxygen of that hemoglobin

GREATER the affinity for oxygen

What drug is an expectorant, and what is its MOA?

GUAIFENESIN (Mucinex) - MOA: Improves mucociliary clearance by DECREASING MUCIN release and its VISCOELASTICITY, as well as increasing transport of mucus

The pulmonary artery catheter is placed where?

Goes through the heart To the Pulmonary Artery

Under what conditions will 100% oxygen breathing increase PaO2? (good response) When would there be LIMITED response to breathing 100% oxygen? What is the clinical implication of this?

Good response to 100% O2: Alveolar hypoventilation of V/Q mismatch Limited response: Shunt Therefore, you can use the response to 100% oxygen to DIAGNOSE shunt!

RECAP: Goodpasture's syndrome is the combination of _________ (renal pathology) with ________ hemorrhage in presence of anti-____ antibodies

Goodpasture's: - GLOMERULONEPHRITIS with ALVEOLAR hemorrhage in the presence of anti-GBM antibodies

What are some characteristic symptoms of subacute hypersensitivity pneumonitis?

Gradual development of productive cough, dyspnea, fatigue, anorexia, and weight loss

Bordatella pertussis is a gram (positive/negative) (coccus/bacillus/spiral) bacteria (with/without) a capsule that can be grown in ______-______ media

Gram NEGATIVE COCCOBACILLI bacteria WITH a capsule that can be grown in *BORDET-GENGOU media*

What kind of granules are contained within granulated cells, and when are they released?

Granules include: - Serotonin - Calcitonin - Enkephalins - Neuropeptides They are released in response to HYPOXIC conditions, as these granules eventually lead to BRONCHODILATION - They also induce differentiation of lung epithelia

A histology slide indicating many giant cells and a horseshoe appearance is indicative of? What type of necrosis is present in a granuloma?

Granuloma CASEOUS necrosis

TEST: Granuloma formation requires _____-__, and without it the infection can spread _________ and lead to what type of tuberculosis?

Granuloma formation requires TNF-ALPHA, and without it the infection can spread HEMATOGENOUSLY, leading to MILIARY tuberculosis (distributed across entire lung as small spots, like "millet" seeds)

TEST: What is the effect of gravity on hydrostatic pressure?

Gravity INCREASES hydrostatic pressure and thus the intravascular pressure as you move DOWN, which contributes to why blood flow is the greatest at the base of the lungs

What is the definition of interstitial lung disease?

Group of diseases that causes *disruption of ALVEOLAR STRUCTURES due to lung infiltrates*

H2 receptor activation effects differ from that of H1 in that it causes sustained _________ without any _____________, whereas H1 causes both. What is the mechanism for this? *Covered in GI*

H2 receptor activation --> sustained VASODILATION without any bronchoconstriction Mechanism: H2 receptor activation --> increase in cAMP --> PKA --> phosphorylation of MLCK --> inactivated MLCK (no phosphorylation and activation of myosin, no bronchoconstriction!)

What are some side effects of MEPOLISUMAB?

HA Dizzy Hypersensitivity Herpes Zoster (shingles)

A PA film is done mainly on a ___ patient. The rays of this xray pass from ___ to ____ and there is ___ _____ of the heart with NO widening of the ____ AP patients are___

HEALTHY, Rays pass from POSTERIOR to ANTERIOR There is NO MAGNIFICATION of the heart and no WIDENING of the MEDIASTINUM AP = sick, unable to come to radiology (portable)

Porphryias are genetic disorders producing errors in _____ synthesis

HEME

List two symptoms that would clue you in to suspect Goodpasture's syndrome, and in what kind of patients would you expect to see this in? What are some alleles that make a patient genetically susceptible for this disease?

HEMOPTYSIS (coughing up blood) or HEMATURIA (blood in urine) - Seen in patients who are genetically susceptible and who SMOKE, or are exposed to HYDROCARBONS or get VIRAL RESPIRATORY INFECTIONS Genetically susceptible alleles: HLA-DRw15, -DR4, and -DRB1 alleles

Again: the ___ the compliance is (steeper the slope) then the ___ the lung is to inflate

HIGHER the compliance = EASIER to inflate

TEST: In what kind of patients would an induration of 5 mm and above indicate a latent TB infection?

HIV-infected patients

What physical test can you do to look for a DVT? Is this diagnostic and useful nowadays for looking for a DVT?

HOMAN'S Sign - Forceful and abrupt dorsiflexion of the patient's foot while the knee is extended This is NOT diagnostic and not very useful in diagnosing a DVT, because a positive sign (pain upon doing the test) isn't necessarily indicative of a DVT

Chronic HP has ___ & _____

HONEYCOMBING & Ground GLASS

As COPD progresses, what physical finding could develop that manifests itself as an increase in chest diameter?

HYPERINFLATION of the lungs --> increase in chest diameter

________ _______ is used in CF patients to make them cough

HYPERTONIC SALINE

AGAIN: Asthma involves (hypo/hyper)-active airways that (dilate/constrict) in response to stimuli, causing increased airway ________

HYPERactive airways that CONSTRICT in response to stimuli, causing increased airway RESISTANCE (or obstruction)

What is the phenomenon called that describes why the inflation and deflation curves are not the same on a pressure-volume curve?

HYSTERESIS

TEST: What sign on CXR is suggestive of a PE? Is this a common or an uncommon finding? Why?

Hampton's Hump & Westermark sign This is UNCOMMON!! This is essentially an infarcted lung, but because the lungs have a DUAL blood supply, it is uncommon for the lung to be infarcted!

Because hemeglobin can attach to sugars, we need to check the _____.

HbA1c

A ____ is the presence of blood in the plural space

Hemothorax

The change from VALINE to GLUTAMIC ACID would create a ___ __ or "__ ___ __" anemia

Hemoglobin S or Sickle Cell disease

Hemoglobin C is ___ ____ specific anemia. This a SINGLE RESIDUE ____ This changes from ___ to ___

Hemolytic anemia Single residue SUBSTITUTION changing LYSINE to GLUTAMIC ACID (base to acid AA)

TEST: What is the difference between hemolytic anemia and sickle cell anemia in terms of the mutation and the resulting abnormality?

Hemolytic: Hemoglobin C, an abnormal BETA GLOBIN where glutamic acid is swapped with a LYSINE at position 6 --> INFLEXIBLE HbC --> Shortened lifespan Sickle: Hemoglobin S, linkages between Hb which forces RBCs into a SICKLING form - Glutamic acid at position 6 is swapped with VALINE

What is the definition of asthma?

Heterogenous disease Characterized by CHRONIC airway inflammation

Answer to Case 1

High resolution chest CT

TEST: What kind of people belong to a high risk TB group? What induration limit indicates a latent TB infection?

High risk TB group includes people who work with TB bacteria, nurses/doctors at TB clinic, and household contacts of TB patients Limit: >10 mm induration = LTB1

Histamine H1 receptor activation causes dilation of _______ and ________ but contracts _____ and (small/large) arteries. It also activates _______, which promotes transcription of pro-inflammatory cytokines

Histamine H1 receptor activation causes dilation of ARTERIOLES and VENULES (post-capillary) but contracts VEINS and LARGE ARTERIES It also activates NF-kB (promotes pro-inflammatory cytokine transcription)

RECALL: What fungal infection do you associate with caving?

Histoplasmosis

TEST: Some of the most common helpful "clues" to trigger you to think hypersensitivity pneumonitis are what?

History of WATER damage Use of HOT TUB, SAUNA, SWIMMING POOL

Where is the minor fissure? What does it separate?

Horizontal ....On the Right Lung Separates the R upper from R middle lobe

TEST: Diffusing capacity (DL) tells us what about gas diffusion?

How EASILY a gas diffuses across the alveolar-capillary membrane

What is one example discussed in class of a type III hypersensitivity reaction? What is this most commonly caused by?

Hypersensitivity pneumonitis (extrinsic allergic alveolitis) - A type of inflammation in and around the alveoli and bronchioles of the lung (essentially an allergic reaction in the lungs) Most common cause: Allergic reaction to INHALED organic DUSTS or chemicals (which contain microorganisms)

In chronic bronchitis, there is hypertrophy of the ______ glands as indicated by the ____ index, which is the thickness of the _____ glands in relation to the thickness of the ____

Hypertrophy of the SUBMUCOSAL glands as indicated by the REID index, which is the thickness of the MUCOUS glands to the thickness of the WALL (bronchial)

What are two possible side effects of pseudoephedrine?

Hypotension and pounding HR

TEST: If the A-a gradient turns out normal, what are the two possible causes of hypoxemia?

Hypoventilation (increased PaCO2) Low PiO2 low inspired O2) - (extreme high altitude)

TEST: How is amphotericin B given?

IV infusion, because it is NOT ABSORBED ORALLY!

What modality can be a prophylaxis AGAINST recurrent PEs?

IVC filter!

What is one of the most severe forms of idiopathic interstitial pneumonia?

Idiopathic pulmonary FIBROSIS

TEST: List the effect on alveolar and arterial PO2 and PCO2 if the V/Q ratio is < 1

If V/Q is < 1, this means you are PERFUSING much more than you are VENTILATING. This means: - Arterial and alveolar PO2 is LESS than normal (ARTERIAL HYPOXEMIA) - Arterial and alveolar PCO2 will be MORE than normal (not getting out of blood as much, and the CO2 that does diffuse into the alveoli won't be expelled as much because of low ventilation)

TEST: List the effect on alveolar and arterial PO2 and PCO2 if the V/Q ratio is > 1

If V/Q is >1, this means you are VENTILATING much more than you are PERFUSING. This means: - Alveolar and arterial pO2 is much HIGHER than normal - Alveolar and arterial pCO2 is ZERO (high rates of ventilation means high rates of CO2 diffusing out of the blood and being expelled)

Which of the three things in the epithelial lining fluid of the inner lung binds to the cell surface of bacteria and *prevents them from colonizing the respiratory epithelial cells?*

IgA antibodies!

List some of the non-genetic risk factors for a pulmonary embolus

Immobilization Women taking birth control pills Estrogen therapy Pregnancy A-fib or CHF Major trauma or burns

TEST: List some of the mainstay pharmacological agents used to treat ILD (two main classes of agents)

Immunosuppressive agents: - Azathioprine - Cytoxan - Cyclosporan Corticosteroids

Obstructive lung diseases impairs the ability of air to (leave/enter) the alveoli during (expiration/inspiration), thus ______ the air

Impairs the ability of air to LEAVE the alveoli during EXPIRATION (air TRAPPING)

List the relative percentages and partial pressures of the following gases in DRY air at sea level: - N2 - O2 - CO2

In DRY air at sea level: Patm = 760 mm Hg P(N2) = 760 x 0.79 = 600 mm Hg P(O2) = 760 x 0.21 = 160 mm Hg P(CO2) = 760 x 0.0003 = 0.23 mm Hg Air is only 0.03% carbon dioxide

TEST: How do you calculate the partial pressure of a gas in DRY air?

In DRY air: - Pgas = %gas x Patm

AGAIN: Under what conditions would you see an ACTIVE mechanism of regulating pulmonary vascular resistance?

In HYPOXIA states: - Alveolar hypoxia --> pulmonary small artery VASOCONSTRICTION --> redirecting blood flow away from poorly ventilated areas to areas where it is well ventilated so you can increase the amount of oxygen getting into the blood!

List the relative percentages and partial pressures of the following gases in INSPIRED air at sea level: - N2 - O2 - CO2

In INSPIRED air at sea level: Patm = 760 mm Hg and Pwatervapor = 47 mm Hg P(N2) = (760 - 47) x 0.79 = 563 mm Hg P(O2) = (760 - 47) x 0.21 = 150 mm Hg P(CO2) = (760 - 47) x 0.0003 = 0.21 mm Hg Overall, since Ptotal is less due to subtracting the partial pressure of water vapor, the partial pressures of each gas is DECREASED relative to that in DRY air

TEST: How do you calculate the partial pressure of a gas in INSPIRED air?

In INSPIRED air: - Pgas = %gas x (Patm - Pwater vapor) The only difference between this equation and the one in dry air is you are *now reducing the total pressure by the pressure of water vapor, because INSPIRED AIR IS RAPIDLY SATURATED WITH WATER VAPOR*

What kind of airways do turbulent flow happen the most?

In LARGER airways (nose, mouth, trachea) where there are HIGHER FLOW RATES

TEST: In the remaining 10% of tuberculosis infections, the patients will develop an active disease by also forming a caseating granuloma. How is this caseum different from that produced in a latent TB infection?

In an active TB infection, the caseating granuloma undergoes LIQUEFACTION, which has a composition similar to broth. This provides an extremely favorable environment for mycobacterial GROWTH and MULTIPLICATION --> CAVITY formation, which is very CONTAGIOUS

In emphysema, loss of pulmonary ______ causes a loss of ______ _____, so that when the patient breathes out the airways will ______

In emphysema, loss of pulmonary PARENCHYMA causes a loss of ELASTIC RECOIL, so that when the patient breathes out the airways will COLLAPSE

TEST/TEST: In general, LONG-acting beta2 agonists can increase the risk of what? In what situation would use of LABAs be contraindicated?

In general, LABAs can increase risk of ASTHMA EPISODES or ASTHMA RELATED DEATH - Due to this risk, LABAs by ITSELF without a long-term asthma control med is CONTRAINDICATED

TEST: In normal breathing, the rib cage (is/is not) expanding so on inhalation, the diaphragm (domes/flattens)

In normal breathing, the rib cage IS *expanding on inhalation, so the diaphragm FLATTENS*

TEST: In type I hypersensitivity reactions, exposure to allergens for the first time results in IgE production by ___ cells, which then binds to CD___ on mast cells. Exposure to this allergen a second time will bind the IgE antibodies on the surface of the mast cells and cause ________, releasing ______ and ______

In type I hypersensitivity reactions, exposure to allergens for the first time results in IgE production by B cells, which then binds to CD23 on mast cells. Exposure to this allergen a second time will bind the IgE antibodies on the surface of the mast cells and cause DEGRANULATION, releasing HISTAMINES and HEPARIN

TEST: As you increase the dose of beta2 agonists, they lose their ________ which increases their risk of ________ ____ effects. List some of the main ones.

Increase dose --> Lose their SELECTIVITY, which increases risk of SYSTEMIC SIDE EFFECTS: - HYPOKALEMIA (drives K into muscle cells) - HYPERGLYCEMIA - TREMORS (common!) - Nervousness/anxiety - Headache - Muscle cramps (with certain ones) - Palpitations - Tachycardia (common!) /changes in BP

TEST: What index do you look at to differentiate between asthma and emphysema? What is the main difference in this index?

Index: DLCO (diffusion capacity of the lung) Asthma: Normal Emphysema: Decreased

Pneumonia and TB are categorized under?

Infectious lung disease

TEST: LABAs are often used with what long term anti-asthmatic? Why?

Inhaled GLUCOCORTICOIDS - These can UPregulate beta2 adrenergic receptors, which can maintain/increase the response to the LABAs and allow better symptom control

Inhaled antibiotics is primarily a(n) (acute/maintenance) therapy for CF, and it has been shown to improve ______ and reduce the likelihood of __________

Inhaled antibiotics is primarily a MAINTENANCE therapy for CF, and it has been shown to improve FEV1 and reduce the likelihood of HOSPITALIZATIONS

TEST: What is the MOA of respiratory fluoroquinolones?

Inhibiting DNA gyrase of gram NEGATIVE rods (concentration-dependent)

TEST: What is the MOA of the azole antifungals?

Inhibits 14-alpha-sterol demthylase, which: - Impairs the biosynthesis of ERGOSTEROL for the cytoplasmic membrane, leading to ACCUMULATION of 14-ALPHA-METHYLSTEROLS - And these methylsterols disrupt and impair the functions of certain membrane-bound enzyme systems of the fungi --> inhibiting their GROWTH! (especially under aerobic conditions)

Zafirlukast inhibits CYP___, which metabolizes this major anticoagulant

Inhibits CYP2C9, which metabolizes WARFARIN (can cause warfarin to be extra effective)

TEST: MOA of Isoniazid Is it bactericidal or bacteriostatic?

Inhibits MYCOLIC ACID SYNTHESIS, thus inhibiting cell wall synthesis BACTERICIDAL

What is the MOA of vancomycin?

Inhibits cell wall synthesis by high affinity binding to the D-ALANYL-D-ALANINE terminus of cell wall precursors

TEST: What is the MOA of echinocandins?

Inhibits fungal CELL WALL SYNTHESIS - Blocks synthesis of a polysaccharide component of the cell wall (beta-(1,3)-D-glucan) --> CELL DEATH

TEST: MOA of Linezolid

Inhibits protein synthesis by binding to the P site of the 50S ribosomal subunit

RECALL: What does Coumadin do?

Inhibits vitamin K clotting factors (II, VII, IX, X, and protein C)

Briefly explain how a PPD test works, and what indicates a previous TB infection

Inject mycobacterial antigens in the skin of the forearm, which is presented by DCs and macrophages to MEMORY T cells. - If there were a previous TB infection, then the *memory T cells will proliferate and release CYTOKINES that cause an INDURATION (swelling).* This occurs after 48h-72h

What are some chemical fumes and vapors that can cause pneumoconiosis?

Insecticides, NO, sulfur dioxide, ammonia, and benzene gas

Inspiration is a(n) (active/passive) process, while expiration is a(n) (active/passive) process

Inspiration: ACTIVE Expiration: PASSIVE (at REST, *not during exercise or forced*)

RECAP: Describe the progression of alveolar pressure through one cycle of respiration

Inspiration: Alveolar pressure slightly DECREASES as you're breathing in, as you reach max inhalation and can't breath in anymore the alveolar pressure starts increasing Expiration: Alveolar pressure continues to increase until you start running out of breath to exhale, in which it starts decreasing

The ________ is the most important muscle of inspiration, while the _______ muscles are the most important muscles of expiration

Inspiration: DIAPHRAGM Expiration: ABDOMINAL muscles

Vital capacity is the sum of what three terms?

Inspiratory reserve volume (IRV) Tidal volume (TV) Expiratory reserve volume (ERV) *VC= IVR+TV+ERV*

Describe what a typical alveolar duct looks like

Interrupted wall with typical smooth muscle KNOBS that are bulging into the lumen and interpose between alveoli

TEST: Intrathoracic pressure refers to pressure within the ______ _____, which normally is slightly (more/less) than atmospheric pressure. This is what is known as _______ pressure and is what allows the lungs to (inflate/deflate)

Intrathoracic pressure refers to pressure within the PLEURAL CAVITY (also known as intraPLEURAL pressure) - This is normally slightly LESS than atmospheric pressure, aka NEGATIVE pressure, and is what allows the lungs to INFLATE (air goes into the lung down gradient)

What is responsible for tension in vocal folds for opening and closing the glottis?

Intrinsic Laryngeal muscles The EXTRINSIC mm. help move the larynx during DEGLUTION

TEST: What is the pathogenesis of emphysema in terms of what causes the destruction of alveolar walls and their elastic fibers?

Irritant such as cigarette smoke --> influx of inflammatory cells in alveoli (LEUKOCYTES) --> PROTEOLYTIC ENZYMES and ELASTASES from leukocytes are released --> alveolar wall damage and loss of elastin fibers

TEST: List what the most common EKG finding is on PE, and list what finding on EKG will indicate a massive PE

It is usually NORMAL - However, you could see an S1Q3T3 and a right axis deviation, which will indicate a MASSIVE PE

TEST: From a ventral view of the embryological pharynx, what structure is forming as a ridge (looks like a groove from the dorsal side) and what does this ridge continue as? Is this structure and its continuations ectoderm, mesoderm, or endoderm?

LARYNGOTRACHEAL ridge - Continues to DIVIDE into the PRIMARY BRONCHI (lung buds)!!! This will then continue to divide to form the rest of the bronchial tree These structures are ENDODERM!!!

TEST: After Bordatella bacteria attach to ciliated epithelial cells, it produces _____ toxin that induce ___-_______ of the ________ subunit of G-proteins, leading to the activation of _______ _______. This causes a build-up of ____, which then stimulates _____-dependent protein kinase. What are the two main effects of this?

It produces PERTUSSIS toxin that induce ADP-RIBOSYLATION of the INHIBITORY subunit of the G-proteins --> activation of ADENYL CYCLASE This causes a build-up of cAMP, which then stimulates cAMP-dependent protein kinase. This induces: 1) More mucous production in the lungs 2) Lymphocytosis (inhibits signal transduction by chemokine receptors, trapping lymphocytes in circulation)

What is the limitation to an unfractionated heparin administration?

It requires CONTINUOUS IV INFUSION

TEST: What is the black box warning with Itraconazole?

Known to cause negative INOTROPIC effects, leading to possible serious CV events such as TORSADES DE POINTE - If used with CYP450 inhibitors, which will significantly increase the amount of itraconazole

With which amphotericin B formulation can the highest systemic levels be achieved? This formulation produces the (most/least) infusion-related reactions

L-AMB - Produces the LEAST infusion-related reactions!! (fever, chills, hypoxia)

TEST: What characteristic feature distinguishes mycoplasmas from other bacteria?

LACK of a well-defined cell wall! Mycoplasma Pneumoniae

TEST: What is the late stage of asthma due to, and what are the subsequent effects?

LATE stage due to *ENZYMES/CYTOKINES released by eosinophils and neutrophils*, which were attracted in the early phase. Neutrophils release PROTEASES Eosinophils release MAJOR BASIC PROTEIN Main end effect: MORPHOLOGIC CHANGES (structural changes) to lung tissue, also: - More bronchoconstriction and edema - Mucus secretion - Airway hypersensitivity

AGAIN: The more compliant the lungs are, the (more/less) the alveolar elastic recoil

LESS (alveolar elastic recoil is OPPOSITE of lung compliance) - If the lungs could distend more and increase in volume, it would make sense that the elastic recoil of the alveoli, which wants to collapse, would be less so it can allow the lungs to stay inflated!

TEST: Ipratropium has (more/less) systemic side effects than atropine. Why?

LESS systemic side effects than atropine, as Ipratropium is POORLY ABSORBED from the respiratory epithelial surface (since it is highly POLAR) It also doesn't impede mucus clearance form the lungs (unlike atropine)

Sensitivity to (light/sound) is a portential adverse reaction of doxycycline

LIGHT (photosensitivity)

TEST: What is the treatment for choice for patients with acute nonmassive PE?

LMW Heparin!

TEST: What is the treatment of choice for outpatient DVT in conjunction with Coumadin?

LMW Heparin!

The treatment period for TB is (short/long). Why?

LONG, because M. tb grows very slowly and you must catch the organism in the process of reproducing

Patients with obstructive diseases often require a (shorter/longer) period of time for expiration

LONGER (may take more than 3 seconds to forcefully exhale)

A patient with a restrictive defect will generally have a (high/low) FEV1, and why?

LOW FEV1, because restrictive defects DECREASE ALL LUNG VOLUMES, so the forced expiratory volume will be lower because the overall lung volume is lower!

The incidence of infertility is (higher/lower) in women than in men

LOWER in women! - Women can still get pregnant even if they have CF (70-85%)

Bronchiectasis primarily involves the (upper/lower) lobes

LOWER lobes

TEST: Asbestos-related diseases predominantly affect the (upper/mid/lower) lobes

LOWER!

TEST: Stage 3 sarcoidosis presents with what common CXR finding?

LUNG INVOLVEMENT WITHOUT bilateral hilar LAD

The laryngopharynx spans from the inferior aspect of the ______ to the part where it diverges into the respiratory and digestive pathways. The laryngopharynx continues as the ________ posteriorly and the _______ anteriorly

Laryngopharynx spans form the inferior aspect of the EPIGLOTTIS to the diverging point of respiratory/digestive pathways The laryngopharynx continues as the ESOPHAGUS (digestive) posteriorly and the LARYNX (respiratory) anteriorly

The _____ is a passway between oropharynx and trachea.

Larynx

TEST: Surface tension can be described by using the Law of _______, which states what?

Law of LaPlace: - P(alveoli) = (2 x surface tension) / alveoli radius - P = (2 x T) / R As *surface tension increases, so does the pressure in the alveoli!* - And as the alveoli get SMALLER, the pressure in the alveoli increases! (assuming tension to be constant)

Which lungs are inferior?

Left lungs are inferior

Which side is this?

Left side Notice air in GI tract

The lingula is located around the __ ___

Left ventricle

TEST: Legionella pneumophila is a gram (positive/negative) bacteria that is associated with _____ sources. It causes _______ ________ and can be grown in ______ _____ media supplemented with _____ and ______

Legionella pneumophila is a gram NEGATIVE bacteria that is associated with WATER sources (AC, cooling towers). It causes ATYPICAL PNEUMONIA and can be grown in CHARCOAL YEAST media supplemented with IRON and CYSTEINE

TEST: What are some limiting factors for the rate of O2 movement into tissues, and how is this optimized for oxygen delivery in active tissues? (what do you want to maximize, and what do you want to minimize?)

Limiting factors include: - Surface area of tissue - Thickness of tissue - Diffusion constant of the gas - Partial pressure difference To optimize oxygen deliver: - MAXIMIZE diffusion area - MINIMIZE diffusion thickness (distance)

The nasal cavity is mostly lined by _____ epithelium (___ ____) however a small portion of the superior conchae and the roof of the cavity are lined by ___ ____

Lined by RESPIRATORY EPITHELIUM (pseudostratified columnar) olfactory epithelium

What is one mechanism of ACTIVE regulation of pulmonary vascular resistance?

Local ALVEOLAR hypoxia (reducing alveolar PO2) - Causes VASOCONSTRICTION in hypoxic region, which will divert blood away from poorly ventilated alveoli to those alveoli which are better ventilated

Appropriate asthma management requires the proper use of ___ ____ Control and __ ___ medications

Long term control and quick-relief medications

What are you looking for on a V/Q scan? Is this test more sensitive or specific than a CT angiogram?

Looking for PERFUSION DEFECTS due to an OCCLUDED vessel (from a PE) This test is more SPECIFIC than a CT angiogram

As bronchi finally divide into terminal bronchioles, what is the major change that allows bronchioles to change their diameter?

Loss of cartilage plates --> progressive increase in ELASTIC FIBERS and SMOOTH MUSCLE that wraps around the bronchioles (so when they contract, they change the diameter of the airways!)

TEST: Emphysema is a disease process characterized by the loss of pulmonary ________ and _______ of terminal airways

Loss of pulmonary PARENCHYMA (loss of alveolar septae and airway walls) and DILATION of terminal airways

Coarse crackles are ___ and ___ pitched. These have a timing that ___ ___ What diseases are indicatory of this type of sound?

Louder LOWER pitch Last longer (early inspiratory) Disease: Brochiectasis/chronic bronchitis

What is the treatment of choice for STEP 2 asthma management?

Low dose INHALED corticosteroid + as needed SABA

Step 3 management for asthma is?

Low dose Inhaled CS / LABA with a SABA as needed.

If all pharmacological treatments fail for ILD, what is the treatment?

Lung TRANSPLANT

ILD specific to premenopausal females is ______

Lymphangioleiomatosis LAM

A chylothorax is a type of pleural effusion which is ___ ___ in the plural space secondary to leakage from the ___ ___

Lymphatic fluid Thoracic duct

Tiotropium is selective for which muscarinic receptors?

M1 and M3

Recall: What can worsen CF lung disease?

MALNUTRITION

TEST: Where is the major site of airway resistance?

MEDIUM-SIZED BRONCHI!

TEST: DCs process antigens and present them along with what two types of molecules to T cells? Which molecule is recognized by CD4 T cells, and which molecule is recognized by CD8 T cells?

MHC class I or II MHC class I is recognized by CD8 T cells MHC class II is recognized by CD4 T cells

O2 diffusion distance must be (maximized/minimized) in active tissues. Why?

MINIMIZED, because active tissues *need oxygen readily* and the *further the capillaries are from the mitochondria, the longer it will take for O2 to reach them!* 8 microns is the RATE LIMITING STEP between RBC and

CT scans are best used for evaluation of ____ for staging and ___ ____

Malignancies Surgical intervention

TEST: Define inspiratory capacity (IC)

Maximum amount of air that can be *INHALED from the end of a tidal volume*

TEST: What is the mean pulmonary arterial pressure at the level of the heart? Compare this to the mean systemic arterial pressure

Mean pulmonary arterial pressure: ~15 mmHg (typically around 25/8) Mean systemic arterial pressure: ~100 mmHg (typically around 120/80)

Define forced vital capacity (FVC)

Measures the amount of air you can exhale with force as you inhale as deeply as possible

Define forced expiratory volume (FEV_)

Measures the amount of air you can exhale with force from FULL INHALATION in a given amount of TIME (1 second, 2 seconds, or 3 seconds) - Corresponds to FEV1, FEV2, and FEV3

Near the mediastinum, the pleura sits next to the pericardium, what is this pleura called? What is the pleura called near the diaphragm?

Mediastinal parietal pleura Diaphragmatic parietal pleura

UIP is usually found more in ___ (M/F) and around age ___ ___% have a history of smoking Type of cough? Located where usually?

Men>Women Age 50 75% smoked NONPRODUCTIVE COUGH *UPI = LOWER LOBES*

AGAIN: Haemophilus influenzae is the leading cause of what?

Meningitis in children!

A humanized Interleukin-5 antagonist monoclonal antibody called ____ was produced by recombinant DNA technology in Chinese hamster ovary cells. This drug is used for what type of asthma attack? MOA:

Mepolisumab Used for severe asthma attacks *Reducing the level of blood EOSINOPHILS that are activated by IL-5* Given q4w

Increasing the exposure to asbestos will increase the incidence of asbestos related diseases, except for what?

Mesothelioma (the risk for developing this remains constant)

Because Zileuton is metabolized by (one/many) CYP enzymes, it can lead to _____________

Metabolized by MANY CYP enzymes, leading to HEPATOTOXICITY (many reactive intermediates)

This ion, when nebulized, can enhance bronchodilation. List two possible mechanisms for how this works.

Mg Mechanisms (possible): 1) Activates adenylyl cyclase and decrease Ca uptake by competing for binding sites (subsequently inactivates MLCK) 2) Promotes Ca uptake by SR Both prevent smooth muscle contraction, which allows for bronchodilation!

TEST: List the rib level that the LUNG and the PARIETAL PLEURA crosses at each of the following lines: 1) Midclavicular line 2) Midaxillary line 3) Paravertebral line

Midclavicular: - Lung = Rib 6 - Parietal pleura = Rib 8 Midaxillary - Lung = Rib 8 - Parietal pleura = Rib 10 Paravertebral: - Lung = Rib 10 - Parietal pleura = Rib 11

TEST: Early stages of asthma could present with mild hypoxemia and respiratory (acidosis/alkalosis). Why? As the severity of airflow obstruction increases, pCO2 can (increase/decrease)!!

Mild hypoxemia and respiratory ALKALOSIS, because the patient is BREATHING RAPIDLY in the beginning (expelling too much CO2) As the severity of airflow obstruction increases, pCO2 can INCREASE!! (causing hypercapnia and respiratory failure!!)

What is the FEV1% limit for: - Mild asthma - Moderate asthma - Severe asthma

Mild: FEV > 80% predicted Moderate: 80% > FEV1 > 60% Severe: FEV1 < 60% predicted

The minor fissure is also called the ____ and separates the ___ from the ___ lobe The major fissure indicates the ____

Minor = Horizontal Upper from Middle lobe Major = Oblique fissure Middle from Lower lobe

What makes up a granuloma?

Monocytes, macrophages, giant cells, and epithelioid cells

Which wheeze sounds like a BARKING SEAL?

Monomorphic - STRIDOR Heard over the trachea Upper Airway is <5mm

What is the most common radiographic pattern on CXR of ILD? What is a LATE pattern on CXR of ILD?

Most common CXR pattern: RETICULAR or a RETICULONODULAR pattern *Late CXR pattern*: HONEY-COMBING

TEST: Of all the symptoms, what is the only consistent finding after PE? What are the most COMMON symptoms?

Most consistent: TACHYPNEA Most common: Dyspnea (SOB) and chest pain

You would expect obstructive diseases to have a much greater (inspiratory/expiratory) phase on a flow-volume loop

Much greater EXPIRATORY phase, since with obstructive diseases patients have trouble getting air OUT

What are some of the main side effects of beta 2 agonists as bronchodilators?

Muscle TREMORS Tachycardia Hypokalemia Restlessness

Mycoplasmas have a cell (wall/membrane) rich in cholesterol, so in order to grow they need media that are supplemented with _____. Although the bacteria can assume any ______, these bacteria have a characteristic _____ ____ morphology under microscope

Mycoplasmas have a cell MEMBRANE rich in cholesterol, so they need media supplemented with LIPIDS to survive/grow Although the bacteria can assume any SHAPE, these bacteria have a characteristic FRIED-EGG morphology under microscope

Myeloperoxidase is an enzyme that catalyzes the formation of _______ ____, an antimicrobial agent that inhibits the ______ of intracellular pathogens, from ______ _______ and _____ ions.

Myeloperoxidase catalyzes the formation of *HYPOCHLOROUS ACID, which inhibits GROWTH of intracellular pathogens, from HYDROGEN PEROXIDE and CHLORIDE ions* Moreover, hydrogen peroxide + hypochlorous acid creates SINGLET OXYGEN species

AGAIN: Myoglobin acts as cellular oxygen _______ in metabolically active tissues because they have a (high/low) affinity, or a (high/low) P50

Myoglobin acts as a cellular oxygen RESERVOIR in metabolically active tissues because they have a HIGH affinity, or a LOW P50

Which mucolytic reduces mucus viscosity by disrupting disulfide bonds within the mucus? This will case ______ mucin monomers

N-acetyl cysteine DEPOLYMERIZES mucin monomers

List the two mucolytics discussed

N-acetyl cysteine DNAase therapy

TEST: Anticholinergics must be administered with caution if patients have ______-_____ _______. Why?

NARROW-ANGLE GLAUCOMA (caution) Cholinergic agonists induce contraction of ciliary muscle, which LOWERS the intra-ocular pressure. With anticholinergics, it will prevent contraction and lead to INCREASED intraocular pressure, which can potentiate the effects of glaucoma

Egophony is a ___ quality sound similar to a ___ bleating. Due to an enhanced transmission of ___ ____ noise across fluid. How is this performed?

NASAL quality similar to a GOAT bleat Enhanced transmission of HIGH FREQUENCY Performed: Pt says EEEEEE while auscultating - abnormal goes E -> A

O2 binds to Hb BEST in what state?

NATIVE Hb state. Temp = 37 deg. pH = 4

AGAIN: At rest, there is (positive/negative) intrapleural pressure, allowing _______ of the lungs

NEGATIVE intrapleural pressure, allowing EXPANSION of the lungs

The endocrine cells in the respiratory epithelium of the nasal cavity aggregate into _________ bodies

NEUROEPITHELIAL bodies

What are the three main sections of the AIR CONDUCTING portion?

Nasal Vestibule Respiratory Region Olfactory Region

What are the main parts of the Air conductingportion of the respiratory system?

Nasal cavities, sinuses, and the PHARYNX (naso, oro, laryngo- pharynx)

the ____ ___ ____ is composed of two chambers, separated by a bony cartilaginous ___.

Nasal cavity proper Septum

TEST: What are the three main subdivisions of the pharynx?

Nasopharynx Oropharynx Laryngopharynx

Flip for case

Next card has answer

List some of the effects of nicotine in the pathogenesis of emphysema

Nicotine: - is a chemoattractant for neutrophils, which release damaging proteases - Inactivates antiproteases - Produces ROS (reactive oxygen species), which depletes antioxidants

TEST: The conducting zone has no _______ and is named the _______ _____ space

No ALVEOLI, and is named the ANATOMIC DEAD space

The _____ fissure separates the R Middle/Lower lobes

OBLIQUE

The left lung has a (horizontal/oblique) fissure

OBLIQUE

The ____ fissure of the R lung travels from the ___th rib to the ___ th rib. It defines the ___part of the lung

OBLIQUE 4th to the 6th Middle lobe of the lung

Case 1: FEV1/FVC is 62% and the FEV1 is 65%. Does this point to a possible obstructive, restrictive, or normal lung function?

OBSTRUCTIVE

RECAP: If the TLC is greater than 120%, what defect do you have?

OBSTRUCTIVE

Pneumoconiosis is a form of _________ lung disease (not obstructive or restrictive), but only a small % of those exposed to dusts will develop it

OCCUPATIONAL

TEST: What is a monoclonal Ab used to treat asthma symptoms, and what is its MOA?

OMALIZUMAB (given SC) These are monoclonal antibodies (IgG) for the Fc region of *IgE, which prevents it from binding to the Fc epsilon receptor on the surface of mast cells.* This PREVENTS DEGRANULATION by mast cells

Crackles signify the ___ of ___ airways or alveoli that have been collapsed or decreased in volume secondary to 1- 2- 3-

OPENING of the SMALL airways Secondary to: Fluid (CHF, pleural effusion) Inflammatory exudate Poor aeration

RECALL: What layer in the cell wall of gram-negative bacteria is absent in gram-positive bacteria?

OUTER MEMBRANE - Gram negative bacterial cell wall consists of an outer membrane and a THIN peptidoglycan layer. This is absent in gram positive bacteria because their peptidoglycan layer is THICK

The duration of the antihistamines effects often ___ their presence in the blood

OUTLAST

TEST: Which long term treatment therapy has actually been shown to reduce the mortality of COPD?

OXYGEN THERAPY!! *IF HYPOXIC*'?

List some of the reasons for a DECREASED DLCO.

Obstructive defects Parenchymal lung disease Pulmonary vascular disease ANEMIA

TEST: In general, FEV1 is (increased/decreased) in obstructive disease but (increased/decreased) in restrictive disease

Obstructive: DECREASED (cannot forcefully exhale as much in 1 second) Restrictive: NORMAL or INCREASED

TEST: In obstructive diseases, the ________ volume is greatly increased. In restrictive diseases, the peak _______ flow and total lung ________ are decreased.

Obstructive: RESIDUAL volume is greatly INCREASED (after expiration, because you have trouble getting air OUT) Restrictive: Peak EXPIRATORY flow and total lung CAPACITY are decreased - Restricts lung expansion (problem getting air IN)

TEST: One of the proposed explanations for why patients with asthma have a hypersensitive airway is due to an overactive _______ ____ _____ kinase (enzyme) on smooth muscles. Briefly describe what this enzyme does NORMALLY.

Overactive MYOSIN LIGHT CHAIN KINASE (MLCK) Normal: Phosphorylates MYOSIN, which will allow myosin crossbridge formation with actin --> Smooth muscle CONTRACTION

RECALL: With restrictive defects, there is an overall decrease in lung _______. Therefore, the FVC should be (</>) 80% and the FEV1/FVC ratio should be (</>) 88%

Overall decrease in lung VOLUME - FVC < 80% (Can't force out much if the lung can't hold as much to begin with) - FEV1/FVC > 88% (high because both FEV1 and FVC decrease, keeping the ratio close to 1)

TEST: In the oxygen (dependent/independent) pathway of microbe control, there are lysosomal enzymes and antimicrobial peptides that kill and digest the ingested pathogen. List at least THREE of these enzymes/peptides and what each does.

Oxygen INDEPENDENT pathway utilizes lysosomal enzymes and other antimicrobial peptides, including: - LYSOZYME (degrades peptidoglycan) - LACTOFERRIN (sequesters iron) - DNA/RNA-ases

In a healthy patient, what is the rule with xray?

PA film WITH A LATERAL VIEW ALWAYS order 2 films, PA and LATERAL

Structures

PA view Posterior are more easily viewed Anterior = seen more laterally

TEST: What is the alveolar air/gas equation?

PAO2 = PIO2 - (PACO2/R) If R = 0.8, then: PAO2 = PIO2 - 1.25(PACO2)

TEST: List the relationship of alveolar ventilation rate and: - PAO2 (alveolar) - PACO2 (alveolar)

PAO2 is DIRECTLY proportional to VA' (alveolar vent rate) PACO2 is INVERSELY proportional to VA' (alveolar vent rate)

Laminar flow is _______ streams of flow in the (small/large) airways, with the velocity at the (edge/center) twice as fast than at the (edge/center)

PARALLEL streams of flow in the SMALL airways, with the velocity at the CENTER twice as fast than at the edges (due to friction at the edges but not in the center)

Amantadine is also used to treat what CNS disorder?

PARKINSON'S Disease

Expiration at rest is ___

PASSIVE

Like NK cells, cytotoxic T cells also produce these two granules that can kill the target cells

PERFORIN and GRANULYSIN

TEST: A (phosphorylated/dephosphorylated) myosin light chain leads to muscle contraction

PHOSPHORYLATED

Diaphragm pain carried by the _____ nerve can be referred to what part of the body?

PHRENIC nerve can be referred to the SHOULDER

Surface area and diffusion distance are examples of (physical/chemical) properties of respiratory/circulatory anatomy

PHYSICAL properties

TEST: Emphysema patients are often referred to as _____ ________ because they are often __________

PINK PUFFERS because they are often HYPERVENTILATING

TEST: List some of the functions of the paranasal sinuses

Passageway for air to CLEAN, WARM, and HUMIDIFY Resonating chambers for speech Lightens the bones of the skull

How are expiratory flows measured?

Patient breathes to total lung capacity and forcefully exhales to residual volume

In regard to age, Typically, patients are ___ ___ presenting with a progressive ___ and nonproductive cough

Patients are usually MIDDLE AGED with progressive DYSPNEA

TEST: Patients with how severe of asthma would benefit from inhaled corticosteroids?

Patients with PERSISTENT asthma

TEST: Patients with asthma usually have (high/low) pCO2 that could progress to _________, which has a (high/low) pCO2 and could be indicative of what?

Patients with asthma usually have LOW pCO2 that could progress to HYPERCAPNIA, which has a HIGH pCO2 and could be indicative of RESPIRATORY FAILURE!!

RECALL: List the relative percentages and their corresponding pressures (mmHg) of the following elements in the atmosphere at sea level: - N2 - O2 - CO2

Patm = 760 mmHg - N2: 79%, or ~600 mmHg - *O2: 21%, or ~160 mmHg* - CO2: 0.03%, or ~.23 mmHg

Which scientist came up with the idea of bond hybridization...and the idea of covalency

Pauling

Which value measures how quickly you can exhale?

Peak expiratory flow (PEF)

TEST: The interalveolar septum is perforated by alveolar _____ to allow for what?

Perforated by alveolar PORES (Pores of Kohn) to allow for FREE FLOW of AIR between the alveoli

TEST: What does PERFUSION limited gas exchange mean? What is one example of this?

Perfusion-limited gas exchange refers to the *diffusion of a substance from the air to the blood being limited by how much non-saturated blood is present* - Example: Nitrous oxide is HIGHLY SOLUBLE and RAPIDLY taken up by pulmonary capillary blood, so the only way to get more N2O gas exchange into blood is by INCREASING the PERFUSION!

TEST: Bronchiectasis is a permanent ______ of the bronchi and bronchioles resulting from the ________ of mucosal and elastic tissues. It is most commonly associated with/caused by what?

Permanent DILATION of the bronchi and bronchioles, resulting from the DESTRUCTION of mucosal and elastic tissues. Most commonly associated with/caused by BACTERIAL INFECTIONS

RECALL: How do you calculate total pressure of a given gas (Pgas) in a mixture if you know the total pressure of the mixture? What else do you need to know?

Pgas = P total x %gas You need to know the FRACTIONAL CONCENTRATION of the gas (in this case, % gas in the mixture) Note: Ptotal for air is just Patm for dry air, or Pinspired for inspired air

What is a common form of SUBACUTE hypersensitivity pneumonitis?

Pigeon-breeder's lung

When listening to adventitious breath sounds you should always try to note what items?

Pitch Location Intensity Timing (insp/exp) Continuous/discontinuous

A sound in BOTH inspiration and exp. is?

Pleural Effusion Rub Pleurisy, PE, Pulm Infarct

TEST: What is the most common manifestation of asbestosis in the lungs?

Pleural PLAQUES There are also asbestos bodies (ferruginous bodies)

An abnormal collection of fluid in the plural space resulting from excess fluid production or decreased absorption is known as a

Pleural effusion

Crackles are attributed to

Pleural effusion ILD/Pulmonary Fibrosis Bronchiectasis Atelectasis

RECAP: Pleural pressure increases on (inspiration/expiration) and decreases on (inspiration/expiration). It will always be less than ________ pressure.

Pleural pressure increases on EXPIRATION and decreases on INSPIRATION. *It will always be less than ALVEOLAR pressure (so the lung doesn't collapse!)*

The common causes of a Pleural Friction Rub are pleural diseases that extend to the pleural surface. This could be: When is this sound heard?

Pneumonia PE Pulmonary Infarction Heard: BOTH inspiration and expiration

Gas within the peritoneal cavity of the abdomen that has escaped from the Bowel lumen is known as? What do you look for in the radiological finding?

Pneumoperitoneum (free air) Look for: free gas beneath the diaphragms on an erect chest radiograph

TEST: What happens if air gets into the pleural cavity space?

Pneumothorax (collapsed lung!)

Based on the Law of LaPlace: 1. ___________ is directly proportional to surface tension (T); and inversely proportional to radius of alveoli (r). 2. Pressure in smaller alveolus would be ___________ (greater/less than) than in larger alveolus, if surface tension were the same in both.

Pressure in alveoli (P) greater

RECALL: What is the pressure of water vapor in inspired air (inside the alveoli)? How does this change the pressure of oxygen in inspired air?

Pressure of H2O in inspired air ~ 47 mmHg (at 37 degrees C) Since the PO2 of dry air is about 150-160 mmHg (20% of total atm pressure), the PO2 of *inspired air would be that minus the PH2O inside the lungs - PO2 of inspired air ~ 100 mmHg (in alveoli)*

RECALL: How do you calculate the pressure of inspired air from atmospheric pressure of air?

Pressure of inspired air = Atmospheric pressure of air - pressure of water vapor at body temperature

A SPONTANEOUS PTX has two types, in a PRIMARY, the patient appears ___ ___ ___ In a secondary the pt appears ___ ___ __

Primary = No lung disease Secondary = WITH lung disease *TALL THIN MALES*

TEST: In what compartment(s) of the DCs is the phagocytosed antigen processed and associated with MHC class I?

Processed: PROTEASOMAL compartment Associated with MHC I: Rough ER - Transported from proteasome to rER by TAP protein

TEST: H. influenzae produces _____ _______ that degrade _____ so that it can colonize the lungs

Produces IgA PROTEASES that degrade IgA!! - Recall, IgA is a SECRETORY antibody that can block colonization by binding to the bacteria

TEST: Prolonged _______ can develop into either ______ or ________ coal worker's pneumoconiosis (CWP)

Prolonged ANTHRACOSIS --> SIMPLE or COMPLICATED coal worker's pneumoconiosis

List some of the main adverse events of macrolides

Pseudomembranous colitis symptoms Symptoms of hepatitis/hepatic dysfunction QT prolongation and ventricular arrhythmias

The olfactory zone is lined with what kind of epithelium?

Pseudostratified columnar epithelium

RECALL: What is Dalton's Law of Partial Pressures?

Ptotal = P1 + P2 + ... Pn Total pressure is the sum of the partial pressure of the individual gases

TEST: The pulmonary (arteries/veins) are always on top in the lung hilum. Why?

Pulmonary ARTERIES, because the pulmonary trunk goes UP and then divides

TEST: What is the effective driving pressure for blood flow to the capillaries in zone 2? What does this imply about the blood flow in zone 2?

Pulmonary ARTERY pressure - ALVEOLAR pressure - This implies that blood flow is INTERMITTENT in zone 2, only when the pulmonary arterial pressure is GREATER than the alveolar pressure can blood flow into the alveolar capillaries

Define the Respiratory Exchange Ratio (R)

Pulmonary CO2 elimination rate (VCO2') / pulmonary O2 uptake rate (VO2') R = VCO2' / VO2'

Crackles with a BIBASILAR PREDOMINANCE has a differential of?

Pulmonary Fibrosis Pulmonary Edema (2/2 CHF) - upright bibasilar Asbestos - bibasilar plaques - *Honeycombing on imaging* Pneumonia Bronchiectasis

TEST: What is the effective driving pressure for blood flow to the capillaries in zone 3? What does this imply about the blood flow in zone 3?

Pulmonary arterial pressure - pulmonary VENOUS pressure - Since *arterial pressure is always higher than venous pressure, this implies that blood flow to zone 3 will be CONTINUOUS* .... hence why the *base of the lungs receive the GREATEST BLOOD FLOW* (highest intravascular pressure)

TEST: Rank the following from greatest to least in zone 3 of a normal lung: - Pulmonary arterial pressure - Pulmonary venous pressure - Alveolar pressure Explain briefly your choice

Pulmonary arterial pressure > Pulmonary venous pressure > alveolar pressure Both the arterial AND venous pressure is GREATER than the alveolar pressure, and this is what contributes to the CONTINUOUS FLOW in zone 3! (blood naturally moves into the alveolar capillaries from BOTH arterial and venous side!)

TEST: Rank the following from greatest to least in zone 2 of a normal lung: - Pulmonary arterial pressure - Pulmonary venous pressure - Alveolar pressure Explain briefly your choice

Pulmonary arterial pressure > alveolar pressure > pulmonary venous pressure The pressure in the pulmonary arterioles must be GREATER than that in the alveoli so blood actually flows INTO the alveolar capillaries! Similarly, the pressure in the alveoli needs to be higher than the venous pressure so it can drive blood flow OUT of the alveolar capillaries

TEST: The pulmonary circulating pressures are remarkably (high/low), the blood vessel walls in the pulmonary circulation are (thick/thin) at all levels, and it contains far (more/less) smooth muscles than systemic circulation vessels

Pulmonary circulating pressures are remarkably LOW, blood vessel walls are THIN at all levels, and it contains far LESS smooth muscle than systemic circulation

Bruising of the lung parenchyma is a

Pulmonary contusion

What are the two pathologies generally found in Restrictive lung disease?

Pulmonary fibrosis - *"hazy ground glass appearance"* Sarcoidosis

Chronic hypersensitivity pneumonitis can ultimately progress to what?

Pulmonary fibrosis or cor pulmonale

TEST: Stage 4 sarcoidosis presents with what common CXR finding? Is this finding worse in the upper or lower lobes of the lung?

Pulmonary fibrosis with HONEYCOMBING, worse in the UPPER lobes

TEST: List the typical MEAN pulmonary vascular pressures at the artery, arterioles, capillaries, and the venules/veins Compare this to the typical MEAN systemic vascular pressures at each of these levels

Pulmonary vascular pressures: - Artery: ~15 mm Hg (100) - Arterioles: ~12 mm Hg (30) - Capillaries: ~10 mm Hg (20) - Venules/veins: ~8 mm Hg (10) () = the corresponding mean systemic pressure

Pulmonary vascular resistance is normally very (small/large) in magnitude, and is under both ______ and ______ influences. Which one dominates under normal conditions?

Pulmonary vascular resistance is normally very SMALL in magnitude, and is under both PASSIVE and ACTIVE influences. - Under normal conditions, PASSIVE dominates

What is Pulmozyme, and how does it help in treating CF?

Pulmozyme = DNase - This works by breaking up long-strands of DNA into smaller segments within the sputum, allowing INCREASED CLEARANCE of mucus in patients with CF

On the ___ side we have 2 fissure where on the other side (___) we have ___.

R = 2 fissures L = 1 fissure

Which bronchus is wider? What else is special about this bronchus? What is clinically significant here?

R bronchus is more verticle and WIDER Foreign body will most likely be ASPIRATED here

TEST: What nerve ascends up the tracheoesophageal groove and innervates the majority of the muscles of the larynx? What is the only laryngeal muscle not innervated by this nerve, and what is it actually innervated by?

RECURRENT LARYNGEAL N. The only laryngeal muscle it doesn't innervate is the CRICOTHYROID muscle - This is innervated by the EXTERNAL branch of the superior laryngeal nerve

TEST: What is the purpose of using oral antibiotics in treating CF?

REDUCING THE INFLAMMATION! - Some are effective against pseudomonas, but most are used for the inflammation

Step 5 asthma care is ____ based

REFERALL Referred to a specialist.

TEST: In order for the lung to collapse on exhalation, what needs to happen?

RELAX the diaphragm!

TEST: The respiratory zone is lined with what kind of epithelium?

RESPIRATORY (pseudostratified ciliated columnar, PSCC)

AGAIN: What represents the transition from conducting to the respiratory portion of the lung?

RESPIRATORY bronchioles!!

RECALL: The inner lining of the lungs are lined by what kind of epithelium?

RESPIRATORY epithelium - Pseudostratified CILIATED columnar epithelium

TEST: What kind of epithelium lines the paranasal sinuses? What kind of cells are contained within this epithelium?

RESPIRATORY epithelium (psedostratified ciliated columnar epithelium, or PSCC) - Contains numerous GOBLET CELLS

TEST: The trachea is lined by _______ epithelium

RESPIRATORY! (PSCC)

TEST: Patient shows: - FVC = 34% - FEV1 = 38% - FEV1/FVC = 95.65 Does this patient have a restrictive or obstructive defect? Why?

RESTRICTIVE - The patients FEV1/FVC is greater than 80%(rules out obstructive as well) - The patients FVC is well under 80%

AGAIN********* What can you not diagnose on spirometry alone? What other test(s) must you do?

RESTRICTIVE defects Other tests to do: - Lung volumes (TLC) - DLCO

Which type of lung disease has a more negative impact on a person's total lung volume?

RESTRICTIVE= decreased peak expiratory flow (PEF) and total lung capacity (TLC) OBSTRUCTIVE= residual volume (RV) is greatly increased, and the effort-independent portion of curve is depressed inward.

What structures are Oriented anteroposteriorly, and defined as the ___ ____ the supporting ___ ligament and striated ___ ____

RIMA GLOTTIS Vocal LIGAMENT VOCAL MUSCLES

From out to in, what are the layers of the pleura?

RIb Endothoracic Fascia Costal Parietal Pleura Pleural Cavity VISCERAL Pleura Lung

If an X ray is ______ it will permit the passage of X-rays with little attenuation so the areas are dark on the film

Radio Lucent

In a radio _____ x-ray, the passage of the x-rays are ____ so the areas are light or white on the film

Radiopaque Obstructed

TEST: What are some of the most common adverse reactions to Rifampin?

Rash INCREASED BILIRUBIN Hepatotoxic ORANGE GATORADE coming out of your body

Patient with FEV1/VC% = 33%

Ratio = 33% = obstruction (<70) FEV1 = 30% = obstructive ventilatory defect

****What is the reed thickness? What is the normal?

Ratio of the *thickness of the mucous gland to the thickness of the wall between epithelium and the cartilage* *Normal: <.04*

What are the main components of the respiratory portion of the respiratory system?

Respiratory bronchioles, alveolar ducts, and alveoli

The recurrent laryngeal nerve ultimately becomes the ______ laryngeal nerve, above the ________ joint

Recurrent laryngeal nerve --> INFERIOR laryngeal nerve above the CRICOTHYROID joint

TEST: Macrolides have been shown to reduce _________ but NOT treat the _________

Reduce INFLAMMATION but NOT treat the INFECTION

What is the difference between reduced oxygen carrying capacity and hypoxemia?

Reduced oxygen carrying capacity refers to HEMOGLOBIN DEFICIENCY, whereas hypoxemia *refers to DECREASED PO2 in blood*

DNAase therapy ___ mucus viscosity in secretions containing ___ from cell debris within the lung. This therapy is useful in ___ patients

Reduces mucus viscosity in secretions containing DNA Useful in cystic fibrosis patients

MOA for Dornase Alfa?

Reduces the viscosity and adhesiveness of infected sputum Improves FEV1 in CF patients

TEST: Define compliance of the lung. What is it a reflection of?

Reflection of the DISTENSIBILITY of the lungs - Compliance = change in lung volume / change in intrapleural pressure

What is a distinguishing feature between the respiratory and terminal bronchioles?

Respiratory bronchioles have scattered ALVEOLI that bud from the walls!

TEST: With a restrictive disorder, the (inspiratory/expiratory) phase of breathing requires more work With an obstructive disorder, the (inspiratory/expiratory) phase of breathing requires more work

Restrictive: INSPIRATORY phase requries more work Obstructive: EXPIRATORY phase requires more work

TEST: List the common side effects and contraindications of ethambutol

Retrobulbar/optic neuritis Decrease in visual field, decrease in red-green color discriminiation Hyperuricemia CONTRAINDICATION: - Pre-existing optic neuritis - Inability to report visual disturbances (young patient)

Males generally develop more ILD associated with ___ __ and ____

Rheumatoid Arthritis and Pneumoconiosis

In a foreign body aspiration, which lobes are mainly affected? An _____ tube used for intubation, if inserted too far, will lodge where?

Right Middle and Lower lobe ENDOTRACHEAL TUBE lodges in *RIGHT mainstream bronchus*

Which ribs are LARGER?

Right Ribs are > left They are *also POSTERIOR with the R hemidiaphragm*

The right superior bronchus has ___ divisions, the middle bronchus has ___ divisions, and the inferior bronchus has ____ divisions

Right: - Superior: 3 divisions (Apical, posterior, anterior) - Middle: 2 divisions (lateral, medial) - Inferior: 5 divisions (anterior/posterior basal, lateral/medial basal, and to the back of the inferior lobe)

What is the black box warning for Omalizumab?

Risk of acute delayed onset anaphylaxis, causing: - Bronchospasm, hypotension, syncope, urticaria, angioedema of throat/tongue

What is a PDE-IV inhibitor that is not a methylxanthine?

Roflumilast

Rhonchi is inconsistently used but refers to ___, ___, ___

Rumbling, course and snoring breath sounds

The alveolar ____ is the terminal part of the airway

SAC

TEST: What is the only long acting beta2 agonist (LABA) discussed in class?

SALMETEROL!

Fluticasone is (widely/scarcely) bioavailable systemically, which makes is (more/less) safe than other corticosteroids

SCARCELY bioavailable as it is 99% bound up and incompetely absorbed - Makes it MORE safe than other corticosteroids, as only the unbound form is pharmacologically active

100% of patients with ________ have interstitial lung disease

SCLERODERMA

The entire pulmonary circulation is in (parallel/series) with the entire systemic circulation

SERIES

Step 4 asthma and step 5 asthma are ___ and their managements are?

SEVERE Combo of LOW dose ICS/Formoterol as maintenance Medium dose ICS/LABA w/ SABA prn

Albuterol is a (LABA/SABA)

SHORT-ACTING beta agonist!

AGAIN: In type III hypersensitivity reactions, antibodies recognize and bind to what kind of antigens? What happens after binding of the Ab to the Ag?

SOLUBLE antigens, forming a Ag:Ab complex that is floating around in the serum - This complex can then get DEPOSITED in the wall of a blood vessel, which will then activate COMPLEMENT and attract NEUTROPHILS

The TIME it takes something to diffuse is proportional to the ___ of the ___ it has to go.

SQUARE of the DISTANCE it has to go. *1 nm = 1nsec*

Males with a deletion of F508 on chromosome 7 is almost always what?

STERILE! - However, sexual function is still preserved

Pyrazinamide has a potent _______ ability within (acidic/basic) environment of areas of acute inflammation

STERILIZING ability within ACIDIC environments

*******STOP HERE - this is all that has been covered***********

STOP

(Simple/stratified) (columnar/squamous) epithelium covers the vestibule of the external nose

STRATIFIED SQUAMOUS epithelium

Define oxygen content of blood

SUM of the oxygen BOUND to Hb and then DISSOLVED oxygen

The innervation to the diaphragm is mostly from the (superior/inferior) surface

SUPERIOR!

TEST: Stage 2 sarcoidosis usually presents (symptomatically/asymptomatically) with what common CXR finding?

SYMPTOMATICALLY Most common CXR finding: *Bilateral hilar LAD WITH lung involvement*

You see bilateral hilar lymphadenopathy on CXR. What is on your DDx?

Sarcoidosis TB Viral infection Fungal infection Lymphoma

What is the MAJOR difference between sarcoidosis and tuberculosis?

Sarcoidosis is characterized by the presence of NON-CASEATING granulomas (collections of inflammatory cells), as opposed to the caseating granulomas (either fibrosed or liquified) in tuberculosis

Patients with intermittent asthma are treated with what?

Short acting beta agonists (SABA)

TEST: What are the major uses of the viral coating inhibitors?

Seasonal PROPHYLAXIS of influenza A virus infections in HIGH-RISK patients

The ______ _______ anchors down the developing diaphragm and is a literal separation of the thoracic and abdominal cavities

Septum transversum!

TEST: What is the most prevalent chronic occupational disease worldwide?

Silicosis (inhaling silicon dioxide)

TEST: What kind of epithelium lines respiratory bronchioles? This is similar to what conduction structure?

Simple CILIATED CUBOIDAL epithelium, just like the TERMINAL bronchioles!

What is the effect of nitric oxide on oxygen delivery in the periphery?

Since hemoglobin also carries nitric oxide, it IMPROVES oxygen delivery in the periphery!

What test allows you to measure the anatomic dead space?

Single-breath nitrogen test (Fowler's method)

What three structures/organs does sarcoidosis MOST commonly affect?

Skin, lymph nodes (especially HILAR lymph nodes), and lungs However, sarcoidosis can affect ANY organ

AGAIN: What parts of the nasal cavity defines the olfactory zone?

Small area of the ROOF of the nasal cavity, and the UPPER portions of the nasal septum and superior concha

***What are some causes of Wheezes?

Smooth airway muscle constriction Muscosal Edema Secretions mucus *classic triad of asthma*

Fine crackles are ___ and ____. Their timing is __. What diseases are indicative of these types of crackles?

Soft & high pitched Timing = BRIEF INTERSTITIAL DISEASES: ILD/Pulmonary fibrosis, CHF, ATELECTASIS

What two species of bacteria do neurophils provide protection against in the context of respiratory infections?

Staph Aureus and Strep Pneumonia

What changes of compliance do you see with and without surfactant?

Surfactant increases the compliance of the lung (and reduces the work of breathing) by lowering the surface tension. It also makes the alveoli more stable and less likely to collapse.

TEST: What is surfactant? What kind of cells produce it, and what is it's function?

Surfactant: Phospholipid Produced by: Alveolar Type II cells Function: REDUCE alveolar SUFACE TENSION by becoming interspersed between H2O molecules (PREVENTS ALVEOLAR WALLS FROM STICKING TO EACH OTHER as they collapse!)

To determine the severity of asthma, what are some factors to look at?

Symptom timing Amount of nighttime awakenings How often they use short-acting beta agonists Lung function tests

TEST: In acute hypersensitivity pneumonitis, the symptoms characteristically occurs __-__ hours after initial exposure, but these symptoms will resolve with what?

Symptoms occur 4-6 hours after initial exposure, but if you REMOVE the OFFENDING AGENT (moldy hay for example), the symptoms will subside within 12 hrs-days

TEST: Describe how beta-2 agonists are used to treat the symptoms of asthma (List the MAIN mechanism)

Symptoms: Smooth muscle contractions --> bronchospasms and NARROWING of airway Beta-2 agonists: Activates ADENYLYL CYCLASE --> cAMP buildup --> PKA activation --> phosphorylates MLCK --> Turns MLCK OFF --> Smooth muscle RELAXATION --> BronchoDILATION Recall: When MLCK is on, it phosphorylates myosin to allow for crossbridge formation with actin --> contraction

Type II alveolar cells synthesize and store pulmonary ________ in membrane bound _______ _______ until it is needed. What is the function of this stored product?

Synthesize and store pulmonary SURFACTANT in membrane bound LAMELLAR BODIES. Surfactant function: REDUCES surface tension at the air-fluid interface, and thus reduces the tendency of the alveolus to COLLAPSE at the end of expiration!

TEST: Systemic capillaries show conditions that make the oxygen affinity of hemoglobin (higher/lower) than that of pulmonary capillaries. Why?

Systemic capillaries show conditions (increased acidity and pCO2) that make the oxygen affinity of hemoglobin LOWER than that of pulmonary capillaries. - By promoting a low oxygen affinity state, hemoglobin can DELIVER OXYGEN TO THE TISSUES MORE EFFICIENTLY!

TRUE or FALSE: The partial pressure of O2 in the dead space is greater than that in the alveoli

TRUE, because the dead space O2 does not decrease after inspiration because there is NO gas exchange with the blood, while the alveoli will lose O2 to the blood to the PO2 in alveoli will be LESSER than that in the dead space)

TRUE or FALSE: At the end of expiration, atmospheric pressure is equal to alveolar pressure

TRUE, both are 0 cm H2O

TRUE or FALSE: The epithelium lining the PRIMARY bronchi is the same as that lining the trachea

TRUE, both are lined by RESPIRATORY epithelium (PSCC)

TRUE or FALSE: Patients with asthma can develop COPD

TRUE, if asthma is left untreated! - Over time, the chronic inflammation associated with asthma can damage the alveoli and result in a fixed obstructive defect!

TRUE or FALSE: Normally, blood does distribute to the top of the upright lung

TRUE, it is just REALLY LOW - Intravascular pressures are just great enough to PREVENT continuous vessel collapse at the top of the lung

TRUE or FALSE: Rifampin should be administered on an empty stomach

TRUE, maximum plasma conc for rifampin can be reduced by 1/3 with food

TRUE or FALSE: Pressure is greater inside the lungs during exhalation

TRUE, so air flows down its pressure gradient and moves OUT of the lungs

TRUE or FALSE: As the environment has evolved in the past couple billion of years, the atmosphere has changed from a reducing to an oxidizing medium

TRUE, there is much more O2 now in the atmosphere, allowing it to be an available electron receptor for oxidative phosphorylation and ATP generation

TRUE or FALSE: CWP is predominantly a benign disease

TRUE, until it progresses into complicated CWP (massive fibrosis)

What catheters are used for chemo?

TUNNELED - create ports

AGAIN: It takes ____ IgE on the mast cell surface and the antigen to trigger degranulation

TWO, as they need to CROSS-LINK

Chronic repeated use of beta2 agonists can cause _________

Tachyphylaxis (sudden decreased response to drug)

TRUE or FALSE: A 22 year old can develop idiopathic pulmonary fibrosis

Technically TRUE, but this is EXTREMELY unlikely because idiopathic pulmonary fibrosis is more of an END-STAGE disease and usually takes many years before reaching this

TEST: In obstructive defects, there tends to be a decreased ________ (volume/flow) at (higher/lower) lung (volumes/flow), producing what characteristic pattern on a flow-volume loop?

Tends to be a decreased FLOW at LOWER lung VOLUMES Produces a characteristic CONCAVE (scoop out) loop on the EXHALATION part of the flow-volume curve (top part)

What developmental period of the lungs does the epithelium become squamous?

Terminal sac period!

Which CD4 T-cell subtype is critical in controlling M. tb infection? List two cytokines that these cells secrete and what the major function of each is in fighting tuberculosis infection

Th-1 cells, secreting: 1) IL-2: Induces T-cell EXPANSION 2) IFN-gamma: Activates MACROPHAGES

Which CD4 T cell subset plays a major role in hypersensitivity reactions?

Th2 cells!!

What is one example of a hemoglobinopathy?

Thalassemias! Enlarged spleen, bone deformities, iron overload

TEST: When you are upright, the majority of pulmonary blood flow is to what part of the lung?

The BOTTOM

TEST: What is the signal from infected cells that triggers NK cells to lyse them? How is this signal different from normal, uninfected cells?

The DECREASED expression of MHC class I on their surface, as viral infections DOWNREGULATE their expression! - So when they sense there isn't as much MHC I, that triggers the NK cells to sense those cells as abnormal and so they target them (whereas in normal cells, the MHC class I expression should be much higher)

90% of all pulmonary emboli (PE) originate from where?

The DEEP VENOUS system (thrombosis) in the lower extremities

AGAIN/TEST: The first division of the laryngotracheal ridge becomes what?

The L and R primary bronchi

TEST: Define inspiratory reserve volume (IRV)

The MAXIMUM amount of air that can be *INHALED AFTER a normal tidal volume inspiration* (so IRV is from the TOP of a tidal inspiration, as in it doesn't include the tidal inspiration)

TEST: Define expiratory reserve volume (ERV)

The MAXIMUM amount of air that can be EXHALED from resting expiratory volume (after normal tidal volume expiration)

AGAIN: What component of bacteria does lysozyme degrade?

The PEPTIDOGLYCAN in their cell walls

What is P50 on a oxyhemoglobin dissociation curve? What does it reflect?

The PO2 at which hemoglobin is HALF-saturated with O2 - Reflects a carriers AFFINITY for oxygen

TEST: The (right/left) bronchus has an intermediate segment

The RIGHT bronchus, which lies between the superior and middle secondary bronchi

TEST: The mesoderm of the body wall ultimately forms what part of the diaphragm? What nerve innervates this, and what region does this mesoderm come from?

The SKELETAL MUSCLE - Innervated by C3, 4, 5!!! - Mesoderm comes from the CERVICAL region (makes sense, since the phrenic nerve starts innervating it there)

The pattern of development in lung maturation is that the (superior/inferior) parts will develop first, and then the rest will follow

The SUPERIOR parts will develop first, regardless of the period of development

Describe the "Compliance Curve".

The curve describes the change in tidal volume over the change in intrapleural pressure. The steeper the line, the more compliant the lungs. Lung compliance is less at TLC compared to FRC.

What sits right under the laryngeal epithelium?

The VOCAL LIGAMENT and VOCALIS m.

TEST: The actual division of the laryngotracheal ridge, which is (ecto/meso/endo)-derm, will only form the ________ of the lung buds/bronchial tree. What structure is embedding the lung buds, and what does this ultimately form?

The actual division of the laryngotracheal ridge, which is ENDODERM, will only form the EPITHELIUM of the lung buds/bronchial tree (trachea, bronchi, alveoli). Embedding these lung buds is SPLANCHNIC MESODERM that ultimately develops into: - Connective tissue (loose and dense) - Cartilage - Blood vessels

AGAIN: In type II hypersensitivity reactions, antibodies recognize and bind to what kind of antigens? What happens after binding of the Ab to the Ag?

The antibodies in type II bind to CELL-BOUND antigens (structural components of cell surfaces) - This Ab:Ag complex activates COMPLEMENT proteins by the CLASSICAL pathway --> cell surface opening --> cell death This is called antibody-dependent cell-mediated cytotoxicity (ADCC)

TEST: Briefly describe the pathogenesis of mycoplasma pneumoniae on respiratory epithelial cells (hint: starts with a tapering end of bacteria interacting with something on the cells)

The bacteria has a TAPERING end with CYTOADHESINS that facilitate attachment to the SIALIC ACID of the epithelial cells. This interaction between cytoadhesins and sialic acid on the epithelial cells --> REACTIVE OXYGEN INTERMEDIATES These intermediates CANNOT kill the bacteria, as they have CATALASE to degrade them. This means that these intermediates end up DAMAGING the HOST respiratory epithelial cells!!

In aqueous solutions, what is the rate-limiting factor for the time it takes O2 to diffuse?

The diffusion* DISTANCE of O2!!*

RECALL: When you swallow, what laryngeal structure folds over to cover the larynx so that food doesn't go down the wrong tube?

The epiglottis!

TEST: What is the most common version of a tracheoesophageal fistula? How does this manifest clinically? (what is the clinical clue for this?)

The esophagus has a break due to stopping development prematurely (leading to a closed end, called ESOPHAGEAL ATRESIA), and a fistula forms between the trachea and the remaining esophagus below the break Manifests clinically as an infant/child that you can't feed! Every time you try to feed the child and they swallow, they will cough up the food! - The risk of this is they can aspirate the food into their lungs!

TEST: The first episode of idiopathic emboli is treated for ___ months, but if they have a high-risk for bleeding then you would treat them for ___ months

The first episode of idiopathic emboli is treated for SIX months, but if they have a high-risk for bleeding then you would treat them for THREE months only NOTE: Now, the guidelines for the first episode of idiopathic emboli is treating for THREE months and then REASSESSING (based on further risks and patient preference)

TEST: List whether the following increases or decreases to cause an INCREASE airway resistance: - Airway diameter - Turbulence - Rapid breathing - Lung volume - Bronchial smooth muscle contraction - Gas density

The following can cause an increase in airway resistance: - Airway diameter DECREASES - Turbulence INCREASES - Rapid breathing INCREASES (which increases turbulent flow) - Lung volume DECREASES - Bronchial smooth muscle contraction INCREASES - Gas density INCREASES

The lingula on the (right/left) lung is equivalent to the _______ lobe on the (right/left) lung.

The lingula on the LEFT lung is equivalent to the MIDDLE lobe on the RIGHT lung

TEST: The lung buds develop (from/around) the _______arches (now called ______ arches). Which two arches are associated with larynx and pharynx development?

The lung buds develop AROUND the BRANCHIAL arches (now called PHARYNGEAL arches). The 4th and 6th arches are associated with larynx and pharynx development

The lung volume at any given pressure during (inspiration/expiration) is larger than that during (inspiration/expiration)

The lung volume at any given pressure during EXPIRATION is larger than that during inspiration - Even as you are exhaling, the pressure in the lungs is still greater than if you were inhaling

TEST: What does expiratory peak flow (PEF) measure?

The maximum FLOW generated during EXPIRATON performed with maximal force and started after a full inspiration

TEST: What is the clinical significance of the bronchus that drains the superior segment of the inferior lobe?

The position of that bronchus is horizontal, so if a patient has a pulmonary infection and they lie down, the excess fluid could build up and go through that bronchus and affect the inferior lobe

TEST: What keeps the lungs from collapsing on itself? What is significant about the fluid within the pleural cavity in regards to this?

The pressure of the FLUID inside the pleural cavity!! - Since the fluid within the pleural cavity is made of water, water is an INCOMPRESSIBLE fluid. This is significant because as the diaphragm flattens and the cavity expands, the volume inside the pleural cavity CANNOT CHANGE (since water can't expand or compress). The only way the lungs can compensate for this increased cavity space is for the lungs to breathe in AIR!! - Likewise, when the diaphragm moves up into the lungs as it relaxes, it is the fluid inside the cavity that prevents it from compressing the lungs!!

TEST: What is the most posterior structure in the lung hilum?

The primary BRONCHUS

What is the critical opening pressure of the lungs?

The pulmonary artery pressure required to open collapsed extra-alveolar vessels and restore flow

The respiratory ciliated epithelial cells produce _________ peptides like ________, which are (anionic/cationic) peptides that bind to the surface of _______ cells and inactivates them

The respiratory *ciliated epithelial cells* produce *ANTIMICROBIAL peptides like BETA-DEFENSINS, which are CATIONIC peptides* that bind to the surface of BACTERIAL cells and inactivates them

TEST: The respiratory epithelium of the respiratory zone contains a highly vascular ______ _______. This has a rich venous network of _______ bodies that are capable of becoming engorged with _____. This system helps with _______ of the inspired air, and together with seromucous glands also help _______ the inspired air.

The respiratory epithelium has a highly vascular LAMINA PROPRIA. This has a rich venous network of SWELL BODIES that are capable of becoming engorged with BLOOD. - This system helps with WARMING of the inspired air - Together with seromucous glands, it also helps MOISTEN the inspired air

TEST: The resulting vacuole in the macrophage that contains the ingested organism is called a ________, which will fuse with ________ present in the cytoplasm of the macrophage to form a __________

The resulting vacuole in the macrophage that contains the ingested organism is called a PHAGOSOME, which will fuse with LYSOSOMES present in the cytoplasm of the macrophage to form a PHAGOLYSOSOME

TEST: Where is the interstitium of the lung?

The space between the basement membranes of the alveolar lining epithelium and the capillary endothelium

TEST/RECAP: The trachea has a superficial mucosal surface lined with ________ epithelium sitting over a _______ ______. Deep to this is the ________ layer, and finally the tracheal _________ as the deepest layer

The trachea mucosal surface is lined with RESPIRATORY epithelium sitting over a LAMINA PROPRIA. Deep to this is the SUBMUCOSAL layer, and finally the tracheal CARTILAGE as the deepest layer.

In Goodpasture's syndrome, anti-GBM antibodies are directed against what component of the glomerular basement membrane?

The type IV collagen, specifically the noncollagenous (NC-1) domain of the alpha 3 chain

TEST: The vocal folds attach anteriorly to the ________ cartilage and posteriorly to the _________, which are extensions of the ________ cartilage. These are what can pivot and allow for __________.

The vocal folds attach anteriorly to the THYROID cartilage and posteriorly to the ARYTENOIDS, which are extensions of the CRICOID cartilage. These are what can pivot and allow for VOCALIZATION!

TEST: Define Functional Residual Capacity (FRC)

The volume of air present in the lungs at the end of passive expiration

What is one major PDE inhibitor? Why is it uncommon to use this?

Theophylline - This is uncommon to use because of its major interactions with liver enzymes, resulting in either increased or decreased clearance

List two drugs that can decrease the plasma levels of Zafirlukast

Theophylline and Erythromycin

What happens to the equal pressure point in obstructive conditions such as emphysema?

There is LESS of a gradient, and then EPP occurs more or less throughout the entire airway so that air DOES NOT LEAVE the airway due to the lack of driving pressure (pleural pressure isn't higher than alveolar pressure to drive air out)

After macrophages recognize the infecting microbe, what happens? Where on the macrophage surface does this occur?

They ENGULF them via PHAGOCYTOSIS - Occurs at CLATHERIN-COATED pits (clathrin mediated endocytosis)

RECAP: Briefly describe the macrophage defense mechanism of microbe control

They engulf the microbe by phagocytosis, and it can digest/kill it by either the oxygen dependent (reactive oxygen/nitrogen species) or independent pathway (lysosomal enzymes) of macrophages

Why are the intercostal muscles important for respiratory function?

They provide tension and maintains the pressure inside the pleural cavity, so the lungs won't collapse on itself!

What is the function of the supportive cells in the olfactory epithelium?

They secrete odorant binding proteins (OBPs)!!! - These bind odorants, and then they get bound by receptors on the olfactory cilia to stimulate signaling for smell!

RECALL: What are the cholinergic side effects?

Think PARASYMPATHETICS: - Diarrhea - Urination - Miosis BRONCHOSPASM (smooth muscle contract) - Bradycardia - Emesis - Lethary - Lacrimation - Salivation (and seizures) This is why it makes sense to use ANTIcholinergics to induce bronchoDILATION

Surfactant reduces surface tension. What does this do as a result?

This prevents the exchange of air from the smaller alveoli into the larger ones. It basically makes the smaller ones open easier (Law of LaPlace is P= 2T/r ).

What type of tube is used to help drain fluid which is trapped in the plural cavity

Thoracostomy tube

Sarcoidosis is more common in what population?

Those of Scandinavian descent and African Americans

What is the mechanism to the resistance against amphotericin B in some fungal species? Name two species that show some resistance.

Those species REDUCE synthesis of ERGOSTEROLS (and may even replace them with precursor sterols)!! - Species: CANDIDA and ASPERGILLUS

What is the ideal patient for Omalizumab?

Those with persistent asthma who have a POSITIVE SKIN TEST (or in vitro) to an AEROALLERGEN, and whose symptoms cannot be controlled with inhaled corticosteroids

Projecting from the lateral wall of the nasal cavity are three _______ or ________. What is their main function?

Three CONCHAE or TURBINATES - Function: Increases the surface area as well as creates turbulent airflow to allow more CONDITIONING of the inspired air

Besides Heparin and Coumadin, what other classes of anticoagulants can be used to treat PE?

Thrombolytics (like tPA) Factor Xa inhibitors (like Rivaroxaban) Direct thrombin inhibitors (Dabigatran)

RECAP: In order to make a Dx of an OBSTRUCTIVE defect, you look at what? Then, to determine the severity, you look at what?

To make Dx: FEV1/FVC < 70% predicted Severity: Look at FEV1%

TEST: What is the fate of the pleuroperitoneal folds?

To migrate towards the septum transversum and fuse with that, as well as with the mesoesophagus, to form a pleuroperitoneal MEMBRANE

When should CF screening be offered?

To patients with: - a positive family history of CF - Partners of people with CF - Couples currently planning a pregnancy - Couples seeking prenatal testing

Resistance to fluoroquinolones is dependent on alterations in bacterial what?

Topoisomerase IV or DNA gyrase!

Total pulmonary vascular resistance is lowest at or near the ______. Briefly explain why.

Total PVR is lowest at or near the FUNCTIONAL RESIDUAL CAPACITY (FRC) - If the *lung volume is LARGE (greater than FRC), extra-alveolar vessels experience traction and are distended to a greater extent* but the alveolar vessels get stretched and compressed (increases PVR) - If the *lung volume is SMALL (smaller than FRC), extra-alveolar vessels experience LESS TRACTION and LESS DISTENSION than at large lung volumes.* Moveover, there is LESS COMPRESSION of the alveolar vessels. (but still increases PVR to an extent) Take home: High or low lung volumes will cause combined effects of alveolar and extra-alveolar vessels, INCREASING PVR in BOTH CASES

RECAP: After spirometry suggests a possible restrictive defect, what do you need to look at now?

Total lung capacity (volume) - TLC < 80% predicted = restrictive defect

Oseltamivir is a transition-state analog of ______ ____, thus acting to inhibit _________ of what influenza type(s)

Transition-state analog of SIALIC ACID, thus acting to inhibit NEURAMINIDASE (since it cleaves sialic acid on host cell) of types A and B influenza virus

TEST: What are the four main types of hypersensitivity reactions, and what is each type MEDIATED by?

Type I - ALLERGY - IgE mediated, bound to MAST cells Type II - CYTOTOXIC, Ab-dependent (ADCC) - IgG (or IgM) mediated on CELL SURFACE leading to COMPLEMENT ACTIVATION Type III - IMMUNE COMPLEX - SOLUBLE Ag:Ab complex mediated (usually IgG) --> COMPLEMENT activation Type IV - DELAYED type - Immune cell-mediated (macrophages, DCs, T cells)

ATOPIC asthma is a type I hypersensitivity rection involving ____ + ____ cells which releases ___, ___ , ___ and _____.... What cell types are stimulated in Atopic asthma?

Type I = CD4 + TH2 Releases IL-4, IL-5, IL-9, IL-13 Stimulates Eosinophils, mast cells and IgE

AGAIN: Which alveolar cell type covers the majority of the alveolar surface?

Type I alveolar cells!

Besides producing surfactant, what is the other function of type (I/II) alveolar cells?

Type II alveolar cells also MAINTAIN and REPAIR alveolar epithelium when injury occurs (in addition to producing surfactant)

AGAIN: What is the main difference between type II and type III hypersensitivity reactions?

Type II involves Ab binding to an antigen on the CELL SURFACE --> complement activation Type III involves Ab binding to SOLUBLE antigen that is floating around in the serum --> complement activation Note: Both involves complement activation, and both takes HOURS-DAYS to activate

Berylliosis is a type __ hypersensitivity reaction and leads to ___ in the longs and __ ___ which mimic ____

Type IV Leads to GRANULOMAS in kungs and HILAR LYMPH nodes mimicking SARCOIDOSIS

TEST: Which hypersensitivity reaction is antibody independent?

Type IV, delayed type hypersensitivity

IPF and ___ are synonymous ___ cause ___ kind of ILD

UIP - Usual Interstitial Pneumonia Unknown cause WORSE kind of ILD

Answer to 2

UIP - because there is CLUBBING

Endogenous reactivation of M. tb usually occurs in which lobe of the lung, and why?

UPPER lobe of the lung, because it is more OXYGENATED (better environment for the bacteria to survive)

TEST: The early stage of silicosis involves small nodules in the (upper/mid/lower) zones of the lungs

UPPER!

Someone has an acute DVT or PE. What is the initial treatment?

Unfractionated Heparin or LMW Heparin

Lobar pneumonia which lobe?

Upper lobe on right

The larynx is lined by ______ ______ epithelium at the entire lingual (upper) surface, while towards the base of the epiglottis on the lower side it is covered by _________ epithelium

Upper surface = STRATIFIED SQUAMOUS Lower side = RESPIRATORY (PSCC)

TEST: CXR is useful in suggessting the _______ but not the ______ of ILD

Useful in suggesting the PRESENCE but NOT the STAGE of ILD POOR SENSITIVITY

Interstitial pulmonary fibrosis (IPF) is more or less now called what?

Usual interstitial pneumonia (UIP) - And this is a form of idiopathic interstitial pneumonias (which is the SAME as idiopathic interstitial lung disease)

TEST: The physical exam is usually (normal/abnormal) for hypersensitivity pneumonitis, except for what finding?

Usually NORMAL, except for the RALES (present at or shorter after onset)

What is the equation for vascular resistance in general?

Vascular resistance = (Input pressure - output pressure) / blood flow

TEST: What is the equation to calculate dead space ventilation rate?

Vd' = Vd x f Vd = dead space volume (anatomic or physiologic) f = Respiratory frequency, or the number of breaths per unit time (usually per minute)

Classically, ventilation hypersensitivity pneumonitis is antigenic exposure to what? What are the two most common antigens that can cause this?

Ventilation hypersensitivity pneumonitis is antigenic exposure to ORGANIC MATERIAL in contaminated VENTILATION systems, with the two most common antigens being: - Thermophilic actinomycetes - Aspergillus fumigatus

Where the trachea bifurcates arises the L and R ______ bronchi. After entering the lung parenchyma, they divide into ________ bronchi. There are ____ of these on the R side and ____ of these on the L side. These will continue to divide and subdivide into _______ and _______ bronchi.

Where the trachea bifurcates arises the L and R PRIMARY (main) bronchi. After entering the lung parenchyma, they divide into SECONDARY (lobar) bronchi. There are 3 of these on the R side and 2 of these on the L side. These will continue to divide and subdivide into TERTIARY and SEGMENTAL bronchi!

The peak flow is primarily designed for you to guage what? What can peak flow NOT differentiate between?

Whether the patient has an obstruction It CANNOT differentiate between restrictive from obstructive defects

Case 1

Which test should be performed next?

What is the sound when a patient whispers 1-2-3 and the loudness of the whispered words INCREASES during auscultation?

Whispering Pectoriloquy

On an xray, the more white the more ____ or _____ The more black the more ___ or ____

White = Dense / Radiopaque Black = Black Gas or air / Radiolucent

TEST: How do macrophages recognize microbial infection?

With PATTERN RECOGNITION RECEPTORS *(TLR-4, TLR-2, complement receptors)*

RECALL: How do you calculate PAO2?

With the alveolar gas equation! PAO2 = PIO2 - (PaCO2/R) Normal A-a gradient is 5-10 mmHg

The bud for the lungs starts when?

With the earliest development of the pharynx

A noninvasive medical test that helps physicians dx and treat medical conditions

X ray

What is the OLDEST and most often used form of imaging What is the pt exposed to?

X ray Ionizing radiation (light or radio waves) exposure

RECAP: List the pattern of blood flow in zones 1, 2, and 3 of an upright lung and why we see these patterns

Zone 1: ABNORMAL, NO blood flow Zone 2: INTERMITTENT blood flow Zone 3: CONSTANT blood flow This is because GRAVITY increase intravascular pressure, and as you move further DOWN, the effect of gravity increases and so does the intravascular pressure (so that by the time you reach zone 3, there is a strong driving force for blood to flow INTO the alveolar capillaries) - Also, remember gravity has no effect of alveolar pressure!

TEST: In terms of the distribution of blood flow, what are the three possible zones of the lung?

Zone 1: Top of the upright lung, WHEN IT OCCURS (bloodflow) Zone 2: ~Upper 1/3 of a NORMAL upright lung (no zone 1 present normally) Zone 3: ~Lower 2/3 of a NORMAL upright lung (no zone 1 present normally)

Active Expiration during exercise or forced expiration uses __________ and _______________ muscles.

abdominal internal intercostals

How can you calculate Lung compliance?

as the change in lung volume divided by the change in surrounding pressure: C = DV / DP

By the end of inspiration, what are alveolar and baromatric (air) pressures? And what about the intrapleural pressure?

both are 0 cm H20 Pi= -8 at the end of inspiration

Reduced lung elastic recoil and loss of radial traction on airways can exaggerate dynamic compression of airways, leading to _____________.

emphysema

COPD is often associated with ___ and ____ image

emphysema and bronchitis.

The NASOGASTRIC tube will go into the

go into the esophagus to the stomach. Aspirate stomach contents

The __________ the compliance is (or the steeper the slop is), the easier the lung is to inflate, and vice versa.

higher

During dynamic compression, flow is determined by alveolar pressure minus ____________ pressure.

intrapleural (not atmospheric pressure).

At the end of expiration, the only compartment that has any pressure gradient is _____.

intrapleural pressure or "Pi" = -5 cm H20 (barometric pressure and alveolar pressure are both = 0)

The tendency of the lung to recoil to its deflated volume is balanced by the tendency of the chest wall to bow out. Without the chest wall, what would happen to the alveolar pressure?

it would be the same as the atmospheric pressure = 0 cm H20.

In EMPHYSEMA most CXR are ____ but the key findings may be:

most - normal Key: Hyperinflation *Wide RETROSTERNAL SPACE* FLATTENED HEMIDIAPHRAGM BULLAE PULMONARY ARTERY HTN

With cystic fibrosis patients, what lobes are affected in the image here?

mucus plug Right upper, bronchus intermedius

The tendency of the lung to recoil to its deflated volume is balanced by the tendency of the chest wall to bow out. As a result the intrapleural pressure remains _____________. If it is not, the lung will collapse and the chest wall will spring out, which is _________________.

negative pneumothorax Note: When inside PCW=0, it is pneumothorax.

Stridor is heard mainly in the ____ population

pediatric

T/F: Increased compliance means MORE air will flow for a given change in pressure.

true

T/F: Increases in density, velocity & airway resistance make turbulence more probable.

true

T/F: Resistance of turbulent flow is much greater than laminar flow.

true

T/F: Very small bronchioles have very little resistance.

true


Related study sets

MKTG303 Exam 1 Study Guide (chapters 1-8)

View Set

ITN Module 16: Network Security Fundamentals

View Set